Book 2

Ace your homework & exams now with Quizwiz!

1284. A mother tells a clinic nurse that she has a history of eczema. Knowing this information, it is most important for the clinic nurse to assess the woman's infant for the presence of: 1. diaper rash, contact rash, seborrheic dermatitis, and eczema. 2. eczema, poison ivy rash, poison oak rash, and mite infestation. 3. scabies rash, eczema, diaper rash, and acne. 4. diaper rash, poison ivy rash, eczema, and mite infestation.

: 1 "Dermatitis" and "eczema" are terms often used interchangeably. Because eczema can be hereditary, assessment should include other common types of dermatitis, including diaper rash, contact rash, seborrheic dermatitis, and eczema. Mite infestation and scabies are unrelated to the woman's history of eczema.

1417. A hospitalized pediatric client has a blood lead level (BLL) of 50 mcg/dL and is diagnosed with acute lead poisoning. Succimer (Chemet®) 10 mg/kg oral q8h for 5 days is initially prescribed. The child weighs 20 kg. An experienced nurse should intervene when observing a new nurse: 1. administering one 100-mg succimer capsule with food. 2. opening a succimer capsule and sprinkling the beads on a small amount of food. 3. offering fluids frequently during the shift to increase the child's urine output. 4. explaining to a parent that chelation therapy removes the lead from the blood and some lead from tissues and organs.

: 1 A 20 kg child should receive two capsules (not one). Use a proportion formula and then multiply the means (inside values) and extremes (outside values) and solve for X: 10 mg : 1 kg :: X mg : 20 kg X 200 mg. Succimer capsules can be opened and sprinkled on a small amount of food or liquid to be swallowed. Fluids should be increased to prevent renal damage because succimer is excreted by the kidneys. Succimer forms a watersoluble compound with lead, allowing urinary elimination of excessive amounts of lead. Lead is removed from the blood and theoretically some lead is removed from tissues and organs.

1252. Four parents call a clinic to have their children seen for unusual lumps or swelling. A nurse is trying to work the children into a physician's overbooked schedule. Which child should the nurse schedule to be seen first? 1. A child with Down's syndrome 2. A child who lives close to power lines 3. A child who has had chronic ear infections 4. A child whose sibling was treated for an osteosarcoma

: 1 A correlation exists between some genetic disorders, such as Down's syndrome (trisomy 21), and childhood cancer. In a child with Down's syndrome, the probability of developing leukemia is about 20 times greater than that of other children. Studies have found marginally significant relationships between electromagnetic exposure and developing childhood cancer. Chronic infections do not automatically increase the risk for cancer. Studies suggest that in general there is not a strong constitutional genetic component for childhood cancers other than retinoblastoma.

1347. A nurse is preparing a 12-year-old female client for a renal/bladder ultrasound. In doing so, the nurse explains to the client: 1. that she cannot void before the procedure because a full bladder will facilitate the best visualization of important structures. 2. that she should void immediately before the procedure because a full bladder impairs the visualization of important structures. 3. that she will have to void during the procedure in order to obtain the best results. 4. that the fullness of the bladder is not relevant to the study findings.

: 1 A full bladder is important during a renal ultrasound to permit the best visualization of structures. Voiding before or during the procedure is not recommended. The fullness of the bladder is quite relevant in properly obtaining desired results.

1154. The parent of a child diagnosed with rheumatic heart disease questions the nurse following the doctor's statement that the child has a heart murmur. The nurse explains that a heart murmur is an abnormal or extra heart sound produced by which malfunctioning structure of the heart? 1. Heart valve 2. Heart vessel 3. Heart chamber 4. Heart conduction

: 1 A heart murmur is an abnormal or extra heart sound caused by an incomplete closure of the heart valve. In rheumatic fever, the heart valves are damaged by an abnormal response by the immune system. Erosion of the valves makes them leaky and inefficient, and a murmur of backflowing blood will be heard. A malfunctioning vessel, chamber, or conduction would not produce a heart murmur but would likely affect blood flow, contractility, or cardiac rhythm.

1340. An 11-year-old child is hospitalized for elective surgery. The child has a neurogenic bladder from a spinal cord injury with a lower motor neuron lesion occurring 2 years previously. In planning care, the nurse considers that the optimal treatment for neurogenic bladder in the hospitalized child is: 1. intermittent catheterization. 2. insertion of a retention catheter. 3. insertion of a suprapubic catheter. 4. administration of an anticholinergic medication.

: 1 A neurogenic bladder from a lower motor neuron lesion produces a flaccid bladder. The bladder is unable to respond to changes in passive pressure, causing overdistention. Periodic emptying of the bladder with intermittent catheterization is necessary. Some facilities may have a procedure in place for using a clean versus sterile technique because research has shown there is no difference in infection rates, and the child uses a clean technique while at home. A retention catheter is unnecessary and can increase the risk of infection. There is no indication that a suprapubic catheter is needed. An anticholinergic medication, such as dicyclomine (Bentyl®), relaxes the bladder musculature and promotes increased bladder capacity and more adequate emptying, but it is used in persons with a spastic and not a flaccid bladder.

1341. A pediatric client with a spinal cord injury undergoes range of motion exercises several times each day. In teaching the parent how to do range of motion at home, the nurse observes the client increasing the angle between the extremity and the midline. The nurse concludes that the client is safely performing: 1. abduction. 2. adduction. 3. flexion. 4. extension.

: 1 Abduction refers to increasing the angle between the extremity and the body's midline. Flexion and extension are not relative to midline. Adduction is decreasing the angle between the extremity and the body.

1186. A nurse teaches the parents of a child diagnosed with Addison's disease signs of Addisonian crisis. Which sign identified by the parents indicates that further teaching is needed? 1. Severe hypertension 2. Abdominal pain 3. Seizures 4. Coma

: 1 Addisonian crisis can lead to circulatory collapse with severe hypotension, not hypertension, Other signs include fever, generalized and pronounced weakness, dehydration, seizures, shock, and coma. Immediate intervention is needed.

1118. A school nurse is teaching adolescents about oral care. Which point should the nurse address with the students? 1. The adolescent should floss daily, brush teeth twice a day, and see the dentist two times per year. 2. The adolescent should brush once a day and see the dentist twice per year. 3. The adolescent should see the dentist once a year and brush teeth twice a day. 4. The adolescent should floss daily, brush teeth twice a day, and see the dentist once a year.

: 1 Adolescent and school-aged children should floss daily, brush teeth twice a day, and see the dentist two times per year.

1386. An albuterol nebulizer treatment is ordered for a 6-month-old infant hospitalized with laryngotracheobronchitis (LTB). A nurse should understand that albuterol (Ventolin®), when used as a nebulizer treatment: 1. relaxes smooth muscles in the airways. 2. reduces inflammation and mucosal edema in airways. 3. removes excess fluid from the lungs. 4. loosens and thins pulmonary secretions.

: 1 Albuterol binds to beta2-adrenergic receptors in the airway's smooth muscles, leading to activation of adenyl cyclase and increased levels of cyclic-3',5'-adenosine monophosphate (cAMP). Increases in cAMP activate kinases, which inhibits the phosphorylation of myosin and decreases intracellular calcium. Decreased intracellular calcium relaxes smooth muscles in the airways. Expectorants reduce inflammation by liquefying mucus and stimulating the natural lubricant fluids from the bronchial glands. Diuretics remove excess fluid by acting to increase renal function, which affects circulation, which facilitates removal of excess fluid from the lungs. Mucolytics reduce the thickness of pulmonary secretions by acting directly on the mucus plugs and dissolving them.

1193. A 12-year-old child is being treated for growth hormone deficiency. The child is angry and refusing to go to school because all the other children are taller. In addition, this child is belligerent toward the mother, who gives the daily injection of growth hormone. Which initial intervention should be attempted by the nurse? 1. Teach the child self-administration of growth hormone. 2. Refer the family for counseling with particular emphasis on anger management. 3. Assist the parents to contact the school district so that home schooling can begin and last until the child has reached normal height. 4. Talk to the mother about requesting an Individual Educational Planning (IEP) team to assist in planning school interventions.

: 1 Allowing the child to participate in the care fosters a sense of control over the problem. The family may ultimately be referred to counseling, but the anger this child is displaying is a normal reaction to seeing himself as different from other children the same age. This should decrease if participation in treatment is promoted. The child should not be removed from the school setting because that will just reinforce to the child that there is a difference. There is no need for an IEP team unless there are additional issues that could interfere with learning. At this point no additional issues have been established.

1381. A clinic nurse receives a call from a mother of a 2-year-old child. The mother states that the child has a temperature of 104°F (40°C), a sore throat, and has been drooling for a few days. The child is now sleepy. Which is the best advice by the nurse? 1. "Take your child to an emergency department immediately." 2. "Bring your child into the clinic to be seen as soon as possible." 3. "Administer acetaminophen (Tylenol®) for the temperature and allow your child to sleep." 4. "Use a spoon to look inside your child's mouth and throat and tell me what you see."

: 1 An elevated temperature and sore throat and drooling could suggest epiglottitis. The sleepiness could be from the effects of the elevated temperature or from respiratory depression. The child should be seen in an emergency department immediately because the infant could develop respiratory failure. Bringing the child to the clinic could delay emergency treatment if needed. Acetaminophen may reduce the child's temperature, but allowing the child to sleep without being assessed could be detrimental. If the epiglottis is swollen and inflamed, stimulating the gag reflex can cause complete obstruction of the glottis and respiratory failure, and it should never be checked for a person with suspected epiglottitis.

1375. A nurse caring for a pediatric client with nephrotic syndrome should anticipate administering: 1. prednisone (Liquid Pred®). 2. ibuprofen (Children's Motrin®). 3. penicillin (Pen-Vee K®). 4. hydrochlorothiazide (Esidrix®).

: 1 An immunological mechanism is involved in nephrotic syndrome. Corticosteroids reduce inflammation and proteinuria. It is given until diuresis without protein loss is accomplished. Ibuprofen is a NSAID used to treat mild inflammation. Penicillin is an antibiotic used to treat infection. Infection is not the cause. Hydrochlorothiazide is a diuretic used to treat hypertension or edema, although it is not commonly used because it tends to decrease blood volume, which is already decreased.

1214. An infant with prolonged vomiting secondary to pyloric stenosis has arterial blood gases (ABG) drawn. Which ABG results should a nurse expect when reviewing the laboratory report if the infant has an acid-base imbalance? 1. Increased pH and increased bicarbonate 2. Decreased pH and decreased bicarbonate 3. Increased pH and decreased bicarbonate 4. Decreased pH and increased bicarbonate

: 1 An infant with pyloric stenosis often has been vomiting for 1 to 2 weeks, resulting in metabolic alkalosis as evidenced by an increase in both serum pH and bicarbonate. A decreased pH and bicarbonate are indicative of metabolic acidosis, which could result if overtreated for the acid-base imbalance or another complication. An increased pH and decreased bicarbonate could be from respiratory alkalosis with compensation. A decreased pH and increased bicarbonate could be from respiratory acidosis with compensation.

1298. A 12-year-old child is seen in a clinic after being hit in the eye by a baseball. A nurse assesses gross hyphema (hemorrhage into the anterior chamber) and a visible fluid meniscus across the iris. Which physician order should the nurse anticipate? 1. Immediate referral to an ophthalmologist 2. Immediate transfer to an emergency department 3. Home treatment with application of ice for 24 hours 4. Instillation of cortisone drops and application of an eye patch

: 1 An ophthalmologist referral is necessary to evaluate for further injury. Transfer to an emergency department will delay evaluation and treatment. If hyphema was not present, then ice would be applied for 24 hours to reduce swelling. If eye contamination is suspected, antibiotic eye drops (not cortisone drops) may be ordered. The child should close and rest the affected eye. An eye patch is usually not necessary.

1407. A child of African descent has a positive acid-fast bacillus sputum culture after returning to the United States from a trip to Africa to visit relatives. During a nursing assessment, a nurse observes that the parent refers to the child's diagnosis by using the impersonal pronoun "it." Which statement made by the nurse is best? 1. "Tell me how you feel about your child's diagnosis." 2. "If your child takes the prescribed medications, 'it' can be cured." 3. "Why do you call your child's tuberculosis 'it', rather than referring to the diagnosis of tuberculosis?" 4. "I need to find out more information about 'it'. How long has your child been having night sweats and a productive cough?"

: 1 Asking the parent about feelings allows the parent time to express feelings and concerns, and a rationale may be provided for referring to the tuberculosis as "it." Some cultures avoid calling the disease by name for fear that it may cause further harm. Telling the parent that if the child takes the prescribed medication, the child will be cured is irrelevant during an assessment. Asking a "why" question is a barrier to therapeutic communication and can result in defensiveness. Although assessing the length of time the child may have had symptoms is important, this question ignores the parent's feelings.

1346. A nurse is preparing an adolescent client to conduct a 24-hour urine collection. The nurse instructs the client that the collection period always starts and ends with an empty bladder so that at the time the collection begins, the client should void and the specimen should be discarded. All urine voided in the subsequent 24 hours should be saved. Twentyfour hours after the precollection specimen was discarded, the client should void. What instruction should the nurse give to the client regarding the final specimen? 1. Add it to the container. 2. Discard it. 3. Measure it and discard it. 4. Store it separately in the refrigerator.

: 1 At the completion of the 24-hour period, the client is asked to void and the specimen is added to the container. It would be inappropriate to discard the specimen or store it separately. If the specimen was discarded, the test would need to be restarted.

1319. A child with autism has been admitted to a four-bed ward on a pediatric unit. The nurse admitting the child should: 1. request that the child be transferred to a private room. 2. request that the child be transferred to a double room. 3. admit the child to the assigned room. 4. request that the child be assigned to an isolation room.

: 1 Because autistic children are unable to relate to other children or to respond to emotional or social cues appropriately, the child should be in a private room. Autistic children can have bizarre responses to the environment, repetitive hand movement, rocking, and rhythmic body movements that can frighten other children. A double room or a fourbed ward is an inappropriate room assignment. Isolation is unnecessary since autism is not a communicable disease.

1103. A 7-year-old child lived in foster homes when he was an infant. He was adopted at the age of 1 year to an intact family who provided him with love and security. Which developmental task was this child most likely unable to complete as an infant? 1. Trust versus mistrust 2. Industry versus inferiority 3. Autonomy versus shame and doubt 4. Initiative versus guilt

: 1 Because the child lived in multiple foster homes, the child may have had delayed development in trust as an infant. The other options are not correct based on the developmental needs of an infant.

1093. Various children are being seen in the clinic for well-baby checks. By what age should a nurse expect a child to begin to use simple words to communicate needs? 1. Age 10-12 months 2. Age 1-2 years 3. Age 6-9 months 4. Age 2-3 years

: 1 By age 10 to 12 months, a child is able to communicate simple words, using four to six words by 15 months of age. By age 1 to 2 years the child communicates in more than simple words, using about 50 words in two-word sentences. At 6 to 9 months, the child is learning to make sounds. At 2 to 3 years, the child's verbal language increases steadily, knowing full name, naming a color, and holding up fingers to show age.

1109. A nurse has reviewed the upper arm blood pressure (BP) results for multiple children between the ages of 3 and 5 years. Which BP reading should the nurse evaluate as being an abnormal BP for this age group? 1. 96/42 mm Hg 2. 101/57 mm Hg 3. 112/66 mm Hg 4. 115/68 mm Hg

: 1 Children between the ages of 1 month to 2 years have a mean BP of 95/58 mm Hg. A BP reading of 96/42 mm Hg has a mean arterial pressure (MAP) of 60, which indicates insufficient perfusion to tissues. MAP = (systolic blood pressure [SBP] + 2diastolic blood pressure [DBP])/3. The mean BP for a child 2 to 5 years is 101/57 (MAP = 74). The 90th percentile BP is 112/66 mm Hg (MAP = 82). The 95th percentile is 115/68 mm Hg (MAP = 85).

1173. A child with a history of type 1 diabetes mellitus presents in the school nurse's office about an hour before the lunch period reporting disorientation. Which information is most important for the nurse to obtain? 1. Blood sugar 2. Temperature 3. Morning insulin dose 4. Urine ketones

: 1 Children who become disoriented or sleepy within an hour of a meal are most likely experiencing hypoglycemia. Urine ketones are not indicated because ketones are not spilled at low plasma glucose levels. The temperature and insulin dose will not provide the information needed to intervene.

1157. A nurse arrives at a local park to find a group of people surrounding a pediatric victim. Witnesses report the child collapsed just seconds ago. The child is not breathing and is without a pulse. While another person dials for emergency assistance, the nurse prepares to initiate single rescuer CPR. Which compression-to-ventilation ratio should be used by the nurse? 1. 30:2 2. 15:2 3. 30:1 4. 15:1

: 1 Current guidelines for single rescuer CPR have the compressionto-ventilation ratio of 30:2 for both pediatric and adult victims. All other options are incorrect.

1379. A physician orders arterial blood gases (ABGs) on a 5-year-old client admitted with severe asthma. Which signs and symptoms noted during a nurse's assessment of the child are consistent with the blood gas findings of pH = 7.30, PaCO2 = 49 mm Hg, and HCO 3 = 24 mEq/L? 1. Diaphoresis, headache, tachycardia, confusion, restlessness, apprehension, and flushed face 2. Rapid and deep respirations, paresthesia, lightheadedness, twitching, anxiety, and fear 3. Rapid and deep breathing, fruity breath, fatigue, headache, lethargy, drowsiness, nausea, vomiting, and abdominal pain 4. Slow and shallow breathing, hypertonic muscles, restlessness, twitching, confusion, irritability, apathy, tetany, and seizures

: 1 Diaphoresis, headache, tachycardia, confusion, restlessness, apprehension, and flushed face are all signs and symptoms of respiratory acidosis without compensation. These occur because of the lack of oxygen and trapping of carbon dioxide in the lower airway from the narrowed airway passages. Rapid and deep respirations, paresthesia, light-headedness, twitching, anxiety, and fear are signs and symptoms of respiratory alkalosis. Respiratory alkalosis may occur in asthma if excess artificial ventilation is used in treatment. Rapid and deep breathing, fruity breath, fatigue, headache, lethargy, drowsiness, nausea, vomiting, and abdominal pain are signs and symptoms of metabolic acidosis. Slow and shallow breathing, hypertonic muscles, restlessness, twitching, confusion, irritability, apathy, tetany, and seizures are signs and symptoms of metabolic alkalosis. Metabolic acidosis and alkalosis are not associated with asthma but may occur from other complications.

1331. A child is admitted to an emergency department with a dislocated kneecap that occurred while skiing. Which most immediate treatment by a healthcare provider (HCP) should a nurse anticipate? 1. Realignment of the kneecap by sliding it back into position in the front of the knee 2. Open surgical intervention to repair the kneecap 3. Arthroscopy to surgically repair the torn cartilage 4. Application of a cast to the affected leg until the kneecap heals

: 1 Dislocation of the kneecap causes it to move to the posterior surface of the knee. The kneecap is realigned by the HCP sliding it back into position. Open surgical intervention is unnecessary since the kneecap is movable. A kneecap dislocation in itself does not cause a torn cartilage. After realignment, a leg immobilizer is applied and used for about a week.

1365. Nursing management of a pediatric client with chronic kidney disease includes knowledge that these clients will often demonstrate normochromic, normocytic anemia primarily caused by: 1. decreased erythropoietin production. 2. increased erythropoietin production. 3. decreased hemoglobin production. 4. increased hemoglobin production.

: 1 Erythropoeitin, formed by the kidneys, stimulates red blood cell production. A decrease causes normochromic, normocytic anemia, where the size and hemoglobin content of red blood cells (RBCs) remain normal, but there are fewer RBCs. Increased erythropoietin would stimulate RBC production and not produce anemia. The hemoglobin production is not a cause of normochromic, normocytic anemia.

1362. A nurse is reviewing the laboratory report of a pediatric client suspected of having chronic glomerulonephritis. Along with proteinuria, which laboratory findings should the nurse expect? 1. Elevated blood urea nitrogen (BUN), creatinine, and uric acid levels 2. Decreased BUN, creatinine, and uric acid levels 3. Elevated BUN and creatinine and decreased uric acid levels 4. Decreased BUN and elevated creatinine and uric acid levels

: 1 Expected laboratory findings in a client with chronic glomerulonephritis include proteinuria with casts and red and white blood cells. Failing renal function is evidenced by increased BUN, creatinine, and uric acid levels due to the accumulation of the by-products of metabolism. Electrolyte alterations include metabolic acidosis, increased potassium levels, increased phosphate levels, and decreased calcium levels.

1357. A pediatric client requires clean intermittent catheterization while at home. Which early signs of infection should the nurse teach the parents to report immediately? 1. Fever, pulse in the upper range of normal, foul-smelling urine 2. Increased appetite, anuria, sweet-smelling urine 3. Tachypnea, tachycardia, hypertension 4. Mental confusion, diarrhea, dehydration

: 1 Fever, increased pulse rate, and foul-smelling urine are early signs of infection that the parent should report to the health-care provider. Changes in appetite and in the odor of urine could be signs of other conditions and need to be properly evaluated. Anuria, or the absence of producing urine, requires immediate evaluation and intervention. Elevated respiratory rate, pulse, and blood pressure could be related to infection but would not likely be the early signs parents would be instructed to monitor. Symptoms such as mental confusion, diarrhea, and dehydration would not be early signs of infection.

1136. Which method should a nurse use to assess the arterial oxygen saturation of a pediatric client? 1. Finger pulse oximetry 2. Arterial blood gases 3. Hemoglobin levels 4. Peak flow

: 1 Finger pulse oximetry should be used to assess the arterial oxygen saturation of a pediatric client. Finger pulse oximetry is a noninvasive measure of arterial oxygen saturation. The hemoglobin absorbs light waves from the sensor on the device. Because the hemoglobin absorbs light waves differently when it is bound to oxygen than when it is not, the oximeter can detect the degree of oxygen saturation in the hemoglobin. When the oxygen saturation is 95%, the PO2 is in the normal range of 80 to 100 mm Hg. Arterial blood gases are invasive, involving an arterial puncture. The hemoglobin level measures the amount of oxygencarrying protein in the blood. Peak flow determines the amount of airflow through the bronchi..

1426. A health-care provider prescribed a dose of acetaminophen (Tylenol®) according to weight recommendations for a child. The package insert reads the recommended dose is 15 mg per kilogram. What dose should a nurse give if the child weighs 48 lbs? 1. 327 mg 2. 327 mL 3. 500 mg 4. 500 mL

: 1 First, change 48 lbs into kilograms by dividing 48 by 2.2 to 21.8 kg. Next multiply 21.8 kg by 15 327 mg. Option 2 gives an answer in milliliters not milligrams. The math is not correct in option 3. In option 4 the math is not correct, and the answer is given in milliliters not the required milligrams.

1225. A nurse is using a food list to address foods to be eliminated from a child's diet when counseling a parent of a child diagnosed with celiac disease. Which foods should appear on the elimination food list? 1. Cereal containing oat, wheat, or rye and certain frozen foods 2. Breads made with potato or corn and white whole or skim milk 3. Soups, sauces, and peanut butter 4. Cereals and breads containing rice, and cottage cheese

: 1 Foods that contain gluten include wheat, rye, oats, and barley products should be eliminated. Packaged and frozen foods typically contain gluten as fillers. Potato bread or cornbread and whole, skim, or low-fat milk are acceptable. Chocolate milk and malt beverages contain gluten. Soups and sauces usually contain gluten used in the wheat flour for thickening. Peanut butter, cereals, and breads containing rice, and cottage cheese are acceptable foods.

1117. A school nurse is concerned about the lack of physical activity in the high school. The nurse has gathered data related to appropriate activity from the document Healthy People 2010: Understanding and Improving Health. Which recommendation for physical activity should the school nurse recommend for this age group? 1. Adolescents should get at least 60 minutes of physical activity daily. 2. High school students should be required to participate in physical education classes. 3. Teenagers should be exercising at least 30 minutes three to five times per week. 4. Adolescents should be exercising every day for at least 15 minutes.

: 1 Getting at least 60 minutes of physical activity daily promotes healthy living related to consistent exercise. All students are not required to participate in high school activity. Adolescents need more exercise than twice per week. Daily exercise is recommended, but it should be for at least 60 minutes for children and 30 minutes for adults.

1172. Which laboratory test results should a nurse monitor in evaluating the long-term success of a child's control of type 1 diabetes mellitus? 1. Hemoglobin A1c levels 2. Blood insulin levels 3. Blood glucose levels 4. Urinary glucose levels

: 1 Glycated hemoglobin (hemoglobin A1c) is used to measure long-term success because glycated hemoglobin provides an index of average blood glucose levels over the past 2 to 3 months. The results of the other laboratory tests provided will evaluate only short-term success.

1391. Which goal should a nurse deem as essential when caring for a 14-month-old infant with bronchiolitis? 1. Promoting and maintaining adequate hydration 2. Setting up and facilitating the use of a mist tent 3. Ensuring that antibiotics are ordered 4. Providing a cough suppressant as necessary

: 1 Hydration is very important in children with bronchiolitis (RSV) to loosen secretions, prevent shock, and maintain basic physiological needs. Independent studies have shown that mist tents offer no additional benefits such as keeping secretions moist to facilitate laboratory testing. Antibiotics do not treat illnesses with viral etiologies; they may be used to treat secondary causes but are not primary for viral diseases. Cough suppressants decrease a child's ability to effectively clear the airway.

1111. A 22-month-old toddler is walking into the examination room independently in front of the toddler's mother. Which method should the nurse plan to use to weigh the child? 1. A standing scale should be used because the toddler is able to stand independently. 2. Weigh using an infant scale because this is the method for all infants until the age of 2 years. 3. Ask the mother which would be best for the child. 4. Have the mother weigh herself and then weigh herself holding her child. Then subtract the mother's weight from the combined child and mother's weight

: 1 If the child is cooperative and able to stand, a standing scale should be used because it would be most accurate and allow for comparisons with later visits. A child's developmental ability and not age should be used to determine the most correct method to weigh the child. The mother depends on a nurse for the best suggestion. Weighing the baby and mother and then subtract the baby's weight is not as accurate.

1287. A nurse is cleansing the skin of a hospitalized child with impetigo. Which action is most important for the nurse to take? 1. Apply clean gloves to prevent the spread of the infection to others. 2. Use sterile technique to prevent any further infection of the lesions. 3. Ensure the water is cold to help reduce pain during cleansing. 4. Keep the child in contact precautions until the child is discharged.

: 1 Impetigo is transmitted by direct contact. Clean gloves are used to prevent the spread of infection to the nurse or others. Sterile technique is not needed because the infection is already present and the goal is to prevent spread of the infection to others. The water temperature should be tepid or one that is comforting to the child. Contact precautions can be discontinued 24 hours after initiation of therapy and do not need to continue during the entire hospitalization time period.

1188. Glucocorticoids are prescribed for a child diagnosed with congenital adrenal hyperplasia. Which manifestation should indicate to a nurse that therapy is successful? 1. Feminization in girls 2. Absence of symptoms of Cushing's syndrome 3. Precocious penile enlargement in boys 4. Increased growth rate in both boys and girls

: 1 In congenital adrenal hyperplasia the body lacks the enzyme to produce cortisol and aldosterone. Without these hormones, the body produces more androgen, a type of male sex hormone. The goal of therapy is to reduce virilization (masculine characteristics) in girls. The presence of Cushing's syndrome is an adverse reaction and does not indicate if therapy is successful. Precocious penile enlargement in males and increased growth rate in both girls and boys indicates the medication is not successful.

1183. Which instruction should a nurse include when teaching parents who have a child diagnosed with hypoparathyroidism? 1. Monitor for muscle spasms, tingling around the mouth, and muscle cramps. 2. Monitor for side effects of excess medication therapy, including dry, scaly, coarse skin. 3. Decrease intake of foods high in calcium and phosphorus. 4. Increase environmental stimuli and encourage participation in high-energy activities.

: 1 In hypoparathyroidism, insufficient amounts of parathyroid hormone is produced and this affects serum calcium regulation. Muscle spasms, tingling around the mouth, and muscle cramps are signs of hypocalcemia which could indicate that treatment with vitamin D and calcium is ineffective. Dry, scaly, and coarse skin are signs of hypoparathyroidism, not medication overdose. Calcium-rich foods, such as dark green vegetables, soybeans, and tofu, should be encouraged (not decreased in amounts). Foods high in oxalic acid (e.g., spinach and rhubarb) and phytic acid (e.g., bran and whole grains) should be avoided because these reduce calcium absorption. Phosphorus-rich foods (dairy products) are also high in calcium and are usually not restricted unless serum phosphorus levels are exceptionally high. During crisis episodes the environmental stimuli needs to be decreased (not increased) and the child kept quiet.

1219. A nurse is planning caring for an infant newly hospitalized with a diagnosis of intussusception. Which nursing diagnosis should the nurse establish as the immediate priority? 1. Pain related to abnormal abdominal peristalsis 2. Risk for deficient fluid volume related to bowel obstruction 3. Altered nutrition, less than body requirements related to vomiting 4. Risk for altered skin integrity related to bloody stools

: 1 In intussusception, a portion of the intestine prolapses and then telescopes into another portion; this telescoping makes peristalsis difficult and intensely painful. The walls of the intestine also may rub together, causing inflammation, edema, and decreased blood flow, all of which can increase pain. Fluid can be lost through vomiting from the intense peristaltic waves or because of lack of absorption of fluids due to obstruction. Risk for deficient fluid volume is important, but not the priority, unless this were an actual problem. Vomiting can lead to altered nutrition, which is also important, but not the priority because the pain can cause the infant to be inconsolable and delay necessary interventions. Bloody stools, described as "currant jelly," occurs from inflammation and necrosis. Risk for altered skin integrity is important, but not the priority.

1311. A woman has just undergone a prenatal screening that indicates that her child might have a neural tube defect. In response to a question about neural tube defects, a nurse describes one possible defect. Which neural tube defect is the nurse describing when stating that the vertebral arch fails to close and the spinal cord and meninges stay within the vertebral canal? 1. Spina bifida occulta 2. Spina bifida cystica 3. Meningocele 4. Myelomeningocele

: 1 In spina bifida occulta there is failure of the vertebrae to close without hernial protrusion. Because the central nervous system structures remain in the spinal canal, there are often no adverse physical sequelae, and the condition might remain undiagnosed, or hidden (occult). Spina bifida cystica is also known as myelomeningocele, and includes the protrusion of the spinal cord through incompletely arched vertebrae because of lack of union between the laminae of the vertebrae. In a meningocele, the meninges protrude, but the spinal cord is not involved

1392. A 5-month-old infant hospitalized with a diagnosis of Pneumocystis carinii pneumonia is being treated with trimethoprim-sulfamethoxazole (TMP-SMZ) with no adverse effects. A nurse assesses that the infant has thick tenacious secretions and poor cough effort, diarrhea, anorexia, and is diaphoretic. Which nursing diagnosis should the nurse establish as the priority? 1. Ineffective airway clearance related to increased secretions and inability to clear the respiratory tract 2. Altered nutrition: less than body requirements related to recurrent illness, diarrheal losses, loss of appetite, and oral candidiasis 3. Risk for infection related to impaired body defenses and presence of infective organisms 4. Risk for fluid volume deficit related to difficulty taking fluids, insensible fluid losses from hyperventilation, and diaphoresis

: 1 Ineffective airway clearance related to increased secretions and inability to clear the respiratory tract is a basic physiological need and should be addressed first. Altered nutrition: less than body requirements related to recurrent illness, diarrheal losses, loss of appetite, oral candidiasis is essential to maintaining physiological needs of an immunocompromised child. Risk for infection related to impaired body defenses, presence of infective organisms, preventing infection in an infant/child with an immunological dysfunction is key to maintaining a healthy lifestyle. Immunocompromised children are susceptible to infection and every effort should be made to keep them risk free. There is risk for fluid volume deficit related to difficulty taking fluids, insensible fluid losses from hyperventilation, and diaphoresis. Preventing risk for volume deficit is secondary to physical needs and can be prevented by maintaining fluid volume and encouraging intake.

1390. An 11-month-old infant is brought to the hospital after experiencing a respiratory infection and severe diarrhea for 5 days. The child has poor skin turgor, respirations 30, temperature 101.3°F (39°C), low serum potassium levels, and watery green stools. A physician orders an antipyretic and to begin intravenous (IV) fluid replacement of D5NS with a potassium additive. It is most important for a nurse to: 1. ensure the infant had urine output before beginning the ordered IV fluids. 2. apply oxygen because the child is experiencing rapid respirations. 3. change the infant's diaper to prevent skin breakdown. 4. administer the antipyretic first.

: 1 It is imperative to know if the infant's kidneys are functioning before administering fluid replacement with a potassium additive because excess potassium is excreted by the kidneys. High levels of excess potassium can cause life-threatening events, such as cardiac dysrhythmias. A respiration rate of 20 to 40 breaths per minute is within the expected range for an infant. Preventing skin breakdown and administering an antipyretic are also important, but they may not result in a life-threatening event.

1356. A nurse is caring for a pediatric client who will be going home with an indwelling catheter. Which important component should the nurse include in the instructions to the client and his or her parents? 1. Keeping the urine drainage bag below the level of the bladder 2. Pulling and taping the catheter securely down the length of the child's leg 3. Decreasing fluid intake to limit the need for frequent bag emptying 4. Taking tub baths in place of showers to minimize standing

: 1 Keeping the drainage bag below the level of the bladder prevents reflux of urine back into the bladder, which increases the risk of a bladder infection. Placing tension on the catheter increases the risk of injury to the urinary meatus. Fluid intake should be increased to reduce bacterial growth. Showering minimizes the potential for contaminating the urinary drainage bag.

1438. A nurse is caring for a 16-year-old client following a barbiturate overdose. The client's blood pressure is 94/48 mm Hg, heart rate 112, and urine output 200 mL for the past 8 hours. The client's cardiac rhythm is sinus tachycardia with premature atrial contractions (PACs). Dopamine 1 mcg/kg/min is prescribed. Which finding should prompt the nurse to conclude that the medication has been effective? 1. Increase in urine output 2. Increase in the diastolic blood pressure (DBP) 3. Decrease in pulse pressure 4. Decrease in PACs

: 1 Low dose dopamine, 0.5-2.0 mcg/kg/min, acts on dopaminergic receptor sites along afferent arterioles in the glomerulus, dilates the renal vasculature, and improves urine output. Positive inotropic effects of dopamine include an increase in systolic blood pressure with little or no effect on DBP and an increase (not decrease) in pulse pressure. Dopamine will have no effect on decreasing the incidence of PACs.

1334. A nurse explains to a child's parents that the role of methotrexate (Rheumatrex®) in treating children with juvenile arthritis is to: 1. decrease the inflammatory response. 2. improve functional ability. 3. control the febrile response. 4. minimize the effects of uveitis.

: 1 Methotrexate is an immunosuppressant that decreases the inflammatory response. It is categorized as a disease-modifying drug. While decreasing inflammation may improve functional ability, this is not the action of the medication. Fever may be a sign of medication toxicity. Uveitis is a nonspecific term for any intraocular inflammatory disorder and is thought to be an autoimmune phenomenon; corticosteroids are prescribed for treating uveitis.

1363. A nurse is caring for a 17-year-old client with renal insufficiency from impaired blood flow to the kidneys sustained during a motor vehicle accident. Which assessment finding related to renal insufficiency should be reported immediately to the health-care provider? 1. Oliguria 2. Dysuria 3. Frequency 4. Urgency

: 1 Oliguria, or low urine output, could indicate impaired kidney function. Dysuria, frequency, and urgency often suggest infection. Dysuria means voiding that is either painful or difficult. Urinary frequency is voiding at frequent intervals. Urgency is the sudden strong desire to void.

1266. A nurse is preparing a child for abdominal irradiation. Which medications should the nurse plan to administer to prevent nausea and vomiting? 1. Ondansetron (Zofran®) and dexamethasone (Decadron®) 2. Promethazine (Phenergan®) and cyclophosphamide (Cytoxan®) 3. Metoclopramide (Reglan®) and methotrexate (Amethopterin®) 4. Marijuana and L-asparaginase (Elspar®)

: 1 Ondansetron is an antiemetic used to control nausea and vomiting. Dexamethasone is a corticosteroid anti-inflammatory agent used in the adjunctive management of nausea and vomiting from chemotherapy. Promethazine is a phenothiazine-type antiemetic, but cyclophosphamide is a chemotherapeutic agent. Metoclopramide is an antiemetic, but it causes extrapyramidal reactions in children. Methotrexate is a chemotherapeutic agent. Marijuana is not approved for use in the United States, but synthetic cannabinoids are now being used in children. L-asparaginase is a chemotherapeutic agent.

1333. A nurse has been asked to continue teaching with a group of parents of children with neurological and musculoskeletal conditions. For which condition should a nurse tell the parents that there is no genetic basis to the condition? 1. Osteomyelitis 2. Muscular dystrophy 3. Spina bifida 4. Tourette's Syndrome

: 1 Osteomyelitis is an infection that might have long term sequelae; however, the cause is not genetic. The remaining conditions are genetic with varying inheritance patterns.

1277. A nurse is caring for a 3-year-old burn victim who is the only child of a single parent. The parent has not visited the child for 2 days, and the child is crying and says, "I want my mommy! Where is she?" The nurse calls the parent, who says, "I cannot stand to see my baby in so much pain knowing that I am responsible for this." The best response of the nurse is: 1. "It sounds like you are feeling guilty. Can you come in to talk about how we can help you and your child?" 2. "I am sorry you are feeling responsible. I just wanted to know when you could be here." 3. "It is very important that your child see you. How can I help you to get here?" 4. "Why do you think that you are responsible for this?"

: 1 Paraphrasing the mother's statements is a therapeutic response. Offering to help the mother demonstrates concern. A burn injury of a child is traumatic for the child and family. The parent acknowledges feeling responsible and may be grieving so deeply that she is unable to successfully support her child. The nurse should explore resources with the parent to assist in dealing with her own feelings so that she in turn can assist the child now and during the long course of treatments, surgeries, and rehabilitation. Option 2 acknowledges the mother's feelings, which is therapeutic, but changing the subject is a barrier to communication. Option 3 fails to acknowledge the mother's feelings. Asking a "why" question, as in option 4, is a barrier to therapeutic communication because it can initiate a defensive response.

1296. A mother tells a nurse that her 1-month-old infant does not react to light. Which response to the parent is best? 1. "You should have your infant's vision tested; I can help you with arranging an appointment." 2. "It is normal for your infant not to react to light because visual acuity improves as the infant develops." 3. "All babies react to light differently. Try seeing what your baby's response is to different lighting." 4. "It is nothing to worry about, but I will let the doctor know in case further testing is needed."

: 1 Parent concerns should be taken seriously. If the infant does not respond to light, it could indicate blindness. Visual acuity does improve with development, but a neonate is able to follow light or ceases body movement when a room is lighted. Infants respond differently based on level of alertness and age, but giving advice to try different lighting is inappropriate. The infant needs an eye examination and evaluation. Telling the mother that there is nothing to worry about is providing false reassurance.

1361. A child presents to an emergency department with periorbital edema, anorexia, and the passage of darkcolored urine. The most significant history reported by the parent related to the possible etiology is: 1. a "cold" approximately 10 days before onset of these symptoms. 2. a fall off a skateboard the night before. 3. eating fast food for lunch. 4. international travel to Europe 1 month ago.

: 1 Periorbital edema, anorexia, and dark-colored urine (often tea colored, reddish-brown, or smoky from hematuria) are the initial signs of nephrotic reaction in a child with acute glomerulonephritis (AGN). Often, the child has been in good health with no history of infection except for symptoms described as a mild cold approximately 10 days before onset. A fall, fast food, and international travel are not likely significant in diagnosing this condition.

1354. A nurse is educating the parent of an infant male client with phimosis. Which important concept should the nurse include in teaching? 1. Occasionally, the narrowing obstructs the flow of urine, resulting in a dribbling stream. 2. Once the infant is older and begins toileting, urinating a straight stream will be impossible. 3. Retract the foreskin away from the glans penis daily to prevent urinary retention. 4. Watch for enlargement of the testicles because fluid is likely to accumulate.

: 1 Phimosis is a narrowing of the preputial opening of the foreskin that prevents retraction of the foreskin over the glans penis. Occasionally, the narrowing obstructs the flow of urine, resulting in a dribbling stream or even ballooning of the foreskin with accumulated urine during voiding. Hypospadias is a condition in which the urethral opening is located below the glans penis or anywhere along the ventral surface. If not corrected, a straight stream may not be possible. Hypospadias is unrelated to phimosis. In phimosis, the foreskin is already retracted and is causing the urinary problems. A hydrocele is the presence of fluid in the processus vaginalis. Fluid does not accumulate in the testicles but can cause urinary retention.

1400. Which fact about cystic fibrosis (CF) should a nurse consider when formulating a plan of care for a child with the diagnosis? 1. Pulmonary secretions are abnormally thick. 2. CF is an autosomal dominant hereditary disorder. 3. Early in the disease, children with CF will also have diabetes mellitus. 4. Chronic constipation usually occurs in CF.

: 1 Pulmonary secretions are abnormally thick, and the lungs are filled with mucus that the cilia cannot clear. CF is an autosomal recessive (not dominant) disorder. In CF the acinar cells of the pancreas producing lipase, trypsin, and amylase are plugged, causing atrophy of the acinar cells and an inability to produce the enzymes. The islets of Langerhans and insulin production are not involved until late in the disease because the islets have ductless activity. Children with CF characteristically have large, bulky, frothy, and foul-smelling stools from the inability to digest fats and protein.

1146. A nurse is managing the care of an infant with an unrepaired heart defect. Which health promotion strategy should the nurse recommend to the parent in planning for discharge? 1. Vaccinate against the respiratory syncytial virus (RSV) monthly during the RSV season. 2. Restrict the child's level of physical activity. 3. Encourage weight loss by restricting caloric intake. 4. Delay immunizations as the child's immune system may be too impaired.

: 1 RSV vaccine is administered monthly during RSV season to infants with unrepaired heart defects. Children generally do not need to restrict activity and, in most cases, are anorexic and require high-nutrient foods. Infants should receive scheduled childhood immunizations according to current guidelines.

1236. After 7 days of iron therapy, a child diagnosed with iron-deficiency anemia has serum laboratory tests completed. Which finding indicates that the medication is beginning to correct the anemia? 1. Increased reticulocyte count 2. Increased granulocytes 3. Increased indirect bilirubin 4. Increased erythropoietin levels

: 1 Red blood cells (RBCs) are small and pale because of the stunted hemoglobin. Administering iron increases the production of RBCs. A reticulocyte count measures how fast the reticulocytes (slightly immature RBCs) are made by the bone marrow and released into the blood. Granulocytes (neutrophils, basophils, and eosinophils) are a type of white blood cell filled with microscopic granules and increase with infection or inflammation. Indirect bilirubin increases from the breakdown of the heme portion of RBCs. RBCs should not be breaking down. Erythropoietin is a hormone produced by the kidneys that is stimulated whenever a child has tissue hypoxia. If anemia is correcting, hypoxia should not occur and erythropoietin production not stimulated.

1226. A child is diagnosed with early hypovolemic shock following surgical intervention for a ruptured appendix. Which nursing assessment findings support this diagnosis? 1. Tachycardia, capillary refill greater than 2 seconds, cold extremities, and weak distal pulses 2. Bradycardia, hypotension, mottled color, and weak distal pulses 3. Irritability and anxiousness, capillary refill greater than 2 seconds, and absent distal pulses 4. Lethargy, cold extremities, decreased urine output, and absent distal pulses

: 1 Signs and symptoms of early hypovolemic shock are tachycardia, increased respiratory rate, capillary refill time greater than 2 seconds, weak distal pulses, pallor or mottled color, cold extremities, blood pressure often normal for age, and decreased urine output (less than 1 to 2 mL/kg/hr in newborns and less than 0.5 to 1.0 mL/kg/hr in other age groups). Bradycardia and absent distal pulses are associated with a later stage of hypovolemic shock.

1327. A nurse at the high school works with the trainers to develop early identification of injuries. The nurse teaches the trainers that adolescent soccer players are at increased risk for: 1. varus knee deformities. 2. valgus knee deformities. 3. varus ankle deformities. 4. valgus ankle deformities.

: 1 Soccer players are at risk for varus knee deformities because of the planting and leaning involved in the game. A varus deformity means that the structures are further apart than expected; valgus means they are closer.

1242. A new nurse is telling an experienced nurse about treatments that a physician discussed with the parents of a child who has thalassemia major. Which statement by the new nurse should the experienced nurse question? 1. "Plasmapheresis will help remove the toxins that are destroying the red blood cells." 2. "Blood transfusions will need to be administered about every 2 to 4 weeks." 3. "A splenectomy may become necessary to reduce the child's abdominal discomfort." 4. "Bone marrow stem cell transplant can possibly cure this child's thalassemia major."

: 1 Thalassemia major occurs from a deficiency in the synthesis of the beta-chain of the hemoglobin resulting in defective hemoglobin that damages the red blood cells. Plasmapheresis, selective removal of the plasma, would be ineffective in treating thalassemia because the cells are not being destroyed by toxins in the plasma. Blood transfusion, splenectomy, and bone marrow stem cell transplant are all treatments for thalassemia major. Bone marrow stem cell transplantation can offer a cure.

1322. Which screening test is a neonatal nurse likely to use to detect developmental dysplasia of the hip (DDH)? 1. Barlow's maneuver 2. Pavlik's maneuver 3. Gower's maneuver 4. Allis's maneuver

: 1 The Barlow maneuver is performed by adducting the hip while pushing the thigh posterior. If the hip goes out of the socket, it is called "dislocatable" and is positive for DDH. A Pavlik harness is used to treat DDH. Gower's maneuver is used by children with muscular dystrophy to stand. An Allis's maneuver is used with children who can respond to positional instruction.

1220. A nurse is preparing a 4-month-old infant diagnosed with intussusception for surgery when the infant passes a normal brown stool. What is the nurse's most important action? 1. Notifying the health-care provider (HCP) 2. Palpating the infant's abdomen 3. Documenting the character of the stool 4. Checking the stool for the presence of blood

: 1 The HCP should be notified because the passage of a normal brown stool may indicate reduction of the intussusception and the course of treatment may be altered and surgery cancelled. Palpating the infant's abdomen may indicate a change in the presence of a palpable mass and is important before notifying the HCP but not the most important action. The stools of intussusception are currant jelly-like in appearance from the mixture of mucous and blood due to inflammation and mechanical rubbing with the intussusception, and they now have changed to a normal appearance indicating the intussusception has been reduced. The documentation of the stool appearance and checking for blood in the stool are important but not the most important action.

1116. A mother brings her 5-month-old to the clinic for a well-child appointment. A nurse is doing an assessment when the mother asks when she can give her baby solid foods. What response is most appropriate? 1. Inquire if the baby can sit well with support and if the baby's tongue thrust has decreased. 2. Ask the mother is she feels the baby is ready for solids. 3. Ask the mother if the baby seems hungry after bottle feeding. 4. Tell the mother to ask the pediatrician.

: 1 The baby needs to be physically and developmentally ready for solid foods. The baby should be able to sit up well and should have less tongue thrust so the food can stay in the mouth. The mother depends on information from the health-care providers (HCP) to help her with the decision. The criterion of a baby being hungry after bottle feeding may just mean the baby needs to take more formula or breast milk. The HCP can help with the decision, but the nurse has responsibility to conduct teaching within the registered nurse's scope of practice.

1181. A nurse is educating the parents of a school-aged child newly diagnosed with hyperthyroidism. Until the disease is under control, which instruction should be included in the education provided by the nurse? 1. Discontinue physical education classes at school. 2. Increase stimulation in the school environment. 3. Restrict the number of calories from carbohydrate foods. 4. Dress your child in cold weather clothing even in warm weather.

: 1 The child should avoid any vigorous activity and unnecessary external stimulation and should wear clothing that is comfortable. The child may experience heat intolerance and should be provided with warm weather clothing, even in winter. The child's appetite may increase, but the child will continue to lose weight until the problem is controlled.

1221. A nurse is taking the history from a parent of an infant diagnosed with Hirschsprung's disease. Which statement is the parent most likely to make? 1. "My baby has ribbonlike stools that have a foul smell." 2. "My baby has projective vomiting and swollen arms and legs." 3. "My baby has gained weight faster than my other children." 4. "My baby cries for 4 hours every evening with leg and fist clenching."

: 1 The common features of Hirschsprung's disease are ribbonlike, foulsmelling stools, chronic constipation, and abdominal distention. These occur from the absence of ganglion cells in affected bowel segments and resulting lack of peristalsis and loss of internal sphincter relaxation. Projective vomiting may occur from abdominal distention. Swollen arms and legs are not characteristics of Hirschsprung's disease. Weight loss and failure to thrive occurs due to malabsorption of nutrients. Recurrent evening crying with leg and fist clenching is more typical of colic, which is associated with increased peristalsis.

1360. A nurse caring for a hospitalized 10-year-old child writes a nursing diagnosis Altered urinary elimination. Which outcome should the nurse include? 1. Urinates six to eight times per day 2. Enuresis diminishes to every other day 3. Related to diminished excretory function of the kidney 4. Ambulates to the bathroom independently

: 1 The elimination system reaches maturity during school age, so the child should urinate six to eight times per day. Enuresis is no longer expected in this age group. A diminished excretory function may be the cause of the altered urinary elimination. If so, it should be included as part of the nursing diagnosis. It is not an outcome. Ambulating to the bathroom independently is an outcome for a nursing diagnosis pertaining to mobility and not urinary elimination.

1323. A nurse is educating a family whose child is newly diagnosed with scoliosis. The nurse explains that the goal of therapy is to: 1. limit or stop progression of the curvature. 2. prepare the child for surgery. 3. minimize the psychosocial complications of prolonged immobilization. 4. develop a pain management protocol that will minimize complications of medications.

: 1 The goal of screening is to limit progression of the curve, obviating the need for more aggressive interventions. Not all children with scoliosis need surgery, nor do most require prolonged immobilization. Most do not require aggressive pain management.

1272. A nurse is developing an educational program targeting parents of toddlers. The nurse should be able to present information aimed at preventing the majority of burn injuries in toddlers. Which is the most common cause of burns in toddlers? 1. Pulling pans of scalding liquid from the stove 2. Touching a curling iron 3. Burns from flames 4. Exposure to lighted candles

: 1 The most common burn injury in the toddler population is scalding. Parents need to be educated regarding pots and pans on the stove, coffee pot cords hanging down that a toddler could pull, and hot liquid scalding the child. Burns from heat and flames are typically seen in an older child.

1271. A nurse is caring for a 5-year-old child with secondary burns over 40% of the body. The child has just been diagnosed with disseminated intravascular coagulation (DIC). Which is the priority nursing diagnosis based on the most recent condition? 1. Ineffective tissue perfusion 2. Impaired urinary elimination 3. Risk for deficient fluid volume 4. Impaired physical mobility

: 1 The most important consideration with this child is to ensure there is adequate tissue perfusion. Impaired urinary elimination, risk for deficient fluid volume, and impaired physical mobility are accurate nursing diagnoses, but not the priority in the acute phase of DIC.

1206. An experienced nurse is observing a new nurse providing care to an 11-month-old child who is 12 hours postoperative from a cleft palate repair. Which nursing action requires the experienced nurse to intervene? 1. Using a suction catheter to remove oral secretions 2. Feeding soft, blended foods 3. Removing an arm restraint to check the skin 4. Administering an analgesic

: 1 The nurse should avoid allowing objects to be placed in the infant's mouth because it can disrupt the suture line and cause bleeding or injury. It is important the infant has blended food (no hard foods such as crackers or ice chips). The palate needs to heal and hard foods may disrupt the sutures in the mouth. The arm restraints can be removed periodically. When they are on for longer periods of time, the arms need to be evaluated for possibly skin breakdown. Pain management is important, and pain medication should be administered especially the first 24 hours.

1158. An emergency department nurse receives a pediatric client who has just been in ventricular fibrillation (VF) and has been defibrillated. The nurse is informed that the child is currently in normal sinus rhythm (NSR) with a blood pressure of 95/51. Which action should be taken by the nurse? 1. Carefully assess the child's cardiac status. 2. Prepare for cardioversion. 3. Begin CPR. 4. Prepare the client for transfer to the pediatric unit.

: 1 The nurse should carefully assess the child's cardiac status. VF is a medical emergency that was quickly corrected with defibrillation to prevent death. In VF the fibrillating ventricles are unable to pump blood and cardiac output decreases sharply. Though the rhythm has converted, the child is unstable and needs further assessment and monitoring. Cardioversion is used for atrial fibrillation, not VF. CPR is not indicated at this time. The child's heart rhythm should be monitored, so the child would likely be transferred to a pediatric unit with telemonitoring.

1428. A nurse is preparing to administer morphine sulfate intravenously for a 6-year-old child in severe pain. The child has an intravenous (IV) infusion of D5W at 50 mL/hr through a peripherally inserted central catheter (PICC). Which action is best for the nurse to take to administer the medication? 1. Dilute the morphine sulfate with 5 mL of sterile water and administer over 5 minutes into the existing IV tubing's medication port closest to the client. 2. Administer the morphine sulfate undiluted into the existing IV tubing's medication port closest to the client. 3. Question the prescribed medication because morphine sulfate cannot be administered through a central line. 4. Disconnect the infusion, inject 3 mL of normal saline, and administer the morphine sulfate undiluted.

: 1 The nurse should dilute the morphine sulfate before administration to prevent too-rapid administration and adverse effects. A single dose should be given over 4 to 5 minutes. To avoid too-rapid administration, a syringe pump should be used. Administering undiluted morphine sulfate to a child increases the risk of adverse effects. Morphine sulfate can be administered into a PICC access device. Unnecessary IV disconnections increase the risk for infection. Morphine sulfate is compatible with D5W.

1179. An infant diagnosed with hypothyroidism is prescribed levothyroxine sodium (Synthroid®). Which independent nursing intervention would assist the nurse in evaluating the effectiveness of this medication? 1. Monthly assessments of growth and development 2. Monthly serum calcium and thyroxin levels 3. Bimonthly catecholamine levels and electrocardiogram (ECG) 4. Absence of thyroid excess

: 1 The nurse should independently monitor the medication's effectiveness by assessing for normal growth and development. If the medication is effective, the child will grow and develop at a normal rate for age. Serum calcium, thyroxin, catecholamines, and ECG require medical prescription. Assessing the absence of thyroid excess is too vague.

1162. A sexually active female adolescent has been diagnosed with hyperlipidemia. After several months of lifestyle changes, the levels have not significantly decreased, and a statin medication is prescribed. A nurse is educating the client about this class of medications. Which instruction should be included in the nurse's teaching plan for this client? 1. Discontinue therapy and contact the prescriber if having new-onset muscle aches or dark urine. 2. Continue therapy even if she becomes pregnant. 3. Take the medication in the morning because it would be most effective. 4. Discontinue lifestyle modifications because these are ineffective in treating the condition.

: 1 The nurse should instruct the client to discontinue therapy and contact the prescriber if having muscle aches or dark urine. Muscle aches and dark urine could be a sign of a dangerous side effect of statins, rhabdomyolysis, which is the rapid breakdown of skeletal muscle tissue due to the chemical effects of the medication. The drug should be discontinued immediately and the prescriber notified. The drug should be discontinued if the client becomes pregnant because the drug is teratogenic. Oral contraceptive therapy should be initiated in conjunction with a statin in clients who are at risk for becoming pregnant. Statins are most effective if taken in the evening. The client should proceed with lifestyle modifications as these are to be used in conjunction with statin therapy in order to receive the greatest benefit of therapy.

1141.A nurse is caring for a pediatric client immediately following a permanent pacemaker placement. Which intervention should be the nurse's first priority for this client? 1. Initiate continuous electrocardiogram (ECG) monitoring. 2. Administer only non-narcotic analgesic medications to avoiding masking signs and symptoms of complications. 3. Transport the child to the radiology department for a chest x-ray. 4. Administer antibiotic therapy to prevent infection.

: 1 The nurse's first priority should be to initiate continuous ECG monitoring. Continuous ECG monitoring during the recovery phase is important to assess pacemaker function immediately following placement. Analgesics, including narcotics, are administered as needed to control pain. A chest x-ray is performed within 24 hours for future comparison, but it is not the priority. While the nurse should carefully monitor the site for signs of infection; prophylactic antibiotic therapy is not the priority.

1133. A nurse is caring for a pediatric client who has congestive heart failure (CHF). The client is receiving digoxin therapy. Which laboratory test result is most important to evaluate when preparing to administer digoxin? 1. Serum potassium levels 2. Serum magnesium levels 3. Serum sodium levels 4. Serum chloride levels

: 1 The serum potassium level is the most important result when preparing to administer digoxin. Hypokalemia increases the risk of digoxin toxicity and life-threatening dysrhythmias. Though important, magnesium, sodium, and chloride levels are not as essential.

1104. A clinic nurse is completing a school physical on an adolescent girl. The girl is concerned because she is 13 years old and has not yet started menstruating. Which statement by the nurse should be most helpful when addressing the girl's concerns? 1. "The average age for a girl to experience menarche is 12.5 years. That means some girls will be younger and some will be older than 12.5." 2. "Don't worry about it; your period will come." 3. "I can see why you are concerned, since some girls get their period when they are 10 years old." 4. "I can refer you to a specialist who can answer your questions."

: 1 This response most directly answers the girl's question. An average age means some will be younger and some will be older. It is demeaning to make comments such as "don't worry about it." It is unnecessary to remind the client when some girls get their periods. There is no need for a specialist, because the girl is within the normal age range for the onset of menarche.

1446. A nurse is preparing to administer intravenous (IV) fluids to a 13-kg child who has been diagnosed with dehydration secondary to gastroenteritis. The daily fluid requirement to administer IV fluid is 1,000 mL 50 mL/kg weight more than 10 kg. The correct hourly rate to administer the IV therapy is: 1. 48 mL. 2. 54 mL. 3. 62.5 mL. 4. 69 mL.

: 1 To calculate the correct hourly rate, it first must be determined what the daily fluid requirement is for this child. Because this child is 13 kilograms, the daily requirement is 1,000 mL plus 150 mL (50 mL 3 150 mL) for a total of 1,150 mL in 24 hours. 1,150 mL is divided by 24 hours, thus 48 mL is the correct hourly rate. All other rates are incorrect.

1444. A nurse records an infant's weight as 16 pounds 8 ounces at a clinic visit. Ranitidine (Zantac®) is prescribed for the infant. The prescription reads, "Give ranitidine 2 mg/kg twice daily." The medication is supplied as 15 mg/mL. The nurse should instruct the parent to withdraw ____ mL in the syringe to administer one dose.

: 1 Use a proportion formula to calculate the dose. First, determine the dose for the child's weight. 16 pounds 8 ounces 16.5 pounds divided by 2.2 pound/kg 7.5 kg Next, multiply the weight times the prescribed dose. 7.5 kg 2 mg 15 mg. The medication is supplied in 15 mg per 1 mL. The dose to administer is 1 mL.

1167. A pediatric nurse is administering metformin (Glucophage®) to a child at risk for developing type 2 diabetes mellitus. The nurse understands that an important use of metformin in children is to: 1. delay the development of type 2 diabetes mellitus in high-risk children. 2. restore fertility in adolescent females. 3. reduce blood sugars in children who have type 1 diabetes mellitus. 4. restore renal function in children who have type 1 diabetes mellitus.

: 1 When a family member has been diagnosed with type 2 diabetes mellitus, children in the family who are overweight will often be given metformin and placed on a weight-reduction diet.

1330. A child who is crying and in pain is assessed by a school nurse. The child describes an injury in which another student twisted the child's right arm. The nurse should: SELECT ALL THAT APPLY. 1. elevate and apply ice at the site of the child's injury. 2. wrap the child's arm with an elastic bandage. 3. notify the child's parent. 4. call the child's health-care provider (HCP). 5. send the child for an x-ray of the arm. 6. have the student who caused the injury come to the nurse's office for questioning.

: 1, 2, 3 A muscle strain is caused by excessive stretching of a muscle-tendon unit from an antagonist muscle group, an external object/person, or active muscle contraction. Rest, ice, compression, and elevation (RICE) are the initial treatments immediately after an injury. Ice reduces edema at the site, and an elastic wrap provides firm support. The parent should be notified to take the child to a HCP. The parent should contact the child's HCP, not the school nurse. The school nurse's scope of practice does not include prescribing; a HCP needs to request an x-ray. Appropriate channels should be used for reporting the incident rather than calling the offending student to the nurse's office.

1110. A nurse has reviewed the upper arm blood pressure (BP) results for multiple children between the ages of 3 and 5 years. Which BP reading should the nurse evaluate as being an abnormal BP for this age group? 1. 96/42 mm Hg 2. 101/57 mm Hg 3. 112/66 mm Hg 4. 115/68 mm Hg

: 1, 2, 3 An appropriately sized cuff should be used on an infant because a too-large cuff results in a lower blood pressure and a too-small cuff a higher blood pressure reading. The arm should be covered two-thirds or less with the cuff. A Doppler device can bounce off high frequency. The readings of the upper arm and thigh should be equal or close to equal in an infant. If they are not equal, it could be a sign of coarctation of the aorta. In coarctation of the aorta, the blood pressure in the upper extremities is higher (more than 15 mm Hg) than in the lower extremities.

1187. A nurse instructs the parents of a child diagnosed with Addison's disease. Which instructions should be included by the nurse? SELECT ALL THAT APPLY. 1. Have the child wear a medical alert bracelet. 2. Encourage the child to ingest adequate fluids, particularly on hot summer days. 3. Include emergency cortisone treatment for Addisonian crisis on the school medical care plan. 4. If the child vomits the dose of cortisone within 1 hour, the dose is not repeated but the healthcare provider notified. 5. Administer epinephrine subcutaneously immediately if Addisonian crisis should occur.

: 1, 2, 3 The child should wear identification at all times so that personnel can identify potential problems in emergency situations. The school medical plan should include emergency treatment of the condition and the school nurse instructed on appropriate emergency measures. The child with Addison's disease is highly susceptible to dehydration. If the child vomits the medication within 1 hour, the dose is repeated. Epinephrine is used for allergic reactions, such as bee stings, and not for Addisonian crisis.

1125. A nurse is assessing an infant for attachment behavior with a parent. Which observations are important in assessing this relationship? SELECT ALL THAT APPLY. 1. The kind of body contact between the parent and infant 2. If the parent is holding and cuddling the infant 3. The kind of comfort techniques being used by the parent 4. The comfort level of the parent while interacting with the baby 5. Whether the infant is crying

: 1, 2, 3, 4 Body contact, holding and cuddling, comforting techniques, comfort in the interaction, and the infant's temperament can affect attachment. The infant and parents will both show an ability to interact if their relationship is healthy. The fact that the infant may be crying is not a sole indication of the relationship between the infant and the parent; however, the way the parent interacts with the crying infant is an indication of this relationship.

1151. A nurse is providing discharge teaching to the parents of a pediatric client following cardiac surgery. Which information should the nurse include in the discharge teaching? SELECT ALL THAT APPLY. 1. Action and side effects of medications 2. Care of the incision and circumstances in which to contact the health-care provider 3. Activity restrictions and follow-up appointments 4. Age-appropriate diet with vitamin C to promote wound healing 5. Prevention of pericarditis following dental procedures and prophylactic antibiotic use

: 1, 2, 3, 4 Discharge teaching would include medications, wound care, activity, appointments, dietary guidance, and circumstances in which to contact the health-care provider. Discharge instructions should include the importance of prophylactic antibiotic therapy prior to dental procedures to prevent bacterial endocarditis (not pericarditis).

1112. An experienced nurse is orienting a new nurse to the care of children in a clinic. Which immunizations should the experienced nurse inform the new nurse to plan to administer to normally healthy children between ages 1 and 5 years? SELECT ALL THAT APPLY. 1. Inactivated poliovirus 2. Diphtheria, tetanus, pertussis (DTaP) 3. Measles, mumps, rubella (MMR) 4. Hepatitis B (HepB) 5. Meningococcal

: 1, 2, 3, 4 Inactivated poliovirus dose three is given between ages 12 and 18 months; dose four between ages 4 to 6 years. DTaP dose four is given between ages 12 and 18 months; dose five between ages 4 to 6 years. MMR dose one is given between ages 12 and 15 months; dose two between ages 2 to 6 years. Other immunizations include varicella zoster (chickenpox; VZV), given between ages 12 and 18 months or at any age after if the child has never had chickenpox, and Haemophilus influenza (Hib), whose dose four is given between ages 12 and 15 months. Meningococcal polysaccharide vaccine (MPSV4) is administered to children aged 2 to 10 years with terminal complement deficiencies or anatomic or functional asplenia and certain other high-risk groups.

1134. A nurse is caring for a pediatric client recently diagnosed with hypertension. Which diagnostic tests should the nurse anticipate being ordered for this client? SELECT ALL THAT APPLY. 1. Complete blood count (CBC) 2. Serum chemistry 3. Renal ultrasound 4. Drug screen 5. Glucose tolerance test (GTT)

: 1, 2, 3, 4 The nurse should anticipate that a CBC, serum chemistry, renal ultrasound, and drug screen may be ordered for this client. A CBC would be important to rule out anemia and a serum chemistry to evaluate for altered blood urea nitrogen, creatinine, and electrolytes, which could be consistent with chronic renal disease secondary to hypertension. A renal ultrasound may help identify a renal scar, congenital anomaly, or disparate renal size. Drug screening is important to identify substances associated with hypertension. If a metabolic condition such as diabetes mellitus is suspected as a causative factor, a fasting serum glucose level should be drawn and, based on the test results, the need for a GTT should be determined

1222. Which assessment findings might a nurse observe when assessing a neonate diagnosed with an anorectal malformation? SELECT ALL THAT APPLY. 1. Ribbonlike stools 2. Stenosed anal opening 3. Vomiting 4. Abdominal distention 5. Meconium in the urine 6. Poorly developed anal dimple

: 1, 2, 3, 4, 5 Ribbonlike stools may occur with some malformations. The rectal opening can be narrowed or stenosed. Vomiting and abdominal distention can occur because stool has not passed. Meconium in the urine could indicate a fistula between the colon and urinary tract. A poorly developed anal dimple is suggestive of an anorectal malformation.

1254. A nurse is completing a health history and an assessment for a male adolescent client tentatively diagnosed with Hodgkin's lymphoma. Which findings should the nurse conclude support this diagnosis? SELECT ALL THAT APPLY. 1. Firm, nontender lymph node enlargement in the axillary area 2. Drenching night sweats 3. Unexplained fever with temperatures above 100.4°F (38°C) for 3 consecutive days 4. Unexplained weight loss of 10% or more in the previous 6 months 5. A diet consisting mostly of seafood and saturated fats 6. A brother who was also diagnosed with Hodgkin's disease when an adolescent

: 1, 2, 3, 4, 6 Hodgkin's disease is characterized by painless enlarged lymph nodes most commonly in the cervical area and less frequently in the axillary and inguinal areas. Drenching night sweats, unexplained fever with temperatures above 100.4°F (38°C) for 3 consecutive days, and unexplained weight loss of 10% or more in the previous 6 months are three symptoms that are recognized by the Ann Arbor Staging System as having prognostic significance in Hodgkin's disease. There is a genetic predisposition with an increased incidence among same-sex siblings. There is no conclusive association between dietary habits and the development of Hodgkin's disease.

1131. A new nurse is managing the care of a pediatric client preparing for a cardiac catheterization under the supervision of an experienced nurse. Which factor identified by the new nurse demonstrates an understanding of the information that can be collected during cardiac catheterization? SELECT ALL THAT APPLY 1. Oxygen saturation of blood within the chambers and great vessels 2. Pressure of blood flow within the heart chambers 3. Cardiac output (CO) 4. Anatomic abnormalities 5. Ankle brachial index (ABI) 6. Ejection fraction

: 1, 2, 3, 4, 6 In cardiac catheterization, a small radiopaque catheter is passed through the major vein in the arm, leg, or neck into the heart. Blood specimens can be obtained to determine oxygen saturation levels, and contrast dye can be injected for angiography and to assess for anatomic abnormalities such as septal defects or obstruction of flow. Pressure of blood flow in the heart chambers, CO, stroke volume, and ejection fraction can be evaluated during the procedure. ABI is a ratio of the ankle systolic pressure to the arm systolic pressure and an objective measurement of arterial disease that quantifies the degree of stenosis. It is not related to a cardiac catheterization procedure.

1301. A nurse is caring for a 4-year-old child who has drainage from the ears following insertion of tympanostomy tubes. Which interventions should the nurse include when caring for the child? SELECT ALL THAT APPLY. 1. Administer acetaminophen (Tylenol®) for mild pain. 2. Apply an ice compress over the ear. 3. Cleanse the exterior canal with sterile cotton swabs. 4. Insert ear plugs. 5. Apply moisture barriers to the ear lobes.

: 1, 2, 3, 5 Analgesics, such as acetaminophen and ibuprofen, are used for minor pain, and codeine can be used for more severe episodes. Ice compresses are used to decrease inflammation. Keeping the exterior clean and dry prevents excoriation. Moisture barriers such as Aqua Vista® and Vaseline® can be applied to prevent excoriation from the exudate. Ear wicks, and not ear plugs, should be inserted to facilitate drainage and should be kept clean and dry and changed frequently as needed to help prevent infection from spreading to the mastoid process.

1300. Which signs and symptoms should a nurse expect when assessing a toddler diagnosed with acute otitis media (AOM)? SELECT ALL THAT APPLY. 1. Fussy, restless 2. Irritable, crying 3. Pulling at the affected ear 4. Rhinitis, cough, and diarrhea 5. Rolls head from side to side

: 1, 2, 3, 5 Fussiness, restlessness, irritability, crying, pulling on the affected ear, and rolling the head from side to side are classic signs and symptoms of acute otitis media from the pain and pressure in the ear. Rhinitis, cough, and diarrhea are symptoms that occur with otitis media effusion and are not usually associated with AOM

1139. A nurse is taking a history on an adolescent client who has a new onset of hypertension. The nurse is aware that a history of substance abuse may contribute to this condition and questions the adolescent. Which abused substances acknowledged by the adolescent could contribute to hypertension? SELECT ALL THAT APPLY. 1. Amphetamines 2. Cocaine 3. Hallucinogens 4. Alcohol 5. Ecstasy 6. Marijuana

: 1, 2, 3, 5, 6 Amphetamines, cocaine, hallucinogens, ecstasy, and marijuana may cause hypertension. Cocaine is a powerful vasoconstrictor that can lead to hypertension. Amphetamines and ecstasy (3, 4-methylenedioxymethamphetamine) are powerful sympathetic stimulants that may mimic the action of cocaine. Hallucinogens, such as LSD and PCP, will also often cause hypertension through autonomic stimulation. Alcohol is a depressant and is the least likely to contribute to new-onset hypertension.

1383. A 3-year-old child, admitted with sudden onset of fever, lethargy, dyspnea, sore throat, and difficulty swallowing, has a suspected diagnosis of acute epiglottitis. A nurse's initial assessment should focus on: SELECT ALL THAT APPLY. 1. reviewing the vital sign results. 2. reviewing the past medical history (PMH). 3. auscultating the lungs. 4. obtaining a throat culture. 5. observing swallowing. 6. allaying anxiety and fear.

: 1, 2, 3, 5, 6 Vital signs and a focused assessment of body systems are needed to establish a baseline for further assessment, and the PMH is required for prior health information. While assessing, the nurse should minimize the child's anxiety and fear. A throat culture is contraindicated in a diagnosis or suspected diagnosis of epiglottitis because performing a throat culture may cause a complete airway obstruction.

1395. A nurse is caring for a newly admitted 4-year-old client diagnosed with asthma who is pale, and has dry mucous membranes, cracked lips, and nasal flaring with inspiration. Which actions should the nurse perform? SELECT ALL THAT APPLY 1. Obtain a pulse oximetry 2. Obtain vital signs 3. Assess lung sounds 4. Administer a nebulizer treatment 5. Offer oral fluids 6. Elevate the head of the bed

: 1, 2, 3, 6 Obtaining a pulse oximetry and assessing vital signs and lung sounds provide important information to assess the respiratory status and can safely be performed within the scope of nursing practice. Elevating the head of the bed promotes oxygenation. Before administering any medication or treatment, such as a nebulizer treatment, a nurse should determine if the treatment or medication has been ordered by a physician or other care provider. Before offering oral fluids, a nurse should check to see if oral fluids can be given.

1176. A nursing assistant reports to a nurse that a 4-year-old child diagnosed with type 1 diabetes consumed 1/2 cup of oatmeal, 60 mL of orange juice, and 60 mL of milk for breakfast. The child's blood glucose was 150 mg/dL before breakfast, and the child did not receive insulin before breakfast. The nurse should conclude that: SELECT ALL THAT APPLY. 1. the total volume of fluid intake should be recorded as 120 mL. 2. insulin will need to be administered to cover for the carbohydrates eaten. 3. insulin will not be needed because the child's blood glucose was normal before breakfast. 4. a double-check of the amounts of carbohydrate eaten is needed before administering insulin. 5. the child should have received insulin before breakfast because the blood sugar is elevated.

: 1, 2, 4 The nurse should record the fluid intake of 120 mL for the child. Because insulin is not being produced in type 1 diabetes, insulin is needed to transport the glucose into the cells. A double-check is done for all medications that have a high potential for error. For insulin, this includes calculating the amount of carbohydrates eaten as well as the insulin dose. While the blood glucose of 150 mg/dL is within the normal before-meal target range of 100 to 180 mg/dL for children under 6 years, insulin is still needed to transport the glucose into the cells. A serum glucose of 150 mg/dL is within the normal range.

1250. A child is neutropenic due to chemotherapy treatments. Which instructions should a nurse include when preparing the parents to take the child home? SELECT ALL THAT APPLY. 1. Prohibit visitors who have been recently vaccinated. 2. Keep the child's immediate surroundings free of plants and flowers. 3. Provide items such as goldfish, television, sanitized toys, or books for your child's playtime. 4. Arrange for the child to sleep alone, preferably in his or her own room. 5. Be sure the child bathes or showers daily. 6. Take your child's temperature, respiratory rate, and pulse four times daily.

: 1, 2, 4, 5 If exposed to a recently vaccinated person, especially those vaccinated with a live virus, the child would be unable to mount an immune response and could develop an infection. Plants and flowers could harbor mold spores. Sleeping alone prevents exposure to those who may be developing an illness. Bathing or showering removes bacteria from the skin. Goldfish in the home should be placed in an area that is off limits to the child because they harbor mold spores. Temperature, respiratory rate, and pulse should be done daily; four times a day is unnecessary.

1364. For which associated complications should a nurse plan to monitor the pediatric client with chronic kidney disease? SELECT ALL THAT APPLY. 1. Dehydration 2. Hypocalcemia 3. Metabolic alkalosis 4. Bone disease 5. Anemia 6. Obesity

: 1, 2, 4, 5 The few functioning nephrons cannot reabsorb enough sodium to maintain a functioning level of body fluid, so dehydration occurs. Hypocalcemia and hyperphosphatemia occur due to the kidney's inability to excrete phosphate. Osteodystrophy (bone disease) occurs as calcium is withdrawn from bones to compensate. Erythropoeitin, which is formed by the kidneys, stimulates red cell production. Anemia develops with decreased red blood cell production. Metabolic acidosis, not alkalosis, occurs from the kidney's inability to excrete the hydrogen ion. Growth failure, not obesity, occurs because of the alteration in calcium metabolism and the kidney's inability to synthesize vitamin D so that it can be used.

1164. A 12-year-old child's medication regimen for treating type 1 diabetes mellitus is changed from administering NPH and rapid-acting insulin to a basal-bolus insulin regimen. To achieve tight glucose control and for therapy to be effective, the nurse should instruct that the child and/or parent to: SELECT ALL THAT APPLY. 1. administer a once daily dose of a long-acting insulin such as glargine (Lantus®). 2. administer rapid-acting insulin such as aspart (NovoLog®) with each meal and snack based on the carbohydrate grams consumed. 3. administer extra rapid-acting insulin when the amount of the child's daily exercise increases. 4. consistently count the amount of carbohydrates the child consumes throughout the day. 5. monitor the child's blood glucose four to eight times a day. 6. monitor the child's blood glucose at midnight and 3 a.m. once a week.

: 1, 2, 4, 5, 6 With basal-bolus insulin therapy, basal insulin is administered once a day using glargine, and then a bolus of rapid-acting insulin is administered with each meal and snack based on the number of carbohydrates eaten and the child's blood glucose level. Depending on the amounts of snacks eaten, blood glucose could be monitored up to eight times a day, and the child may get six to seven injections a day. Because of the potential for hypoglycemia at night, the child's blood glucose should be monitored at midnight and 3 a.m. once a week. Exercise increases the need for carbohydrates and not insulin.

1138. A school nurse is educating school-aged children on modifiable risk factors for coronary artery disease (CAD). Which modifiable risk factors should the nurse include in the presentation? SELECT ALL THAT APPLY. 1. Diabetes mellitus 2. Hypertension 3. Age 4. Family history 5. Sedentary lifestyle 6. Obesity

: 1, 2, 5, 6 Diabetes mellitus, hypertension, sedentary lifestyle, and obesity are modifiable risk factors for CAD. While age and family are risk factors, they are nonmodifiable.

1194. Which assessment findings should the nurse expect for a child diagnosed with diabetes insipidus? SELECT ALL THAT APPLY. 1. Polydipsia 2. Polyphagia 3. Polyuria 4. Glycosuria 5. Ketonuria

: 1, 3 Children who are diagnosed with diabetes insipidus produce large amounts of very dilute urine and are constantly thirsty. There is an absence or insufficiency of antidiuretic hormone, so the body cannot concentrate urine. There is no associated increase in appetite, and the amount of glucose and ketones in the urine is not elevated. Children diagnosed with diabetes mellitus will exhibit polydipsia, polyphagia, polyuria, glycosuria, and ketonuria because the body is metabolizing fats and excreting glucose due to the lack of insulin.

1265. A child is receiving radiation to the left thorax to treat metastases. A nurse adds a nursing diagnosis of Risk for impaired skin integrity to the child's plan of care. Which interventions should the nurse include for this nursing diagnosis? SELECT ALL THAT APPLY. 1. Avoid using soap on the irradiated area 2. Apply lotion to the target skin area after bathing 3. Use water to carefully wash the area, leaving markings on the skin 4. Wear a lead apron when in direct contact with the child 5. Apply a Tegaderm®-type dressing after irradiation to the target area

: 1, 3 Radiation can cause burns to the skin. Soaps, lotions, and powders should not be applied to the irradiated area to prevent further skin irritation and possible skin breakdown. Talcum powder can potentially alter the radiation dosage. Only water should be used to gently cleanse the area. The markings outline the irradiation target area for precise positioning of the radiation and should remain until treatment is completed. Lotions and rubbing to apply the lotion can cause skin breakdown if applied to traumatized skin. A lead apron is unnecessary because radiation is not present in the client's body or in the room. The irradiated area should be left open to air. A Tegaderm®-type dressing is adherent and can cause skin tearing when removed.

1419. A 5-year-old client is receiving dextrose 5% in water and half-normal saline (D5 0.45 NaCl) solution at 100 mL/hr. Which findings during a nursing assessment suggest excessive parenteral fluid intake? SELECT ALL THAT APPLY. 1. Dyspnea 2. Gastric distention 3. Crackles 4. Lethargy 5. Temperature of 102°F (38.9°C) 6. Dry mucous membranes

: 1, 3, 4 Dyspnea, crackles, and other signs of respiratory distress indicate fluid volume overload and occurs from fluid rapidly shifting between the intracellular and extracellular compartments. Lethargy and change in level of consciousness can occur from fluid shifting in brain cells. Gastric distention can occur from excessive oral fluid intake or infection. An elevated temperature and dry mucous membranes are signs of fluid volume deficit, not excess.

1292. A child is diagnosed with strabismus and a school health nurse is meeting with the child's teacher. Which suggestions should the school nurse make to the teacher? SELECT ALL THAT APPLY. 1. Ensure that the child has preferential seating. 2. Seat the impaired child with other impaired children. 3. Allow extra time for assignments and test taking. 4. Order additional materials and books in large print. 5. Speak louder than usual when the child is present in the classroom.

: 1, 3, 4 Strabismus is unequally aligned eyes (cross-eyes) due to unbalanced muscle control. By law, this is a disability that allows for school accommodations. Preferential seating will enable the child to see assignments, class presentations, and the teacher. Allowing for extra time to complete assignments ensures that the child has time to adjust and focus his or her vision to read the necessary documents. Federal and state laws provide educational plans to ensure that individuals with disabilities have the materials and equipment needed to attain an education like any other student in the system. Seating or grouping impaired children violates federal Individual with Disabilities Education Act (IDEA) guidelines. Speaking louder is unnecessary because this is a visual, not a hearing, problem.

1144. A hospitalized preterm infant diagnosed with tetralogy of Fallot is experiencing a hypercyanotic spell. Which actions should be taken by the nurse? SELECT ALL THAT APPLY. 1. Place the infant in a knee-chest position. 2. Administer 2 L of oxygen via nasal cannula. 3. Administer intramuscular morphine sulphate. 4. Use a calm and comforting approach. 5. Administer oral propranolol (Inderal®). 6. Prepare for emergency surgery.

: 1, 3, 4, 5 During a hypercyanotic episode, the infant becomes hypoxic and should be placed in a knee-chest position. This traps blood in the lower extremities and keeps the heart from being overwhelmed. Morphine sulfate decreases preload and afterload. A calm approach is also useful in settling the infant and decreasing the infant's oxygen demand. Propranolol may be given to aid pulmonary artery dilation. The infant should receive 100% oxygen via face mask in this condition. Emergency surgery is delayed for preterm infants. However, if the episodes increase, a temporary or palliative surgical repair can be done to create a shunt between the aorta and the pulmonary artery to enable blood to leave the aorta, enter the pulmonary artery, oxygenate the lungs, and return to the left side of the heart, the aorta, and the body.

1355. Which of the following should a nurse expect in the plan of care for a client newly diagnosed with ambiguous genitalia? SELECT ALL THAT APPLY. 1. A physical examination by a pediatric urologist 2. Immediate surgical intervention 3. Chromosome analysis 4. Biochemical tests 5. Ultrasonography 6. Radiographic contrast studies

: 1, 3, 4, 5, 6 Uncertain gender is a potential lifetime social tragedy for the child and family so the identification of appropriate gender must be done with precision and accuracy. Therefore, careful history and physical examination, chromosome analysis, biochemical tests, and ultrasonography and radiographic contrast studies are important components of the assessment to determine gender assignment. The assignment of a gender followed by immediate surgical intervention was the traditional approach to treatment for ambiguous genitalia. A multidisciplinary team approach is now used to assign gender, and surgical reconstruction may be delayed to avoid irreversible surgical interventions.

1267. A 15-year-old adolescent is scheduled to have total body irradiation in preparation for a bone marrow transplant. A nurse has completed teaching about care following irradiation. Which statements by the adolescent indicate correct understanding of the information? SELECT ALL THAT APPLY. 1. "I should report if I have any bleeding, such as after brushing my teeth, because my platelets will be low." 2. "I will work hard to improve my health by eating plenty of raw fruits and vegetables." 3. "To relieve the dry mouth, I can suck on lozenges or popsicles or drink cold liquids." 4. "I will need to take an antiemetic around the clock to help prevent nausea and vomiting." 5. "Once the irradiation is completed, I will no longer need to be in protective isolation." 6. "My friends know that I can't have live plants or flowers so they wanted to know if silk flowers are okay."

: 1, 3, 4, 6 Irradiation results in bone marrow suppression and pancytopenia. Bleeding precautions are necessary. Xerostomia (dry mouth) is a side effect that can be combated by lozenges, liquids, or oral hygiene. Antiemetics are given around the clock to control nausea. Plants, flowers, and goldfish can harbor mold spores and are contraindicated. Visitors should check before bringing items from home. Raw fruits and vegetables should be avoided because these increase the risk of infection. Foods must be fully cooked. Protective isolation continues until after the bone marrow transplant.

1095. A student explains to an instructor that the infant period is categorized as the "oral phase" according to Freud's theory. Which statements by the student suggest an understanding of this phase? SELECT ALL THAT APPLY. 1. An infant sucks for nourishment as well as pleasure 2. An infant does not find pleasure in sucking but does find enjoyment from the nourishment 3. An infant may have more pleasure in breastfeeding than bottle feeding because it expends more energy 4. An infant does not find pleasure in use of a pacifier 5. An infant explores the world through the mouth 6. An infant begins to explore the genital area to learn sexual identity

: 1, 3, 5 An infant has the desire to suck, which may actually build the ego and self-esteem of an infant. There is more pleasure in breastfeeding because it expends more energy and also provides other comforting mechanisms, such as warmth. An infant explores the world through the mouth, especially the tongue. Freud believed the oral phase is important for both nutrition and pleasure in the first year of life. An infant finds pleasure in sucking on a pacifier, according to Freud. A preschooler learns sexual identity through awareness of the genital area.

1286. Which instructions should a nurse include when teaching a parent with a child diagnosed with contact dermatitis from poison ivy? SELECT ALL THAT APPLY. 1. Apply dressings moistened with either saline or water. 2. Apply a paste of baking soda and water. 3. Apply calamine or Caladryl® lotion. 4. Remove scabs to promote healing. 5. Inspect the yard for plants with three pointed leaflets that are different shapes.

: 1, 3, 5 Applying moistened dressings and calamine or Caladryl® lotion should relieve itching. The allergen should be identified and eliminated. The yard should be inspected and poison ivy destroyed. Poison ivy has three pointed leaflets that are different shapes and shiny. The color varies by season. A baking soda bath can relieve itching, but applying a paste of baking soda can be harmful. Scabs should fall off naturally, not removed, so that healing tissues are protected.

1290. A child with a tentative diagnosis of otitis externa is being evaluated in the clinic. A nurse should expect to prepare the child for: SELECT ALL THAT APPLY. 1. an otoscopic examination. 2. a complete blood count (CBC) with differential. 3. a culture of the external auditory canal. 4. x-rays of the face and skull. 5. ear curettage for removal of debris present in the ear canal.

: 1, 3, 5 External otitis is inflammation of the external ear canal. An otoscopic examination allows visualization of the tympanic membrane to ensure that there is no extension of the external otitis into the middle ear. If there is any drainage in the ear, cultures are taken to identify the organism. Ear curettage for removal of debris present in the ear canal may be necessary to visualize the tympanic membrane. A CBC is usually not done because diagnosis is based on the symptoms and physical examination. X-rays are not beneficial.

1190. Which outcomes should a nurse plan for a child diagnosed with adrenal insufficiency? SELECT ALL THAT APPLY. 1. Child demonstrates a positive body image. 2. Child demonstrates no complications related to inactivity. 3. Child responds to oxygen regimes to avoid hospitalization. 4. Child and family verbalizes causes of the disease and treatment regimen. 5. Child responds to activity restrictions to conserve energy.

: 1, 4 Adrenal insufficiency is associated with stress, fluid volume regulation, and hyperpigmentation of the skin which can affect body image. Verbalizing causes and treatments is a statement that can be measured. There are not usually problems related to oxygenation or activity.

1180. A child is admitted in thyrotoxic crisis. Which manifestations should a nurse expect to observe during assessment? SELECT ALL THAT APPLY. 1. Delirium 2. Hypothermia 3. Bradycardia 4. Nausea 5. Vomiting

: 1, 4, 5 Thyrotoxic crisis, or thyroid storm, is a medical emergency resulting from highly elevated circulating levels of thyroid hormone. Additional manifestations of thyrotoxic crisis are hyperpyrexia, tachycardia, hypertension, mental status changes, and multisystem organ failure.

1216. A nurse is admitting a 5-week-old infant through an outpatient surgical unit for a laparoscopic correction of pyloric stenosis. Which manifestations should the nurse expect when asking the parents about the infant's symptoms? SELECT ALL THAT APPLY. 1. Projective vomiting 2. Bile-colored emesis 3. Sweet smelling vomitus 4. Weight loss 5. Absence of tears when crying 6. Hungry immediately after vomiting

: 1, 4, 6 Because the sphincter is stenosed and the muscle hypertrophied, preventing emptying of the stomach into the duodenum, the infant would begin to vomit immediately after each feeding. Weight loss occurs due to lack of nourishment. With an empty stomach after vomiting, the infant feels hungry. The emesis should not be bile-colored because the feeding does not reach the duodenum to be mixed with bile. The vomitus is sour smelling and not sweet smelling because it has reached the stomach, where it is in contact with stomach enzymes. Absence of tears could indicate dehydration in an older infant, but infants younger than age 6 weeks do not tear.

1261. Leukemic cells have invaded a 16-year-old male's testes and irradiation of the testes is planned. The client asks a nurse if this means he will be sterile. The nurse's best response to the client is based on knowing that: SELECT ALL THAT APPLY. 1. the irradiation to the testes will lead to sterilization. 2. the irradiation of the testes will decrease sperm production but not cause sterilization. 3. this is a question that only the oncologist and radiologist would be able to answer. 4. a lead shield will be used to protect the pelvic area and preserve reproductive organs. 5. if the male is past puberty and is forming sperm, sperm banking may be an option before treatment.

: 1, 5 Because irradiation of the testes leads to sterilization, sperm banking might be suggested before chemotherapy and radiation to preserve sperm for reproduction later in life. No sperm will be produced after irradiation of the testes. A nurse would be expected to answer the client's question and not defer the question. Deferring could cause distrust between the client and nurse or lead the client to suspect that there is more that is not being told to him. The testes are reproductive organs of the male.

1247. A child diagnosed with leukemia is to receive a unit of platelets. The child's weight is 33 lbs. The ordered rate for the platelets is 10 mL/kg/hr. A nurse should plan to transfuse the platelets at a rate of _______ mL/hr.

: 150 First convert pounds to kilograms using a proportion formula (2.2 lbs = 1 kg). 2.2 lbs : 1 kg :: 33 lbs : X kg Multiply the extremes and then the means and solve for X. 2.2 X = 33 X = 15 kg. The child weighs 15 kilograms. Next determine the rate: Milliliter per hour at 10 mL/kg/hr. 10 mL : 1 kg :: X mL : 15 kg X = 150 mL/hr. The commonly accepted rate for transfusions in a child is 10 mL/kg/hr.

1099. A clinic nurse is meeting with a mother and her 3-year-old son. The toddler is acting out, and the mother asks the nurse what a good form of discipline would be for her son. The nurse recommends a "time-out" for the child. Which statement regarding a time-out is most accurate? 1. The child should sit still for as many minutes as he misbehaved 2. The child should sit still at a time-out for as many minutes as his age in years 3. The child should be able to read a book during time-out 4. Children should not be expected to sit still until they are in school

: 2 A 3-year-old should be expected to have a time-out of 3 minutes. Three minutes may not seem long to an adult, but to a toddler 3 minutes is an excellent form of discipline because it seems like a long time to be restricted from activity. The child should not have his time-out related to his minutes of misbehaving because the time-out may be too long an expectation. A child can act out for a long time. The time-out is a form of discipline, and reading a book should not be related to misbehaving. Reading a book should be a reward. Children can be expected to sit still for short periods of time at the toddler level.

1100. A clinic nurse is caring for a 2-year-old client. During the examination the child's parents ask the nurse when their toddler should be toilet trained. Which response by the nurse is most appropriate? 1. "Children should be placed on the potty chair often so they get used to the task and should be rewarded immediately for staying on the potty chair." 2. "Children need sphincter control, cognitive understanding of the task, and the ability to delay immediate gratification." 3. "Children should be ready to toilet train at about 2 years old." 4. "First put training pants on your child so the child gets used to not wearing a diaper."

: 2 A child will be ready for toilet training based on his or her own readiness. Toilet training should not be pushed too soon or the child will become frustrated. A child's development is cephalocaudal, and therefore the child's body may not be able to control the rectal and urethral sphincters. A 2-year-old may not want to sit on a toilet when they do not feel it is necessary. There is no set age for potty training. A child may not be physiologically ready for controlling his or her bladder.

1404. Which specific expected outcomes should a nurse establish for a child diagnosed with cystic fibrosis (CF)? 1. Adequate urine output, absence of injury, and normal growth and development 2. Absence of pulmonary infection, adequate nutrition, and skin remains intact 3. Adequate hydration, absence of diarrhea, and eats at least 75% of meals 4. Absence of dehydration, maintains cleanliness, and adheres to medication regimen

: 2 A child with CF has a generalized dysfunction of the exocrine glands such that mucus secretions, particularly in the pancreas and the lungs, are so tenacious that they have difficulty flowing through gland ducts. Thickened secretions in the bronchioles are a media for microorganism growth. Interventions are necessary to achieve the outcome of absence of pulmonary infection. Enzymes for digesting fat, protein, and some sugars are unavailable because the ducts in the pancreas for secreting the enzymes are plugged by the enzyme secretions. Interventions are necessary to achieve the outcome of adequate nutrition. Because of the high fat content, stools are bulky, foul-smelling, and irritating and can cause skin breakdown. Interventions are necessary to achieve the outcome of absence of skin breakdown. Although options 1 and 3 include outcomes, adequate urine output and absence of diarrhea are not specific to the child with CF. Absence of dehydration, maintains cleanliness, and adheres to medication regimen (option 4) are pertinent but are not the most specific outcomes to be achieved.

1191. A 10-year-old child is admitted for testing to diagnose Cushing's syndrome. For which initial test should a nurse prepare the child and parents? 1. Glucose tolerance test (GTT) 2. Urine or saliva cortisol level 3. Dexamethasone suppression test 4. Serum 17-hydroxyprogesterone level

: 2 A cortisol level, which is increased in Cushing's syndrome, is used in the initial screening for the syndrome. Hyperglycemia is associated with, but not diagnostic of, Cushing's syndrome; a GTT is used to diagnose diabetes mellitus. If the results for cortisol levels are borderline, a dexamethasone suppression test may be done. The serum 17-hydroxyprogesterone level is used to evaluate for adrenal hyperplasia.

1308. A child is being evaluated for possible increased intracranial pressure following head trauma. Which assessment finding associated with increased intracranial pressure (ICP) should a nurse report to a health-care provider? 1. Increasing alertness 2. Widened pulse pressure 3. Tachycardia 4. Decreased systolic blood pressure (SBP)

: 2 A widened pulse pressure (increased SBP and a decreased diastolic blood pressure) is one of the signs of Cushing's triad and is indicative of ICP. Decreasing level of consciousness (not increasing alertness), bradycardia (not tachycardia) and an increased SBP (not decreased SBP) are other signs and symptoms of Cushing's triad.

1358. A nurse is preparing to administer an analgesic for short-term, mild pain in a pediatric client with a history of acute renal insufficiency. Which pharmacological agent should the nurse select from the list of standing orders from the health-care provider? 1. Ibuprofen (Motrin®) 2. Acetaminophen (Tylenol®) 3. Morphine sulfate 4. Meperidine (Demerol®)

: 2 Acetaminophen is an appropriate analgesic for short-term, mild to moderate pain in this client as it will not contribute to renal insufficiency. Ibuprofen could contribute to renal impairment. Morphine sulfate and meperidine are narcotic analgesics and are best used for moderate to severe pain.

1299. A nurse is planning care for a 4-year-old child who is to have eye surgery. Which intervention should the nurse most definitely include in the plan to prepare the child for surgery? 1. Discuss the impending surgery with parents who should then discuss it with their child. 2. Provide a doll with an eye patch in place and allow time for the child to play with it. 3. Introduce the child to other children on the unit who have had surgery. 4. Show the child a 30-minute movie featuring a child being prepared for surgery.

: 2 Allowing the child to play with a doll with an eye patch provides the opportunity to play out fears and concerns and allows clarification of misconceptions. Neutral words should be used to describe the procedure and explanations should be provided by both the nurse and parents. The child should also have the opportunity for rehearsing and handling items that will be used in providing care. A preschooler is egocentric and needs information on how surgery will affect him or her and not how it affects other children. A 30-minute movie is too long. A child has a limited concept of time, and a teaching session should be no more than 10 to 15 minutes.

1424. A nurse is taking a history for a 4-year-old child diagnosed with Reye's syndrome. A parent states prior to hospitalization that multiple over-the-counter medications were administered to treat the child's influenza. Which medication stated by the parent is most important for the nurse to report to a healthcare provider? 1. Acetaminophen (Tylenol®) 2. Bismuth subsalicylate (Pepto-Bismol®) 3. Pseudoephedrine (Dimetapp®) 4. Diphenhydramine (Benadryl®)

: 2 Although the etiology of Reye's syndrome is unknown, the condition typically occurs after a viral illness, such as influenza, and is associated with aspirin (acetylsalicylic acid) use during the illness. Bismuth subsalicylate contains aspirin. Acetaminophen is an aspirin-free analgesic and antipyretic. Pseudoephedrine is an allergy and/or cold remedy used for nasal drying and decongestion, and diphenhydramine is an antihistamine. These do not contain aspirin.

1326. A school-aged child has an Ilizarov external fixator applied to a lower extremity for bone lengthening. Which action should a nurse include when caring for the child? 1. Loosening the bolts and lengthen the rods on the fixator every other day 2. Cleansing the pin sites with sterile saline twice daily 3. Discouraging the child from bearing any weight on the involved extremity 4. Removing sections of the fixator apparatus when the child is positioned in bed

: 2 An Ilizarov external fixator is an external fixation device that uses wires, rings, and telescoping rods to permit limb lengthening to occur by manual manipulation of the rods. Pins secure the device into the bone. Pin site care is necessary to prevent infection and allow inspection for loosening of the pins. Only the health-care provider should manipulate the rods to lengthen them. Partial weight bearing is allowed. The fixation device must remain intact and should be supported on pillows when the child is in bed.

1262. Prior to administering L-asparaginase to a 12-year-old child with acute lymphocytic leukemia, a nurse reviews the child's laboratory report. Which lab value should prompt the nurse to notify a physician before administering the chemotherapeutic agent? 1. Hemoglobin (Hgb) 11.8 mg/dL 2. Blood glucose 252 mg/dL 3. Total bilirubin 1.2 mg/dL 4. Absolute neutrophil count (ANC) 1,078

: 2 An adverse effect of L-asparaginase is hyperglycemia, which may need to be treated with insulin before administration of another dose. The normal Hgb for ages 6 to 12 years is 11.5 to 15.5 g/dL and total bilirubin is 0.3 to 1.2 mg/dL. An ANC of 1,078 is acceptable for L-asparaginase administration. An ANC of less that 1,000 increases the child's risk of infection.

1418. A child, admitted to an emergency department, is experiencing nausea and vomiting, salivation, respiratory muscle weakness, and depressed reflexes an hour after exposure to pesticides. Which medications should a nurse anticipate administering to the child? 1. Atropine and flumazenil (Romazicon®) 2. Atropine and pralidoxime (Protopam chloride®) 3. Epinephrine and naloxone (Narcan®) 4. Epinephrine and digoxin immune Fab (Digibind®)

: 2 An organophosphate base in pesticides causes acetylcholine to accumulate at neuromuscular junctions. Atropine, an anticholinergic medication, and pralidoxime chloride, a cholinesterase reactivator, are effective antidotes to reverse the symptoms. Flumazenil antagonizes the effects of benzodiazepines on the central nervous system (CNS), such as sedation, impaired recall, and psychomotor impairment. Epinephrine is an alpha- and beta-adrenergic agonist and cardiac stimulant that strengthens myocardial contractions, increases systolic blood pressure and cardiac rate and output, and constricts bronchial arterioles, inhibiting histamine release. Naloxone is a narcotic antagonist that reverses the effects of opiates. Digibind is the antidote for digoxin and digitoxin, which acts by complexing with circulating digoxin or digitoxin, preventing the drug from binding at receptor sites.

1370. To allow a 10-year-old child receiving peritoneal dialysis a sense of control, a nurse should best allow the child to: 1. cleanse the abdomen before inserting the needle for administering the local anesthetic. 2. select liquids to drink during the dialysis procedure and for meals from a list of options. 3. engage in therapeutic play with a cloth doll that has a dialysis catheter. 4. play with a toy that is only allowed during the peritoneal dialysis procedure.

: 2 Because the bulk of peritoneal fluid causes pressure on the stomach and causes a feeling of fullness, a child is provided with a liquid diet or small frequent feedings. Allowing the child to choose the preferred, allowable liquid ensures a sense of control. Other suggestions include allowing the child to record the amount of solution infused and drained and then compare this to the nurse's recordings. The nurse needs to ensure sterility of supplies and prevent infection. Allowing a 10-year-old to cleanse the abdomen is not the best option. Therapeutic play is used to enhance self-esteem and reduce anxiety. Playing with a toy allowed only during dialysis helps to reduce boredom.

1366. A nurse is caring for a 3-year-old child with chronic renal failure. The nurse has established a nursing diagnosis of Risk for interrupted family processes related to chronically ill family member. The nurse evaluates that an outcome has been met for this nursing diagnosis when which observation is made? 1. Parents stay with the child continuously while hospitalized. 2. Parents participate in the care of their ill child. 3. Parents inform the nurses when they are unable to visit. 4. Parents find child care for children at home so they can visit.

: 2 Because the parents should be assisting with the child's care if healthy, then participating in the child's care when ill should promote family processes. Continuously staying with the child doesn't demonstrate normal family processes where parents should be taking time for themselves. Informing the nurses when they are unable to visit is unrelated to the nursing diagnosis of risk for interrupted family processes. Although visiting an ill child is expected, it does interrupt normal family processes for children at home. However, the focus of this nursing diagnosis is the interruption of family processes between the child and the parent. In normal family processes, the parent should be involved with the child and not just visiting

1399. Which nursing assessment findings and therapy should a nurse expect for a child diagnosed with cystic fibrosis (CF)? 1. Pica appetite; increasing nutritional choices 2. Abnormal accumulation of mucus in respiratory and other mucous duct tracts; managing infection 3. Steatorrhea; increasing oral fluids 4. Decreased sodium and chloride secretion; vitamin and mineral supplements

: 2 CF is an autosomal recessive disorder of the exocrine gland. The thick, stagnant, mucous secretions become a hospitable environment for bacteria, leading to infection. Pica appetite is an appetite for nonnutritive substances such as soil, chalk, coal, or paper. No research has linked a pica appetite directly with CF. Even though children with CF do manifest appetites for either sweet or salty foods on occasion, that appetite has not been termed "pica." Although steatorrhea may be present, the therapy does not include increasing oral fluids. With CF there is increased (not decreased) sodium chloride secretion. Vitamin and mineral supplements are important, but pancreatic enzyme supplementation is a much greater concern.

1448. A clinic nurse is reviewing prescriptions for a 5-year-old client who has a new diagnosis of generalized contact dermatitis. Which medication prescription should the nurse question? 1. Diphenhydramine (Benadryl®) 2. Prednisone (Deltasone®) 3. Calamine lotion 4. Hydrocortisone cream (Cortaid®)

: 2 Children who have new diagnoses of contact dermatitis require medication that will relieve the symptoms and discomfort while the identified allergen is being removed from their environment. Prednisone would not be a first-line treatment for contact dermatitis. This medication potentially could be added at a later date if there has not been significant and expected resolution of the contact dermatitis. Diphenhydramine, calamine lotion, and hydrocortisone cream should all be expected medications for the treatment of contact dermatitis.

1149. A nurse is planning the care of a pediatric client with congenital heart disease. For which specific complications related to congenital heart disease should the nurse plan to monitor the client? 1. Congestive heart failure and pulmonary hypotension 2. Congestive heart failure and hypoxemia 3. Hypoxemia and pulmonary hypotension 4. Pulmonary hypotension and cyanosis

: 2 Clients with congenital heart disease are at risk for developing congestive heart failure and hypoxemia. Congenital heart disease lessens the effectiveness of the heart's pumping action (heart failure), causing blood pooling in the heart or in the pulmonary circulation. Hypoxemia results when the blood is inadequately oxygenated. Pulmonary hypertension, not hypotension, is another possible complication.

1373. A nurse is managing care of a pediatric client following renal trauma. The nurse should monitor the client first for: 1. electrolyte imbalance. 2. profuse bleeding. 3. hypertension. 4. hypotension.

: 2 Following renal trauma, the client is at risk for profuse bleeding because the blood supply to the kidney constitutes approximately one-fifth of the total cardiac output. Changes in blood pressure or electrolyte imbalances are important considerations but these are not the first considerations for monitoring.

1143. The parents of a pediatric client report that their child is experiencing palpitations, dizziness, diaphoresis, and chest pain. The client is diagnosed with supraventricular tachycardia (SVT). A nurse instructs the parents on techniques to reverse future episodes of SVT. Which technique stated by a parent indicates further teaching is needed? 1. Wrap the child's head with a cold, wet towel. 2. Massage the child's carotid arteries bilaterally. 3. Have the child perform the Valsalva's maneuver. 4. Administer medications after taking the child's pulse for 1 full minute.

: 2 Further teaching is needed when a parent states to bilaterally massage the carotid arteries. This should not be employed to reverse SVT because it may restrict blood flow. Massaging the carotid arteries unilaterally, however, is a technique used to convert SVT and should be included in the teaching plan. Wrapping the child's head with a cold, wet towel, having the child perform the Valsalva's maneuver (bearing down), and taking the pulse prior to administering medications are techniques that the nurse should teach the parents to use when symptoms of SVT are present because these can potentially convert the SVT rhythm to a sinus rhythm

1142. The parents of a pediatric client report that their child is experiencing palpitations, dizziness, diaphoresis, and chest pain. The client is diagnosed with supraventricular tachycardia (SVT). A nurse instructs the parents on techniques to reverse future episodes of SVT. Which technique stated by a parent indicates further teaching is needed? 1. Wrap the child's head with a cold, wet towel. 2. Massage the child's carotid arteries bilaterally. 3. Have the child perform the Valsalva's maneuver. 4. Administer medications after taking the child's pulse for 1 full minute.

: 2 Further teaching is needed when a parent states to bilaterally massage the carotid arteries. This should not be employed to reverse SVT because it may restrict blood flow. Massaging the carotid arteries unilaterally, however, is a technique used to convert SVT and should be included in the teaching plan. Wrapping the child's head with a cold, wet towel, having the child perform the Valsalva's maneuver (bearing down), and taking the pulse prior to administering medications are techniques that the nurse should teach the parents to use when symptoms of SVT are present because these can potentially convert the SVT rhythm to a sinus rhythm.

1239. A child diagnosed with hemophilia is brought to a clinic due to pain and restricted movement of the left knee after tripping going upstairs. A nurse assesses that the knee is hot, swollen, and tender to touch. The nurse should initially conclude that the child is likely experiencing: 1. a Baker's cyst. 2. hemarthrosis. 3. a patella fracture. 4. disseminated intravascular coagulation (DIC).

: 2 Hemarthrosis is bleeding into the joint. Signs include restricted movement, pain, tenderness, and a hot, swollen joint. A Baker's cyst is a bulge occurring in the popliteal bursa behind the knee from excess synovial fluid. A patella fracture is a broken kneecap. Signs include intense pain and swelling. Based on the child's history of hemophilia, patella fracture would be an unlikely initial conclusion for the nurse, although this could be the eventual diagnosis. DIC is widespread clotting throughout the body. Symptoms include diffuse bleeding from multiple sites.

1253. Following diagnostic testing for an enlarged cervical lymph node, a health-care provider informs a 20-year-old female client of a diagnosis of Hodgkin's disease and explains the disease process and recommended treatment. Which statement, overhead by a nurse when the client telephoned her parents, indicates that the client understands the diagnosis and treatment? 1. "I am so relieved; I was worried that I had cancer and there wasn't anything that could be done to treat it." 2. "I have a good chance of being cured with radiation therapy, chemotherapy, or a combination of both." 3. "I will need to have a laparotomy to stage the disease before I can start irradiation and chemotherapy." 4. "I am so upset; I wanted to go to college, marry, and raise a family. Now, I won't be able to do any of this."

: 2 Hodgkin's lymphoma, which peaks at 20 years of age, is potentially curable with radiation therapy alone or with a combination of several chemotherapeutic agents. The overall 10-year survival rate is as high as 90%. However, 30% of persons who survive pediatric Hodgkin's lymphoma develop a secondary malignancy 30 years after their Hodgkin's lymphoma is diagnosed. Hodgkin's disease is a B-cell malignant disorder (cancer) that affects the reticuloendothelial and lymphatic systems. A laparotomy and splenectomy are no longer performed to determine the staging. However, if lesions found on imaging performed for staging are suspicious and if the findings might alter the treatment regimen, then a biopsy may be performed. There is no indication that the client has an advanced stage of lymphoma.

1235. The parents of an 8-year-old African American child diagnosed with sickle cell anemia are being taught pain control measures for their child. Which measure is most important to teach the parents to prevent the onset of vaso-occlusive pain? 1. Apply ice packs to all joints as soon as the child awakens. 2. Encourage drinking large amounts of fluids daily. 3. Administer acetaminophen (Tylenol®) 650 mg orally daily. 4. Increase outdoor exercise and exposure to the fresh air and sunshine.

: 2 Hydration promotes hemodilution, reduces blood viscosity, and prevents vessel occlusion. Ice packs can cause vasoconstriction and decrease tissue perfusion, thereby increasing pain and sickling. Acetaminophen may be used for pain relief, but the dose is excessive for a child. Exercise increases oxygen demand, decreases oxygen tension, and increases sickling. Sunshine exposure can lead to dehydration, which increases blood viscosity.

1279. A nurse is planning the discharge of a pediatric burn victim to the child's home. The child is able to ambulate with assistance but is cognitively and developmentally unable to function at the ageappropriate milestones due to asphyxiation. Which component should the nurse include as most important in the discharge planning of this child? 1. Identifying support groups for the child's parents 2. Coordinating care and services for the child's rehabilitation 3. Assessing the child's home for safety concerns 4. Communicating with the school to ensure that the child will receive mandated services

: 2 Identifying and making referrals to services for the rehabilitation of the child with neurological sequelae secondary to asphyxiation is imperative because the child will require assessments posthospitalization on a physical, emotional, cognitive, and social level. The parent will need support systems and support groups, but it is not the top priority because the initial focus should be on the child's needs. A home safety assessment is included in rehabilitation services. Communication with the school will need to occur but will likely happen postdischarge.

1209. In an infant who has been tentatively diagnosed with esophageal atresia, what should be the priority nursing outcome? 1. Infant will maintain adequate fluid volume. 2. Infant will demonstrate effective breathing pattern. 3. Nutritional status will be maintained. 4. Parents will exhibit emotional health.

: 2 In esophageal atresia, the infant has a high risk for aspiration if feedings are not withheld and if the infant is not evaluated before the next feeding. The infant also is at high risk for aspirating secretions if there is a fistula connecting the esophagus and the trachea. The other options are important but not the priority. Adequate fluid volume and nutrition are important because the infant is unable to take nourishment orally. The parents are concerned because surgical intervention will be required as there is no opening between the esophagus and the stomach and a fistula may exist between the esophagus and trachea.

1129. A nurse assesses the pain level of a Native American pediatric client recovering from cardiac surgery. Knowing that Native American pediatric clients may not express pain, the nurse reviews the child's pulse and blood pressure readings following analgesic administration. Which finding should indicate to the nurse that the client's pain is not well-controlled? 1. Decreased heart rate and decreased blood pressure 2. Increased heart rate and increased blood pressure 3. Increased heart rate and decreased blood pressure 4. Decreased heart rate and increased blood pressure

: 2 Increased heart rate and blood pressure may be indicative of postoperative pain in a pediatric client. A decreased heart rate and blood pressure could indicate that analgesics are effective for pain control. An increased heart rate and decreased blood pressure could be signs of bleeding. A decreased heart rate and increased blood pressure could be a sign of a neurological complication associated with cardiac surgery.

1123. A 4-year-old child is hospitalized with a high fever. While the child is in bed, the child comforts himself by sucking the thumb. The mother of the child becomes concerned because her child has not sucked his thumb for 6 months. Which nursing response to the mother's concerns is most appropriate? 1. "I don't know why he is sucking his thumb; maybe your child just needs more attention." 2. "This is a form of developmental regression and can be a normal response for a child who is hospitalized. Continue to love and support your child." 3. "Is there anything else going on in your family right now that may be causing your child to feel anxious?" 4. "Where is the child's father? Maybe the child wants his father?"

: 2 It is common for preschoolers to revert to a behavior they have outgrown in an effort to cope with difficult situations. Sucking a thumb is a comfort measure for the child. Thumb-sucking does not indicate that the child is not getting enough attention. Thumb-sucking does not mean there are other contributing factors. The hospitalization is enough stress in a child's life. Having a father present may or may not be helpful to the child.

1276. A nurse is in an emergency department when a parent calls sobbing hysterically and stating, "My baby has just put an electrical cord in her mouth! What do I do?" Which statement or question identifies the first priority of the nurse? 1. "Call 911 and have them bring your baby to the emergency department." 2. "Have you removed the cord from the baby's mouth?" 3. "Is there bleeding at or around the mouth?" 4. "What does your baby's mouth look like?"

: 2 It is imperative to ensure that the cord has been removed from the baby's mouth to prevent further harm. Electrical current is conducted from the plug to the underlying tissue. The actual amount of tissue damage is much larger than where the cord actually touched. Telling the parent to call 911 or asking about the condition of the baby's mouth are correct actions, but not the first priority.

1325. A nurse is completing a thorough assessment of the spine. The nurse is concerned about a curve in a young child and records the exaggerated lumbar curve as: 1. scoliosis. 2. lordosis. 3. kyphosis. 4. kyphoscoliosis.

: 2 Lordosis involves an exaggerated lumbar curve. Scoliosis is a lateral curve; kyphosis is an exaggerated thoracic curve. Kyphoscoliosis involves both curves in the same person.

1405. A community health nurse is planning a follow-up visit to a family after their firstborn child died from sudden infant death syndrome (SIDS). Which action is most important for the nurse to include in the initial visit? 1. Help the family in making a plan for future children. 2. Allow time for the parents to express their anger and grief. 3. Make a referral for genetic counseling and education. 4. Educate the family on the causes of sudden infant death syndrome.

: 2 Many families are unable to express their grief and loss openly. Helping the parents understand SIDS and that they are not to blame for the death of their child is the most important action at the current time. Helping families make plans for future children is essential once the grieving process resolves. There is no definitive etiology for SIDS, and making a referral for genetic counseling and education is not necessary. Educating the family on what are associated incidences of children who have died from SIDS can help the parents plan for and use safety precautions for their future children.

1258. A 5-year-old child, hospitalized following surgical intervention for osteosarcoma, is uninterested in eating. Which nursing action would best support the child's nutrition? 1. Providing only foods that the child likes best 2. Asking the child's parents to visit at mealtime 3. Turning on the television so the child is distracted while eating 4. Offering juice, popsicles, or ice cream every 2 hours

: 2 Meals at home are often a time for family socialization. Mealtime visiting with the parents may offer the emotional, social, and physical support to enhance the child's nutritional intake. While allowing the child to select only best-liked foods may increase the child's intake, a young child is unlikely to select the most nutritious foods. Distractions may inhibit the child's intake. Juices, popsicles, and ice cream lack protein, and offering these this frequently may decrease the child's intake of nutritious foods.

1285. A child has a tentative diagnosis of Albright's disease (neurofibromatosis). A nurse assisting the child to disrobe prior to a physical exam should expect the presence of: 1. pediculosis. 2. café-au-lait spots. 3. tick bites. 4. congenital nevi.

: 2 Neurofibromatosis is a neurocutaneous autosomal dominant disorder that causes tumors to grow along nerves. The presence of six café- au-lait spots or café-au-lait spots larger than 4 6 cm may indicate neurofibromatosis or Albright's disease. Pediculosis (lice infestations) and tick bites are not genetic disorders and have not been clinically shown to lead to neurofibromatosis. Congenital nevi (moles) are present at birth and result from a proliferation of benign melanocytes in the dermis, epidermis, or both and may lead to melanoma. Congenital nevi are different in appearance from café-au-lait spots of neurofibromatosis.

1263. Prednisone is ordered three times a day for a child receiving chemotherapy. Which is the best schedule for a nurse to suggest to a parent? 1. 6 a.m., 2 p.m., and 10 p.m. 2. 8 a.m., 1 p.m., and 6 p.m. 3. 10 a.m., 6 p.m., and 2 a.m. 4. 11 a.m., 4 p.m., and 9 p.m.

: 2 Prednisone should be taken with meals or snacks to decrease or prevent gastrointestinal upset. This schedule is the closest of the options to meal time, yet it spaces the medication for effectiveness. While options 1 and 2 space the medication over a 24-hour time period, it is not given consistently in relation to meals. Option 4 would be administering prednisone before meals and at bedtime.

1374. An emergency department nurse is triaging a group of pediatric clients. Which client should the nurse assess first? 1. A child with an oral temperature of 102.1°F (38.9°C) 2. A child with dyspnea and a palpable abdominal mass 3. A child with a 3-cm facial laceration 4. A child with vomiting and diarrhea for 24 hours

: 2 Priority should be given to the child with respiratory compromise and a palpable mass. The most common presenting sign of Wilms' tumor is a swelling or mass in the abdomen. Priority nursing care of a client with this condition involves ensuring swift diagnosis and surgical intervention. Fever, facial lacerations, vomiting, and diarrhea may be symptoms requiring immediate care. However, the priority client has the abdominal mass.

1349. An adolescent male with a history of spinal cord injury reports a leaking of urine at fairly regular intervals. A nurse should document in the client's plan of care a nursing diagnosis of: 1. functional urinary incontinence. 2. reflex urinary incontinence. 3. stress urinary incontinence. 4. urge urinary incontinence.

: 2 Reflex urinary incontinence is an involuntary loss of urine at fairly regular intervals. It occurs from an autonomic response when the bladder reaches a certain urine volume. Functional, stress, and urge incontinence do not occur at fairly regular intervals. Functional incontinence occurs when a usually continent person is unable to reach the toilet in time to avoid unintentional loss of urine. Stress incontinence occurs when activities that increase abdominal pressure cause a sudden leakage of urine. Urge incontinence occurs when there is involuntary passage of urine soon after feeling a strong sense of needing to void.

1425. A nurse is caring for a 5-year-old child from Italy. The child is crying and the interpreter is stating that the child has extreme pain. The nurse's first priority should be to: 1. have the child's mother who knows limited English ask the child what hurts. 2. assess the level of the child's pain using an appropriate FACES pain rating scale. 3. administer morphine sulfate 1 mg IV as prescribed. 4. call the health-care provider to request a change in pain medication dosage as it is not adequately controlling the child's pain.

: 2 The FACES pain-rating scale has been translated to a variety of languages. The nurse's judgment regarding the choice of pain medication and dose should be based on the reported level of pain. If possible, the nurse should do an independent assessment because sometimes information can be misinterpreted if there is limited knowledge of the language. Assessment should be completed prior to a pain intervention. There is no information indicating the need for the pain medication to be changed.

1329. The parents of a child with Duchenne muscular dystrophy have just learned that children with the disease have a limited life expectancy. They ask what this means for how they will raise their son. Which explanation by the nurse is best? 1. "Because he will be cognitively impaired, there is no reason to deal with the prognosis." 2. "Throughout his disease, we will focus on maximizing his abilities and keeping him comfortable." 3. "There is not enough known about this disease to know what will happen to your son." 4. "Nothing is likely to happen for a long time; we'll deal with it when the time comes."

: 2 The answer acknowledges the prognosis, describes a plan, and gives the message that the family would not be abandoned. Much is known about the disease, so it is possible to maximize opportunities. Information can allow the family to begin planning, including genetic planning.

1199. A child is to have a breath hydrogen test to evaluate for malabsorption syndrome. Which instruction is most important for a nurse to include when teaching the parents about the preparation needed for the test? 1. "Be sure to administer the prescribed antibiotics an hour before the test." 2. "The dinner the night before the test should consist of meat, rice, and water; avoid other starchy foods." 3. "Give the child an enema to cleanse the child's bowel the morning of the test." 4. "Encourage fluids just before the test to moisten the child's mouth for blowing into the mouthpiece."

: 2 The breath hydrogen test is used to detect a rise in expired hydrogen after oral loading with a specific carbohydrate. A meal high in starch can interfere with test results. Antibiotics should not be given because they may reduce hydrogen levels. Bowel cleansing is not required because the colon is not being examined. The child should be NPO 12 hours before the test.

1101. A nurse is preparing a 4-year-old boy for surgery. Which nursing action is appropriate for preoperative teaching based on Erikson's developmental stages? 1. Allowing the child to make a project related to the surgery 2. Having the child put a surgical mask on a doll 3. Asking the child how he feels about surgery 4. Allowing the child to listen to music without further instructions

: 2 The child should practice putting a mask on the doll so he can feel the mask and know how it will fit on the doll. This will help to reduce the child's anxiety. The child is too young to do a project related to surgery. The child is not old enough to verbalize his feelings. Music may be helpful, but it cannot replace appropriate preoperative teaching based on the child's developmental stage.

1431. A home care nurse is observing a 7-year-old client self-administer a dose of albuterol (Ventolin®) via a metered-dose inhaler with a spacer. Within a short time, the client begins to wheeze loudly. The nurse should: 1. consult with the health-care provider (HCP) to have the child's medication dosage increased. 2. report the findings to the HCP because it indicates paradoxical bronchospasms. 3. reassure the parent that this usually only occurs when the medication is initially begun. 4. reassess the child's technique; contact of the inhalation drug with the eyes can cause this reaction.

: 2 The client's wheezing suggests paradoxical bronchospasms, which can occur with excessive use of adrenergic bronchodilators. The medication should be withheld and the HCP notified. A paradoxical bronchospasm can occur from excessive use, so the dosage should not be increased. Reassuring the parent is an inappropriate action. There is no indication that this is an initial dose. Contact with the eyes can cause eye irritation.

1218. A nurse is admitting an infant with a tentative diagnosis of intussusception. Which question to the mother would be most helpful in obtaining additional information to confirm the diagnosis? 1. "Does your baby vomit after each feeding?" 2. "What does the infant do when experiencing pain?" 3. "Is your infant passing ribbonlike stools?" 4. "Have you felt a mass in your infant's abdomen?"

: 2 The diagnosis is suggested by the infant's history. In intussusception, infants suddenly draw up their legs and cry as if in severe pain due to a peristaltic wave that occurs from the invagination of one portion of the intestine into another. When the peristaltic wave passes, the infant is symptom free. Vomiting can occur with the pain from the peristaltic wave; this is unrelated to the time of feeding. Ribbonlike stools occur in Hirschsprung's disease because stool passes through areas of impacted feces and narrowed aganglionic distal segments of the bowel. The stools of intussusception are red, currant jelly-like in appearance from the mix of blood and mucous. A mass may be present and the baby's abdomen distended, but this is common to other gastrointestinal conditions and too general to confirm the diagnosis.

1228. A nurse is caring for a 5-year-old child who has been diagnosed with peritonitis secondary to a ruptured appendix. The child begins complaining of abdominal pain and nausea, even though a nasogastric tube (NG) is in place. When pulling back the covers, the nurse notes that the child's abdomen is distended. Which action should be taken by the nurse first? 1. Call the primary health-care provider (HCP). 2. Check the NG to determine if fluid is moving within the tubing to the drainage container. 3. Finish the abdominal assessment and check the child's vital signs. 4. Obtain and administer an antiemetic.

: 2 The first action is to check the functionality of the NG, including suction and correct placement. Once it has been established that the NG is functioning, the nurse should complete the assessment and contact the primary HCP because nonmechanical paralytic ileus can develop in the immediate postoperative period. An antiemetic should be administered if prescribed, and reestablishment of a nonfunctioning NG does not relieve the nausea, but these are not the first action.

1119. A nurse is caring for a 14-year-old client who was admitted for dehydration from nausea and vomiting. The client is ready for discharge and says to the nurse, "I will tell you something, but you can't tell anyone." Which nursing action is most appropriate? 1. Promise the client that the information will not be told to anyone due to Health Insurance Portability and Accountability Act (HIPAA) laws. 2. Tell the client that the information will be confidential unless it is life threatening or harmful. 3. Tell the client that only the physician will be told; otherwise the information will remain confidential. 4. Ask the client to tell a social worker who then can follow through with the information if it is concerning.

: 2 The information can remain confidential unless there is adult abuse involved or if there are legal implications. The HIPAA law does not state that all information is to remain confidential. The nurse should not tell the physician unless the information is life threatening or could potentially result in harm to the client. The nurse should listen to the client. Though the client is willing to confide in the nurse, the client may not feel as comfortable with the social worker. Both the nurse and the social worker have the same obligation to report the information if it is life threatening or harmful.

1275. A nurse is caring for a 7-year-old client who has been hospitalized for several days with severe burn injuries in the lower extremities. On initial examination, the nurse makes the following assessments regarding the client's right leg: distal pulses are weak, capillary refill is greater than 3 seconds, and the child reports feelings of numbness and tingling in the leg. What should be the nurse's interpretation of this information? 1. This is to be expected during this phase of burn healing. 2. This is an emergency situation and a health-care provider should be notified. 3. Comparative assessment of the extremity in 1 hour is necessary. 4. Fluid has accumulated under the scab of the burn and is decreasing blood flow to the area.

: 2 The information suggests inadequate circulation and impairment of nerve function. This is an emergency situation and an escharotomy needs to be performed. The tough, leathery scab (eschar) that forms over moderate to severe burns may create a tight band that constricts an area, such as the anterior and posterior portion of an extremity or the trunk. These are not expected findings during any phase of burn healing. Waiting an hour will delay treatment. This is not fluid, but an emergency situation because of constriction from the eschar.

1281. A new nurse asks an experienced nurse during a child's dressing change, after the child had a skin graft, why the skin appears lattice-like and not smooth like the unburned areas of the child's body. Which is the experienced nurse's best response? 1. "The skin is an allograft from a cadaver donor, and the freezing of the skin causes this appearance." 2. "The skin is an autograft from a distal, unburned portion of the child's body, but the skin was meshed so it could be stretched to cover more area." 3. "After the grafting procedure, the area is covered by a bulky dressing. The lattice-like appearance is from the indentations of the dressing." 4. "The fluids that seep through the child's tissues cause the new skin to stretch and separate, but as it heals, the skin pulls back together."

: 2 The lattice-like appearance is from a mesh graft. In a meshed autograft, a partial-thickness of skin is taken from another area of the body and is slit at intervals so that it can be stretched to cover a larger area. An allograft can be from a cadaver or a donor and is sterilized and frozen until needed. But unless it is meshed, it will not have a lattice-like appearance. Though bulky dressings applied too tightly can leave indentations, it will not be a uniform lattice-like appearance. Although fluid loss does occur with burn injuries, it does not cause the lattice-like appearance to a graft.

1306. A nurse is working in a clinic and answers a telephone call from a parent who states that a 6-year-old child has a nose bleed that is not stopping. Which direction should the nurse provide to the parent? 1. Tilt the child's head back. 2. Squeeze the child's nares below the nasal bone for 10 to 15 minutes. 3. Take the child to urgent care as soon as possible. 4. Immediately insert a cotton ball or swab in each nare.

: 2 The nares should be squeezed for 10 to 15 minutes just below the nasal bone and the child should be instructed to breathe through the mouth. An ice pack may be placed on the back of the neck or on the nose. The child's head should be tilted forward to prevent blood from going down the throat, which can lead to vomiting. If the bleeding cannot be stopped, there may be a need to access health care through an urgent-care setting. If the bleeding cannot be controlled, one of the treatments is the insertion of a cotton ball or swab soaked with phenylephrine, which causes vasoconstriction.

1155. A nurse is caring for a child who has liver enlargement secondary to infectious endocarditis. For which associated cardiac condition should the nurse assess the client? 1. Dysrhythmia 2. Right-sided heart failure 3. Myocardial infarction (MI) 4. Tetralogy of Fallot

: 2 The nurse should assess for the presence of right-sided heart failure. Back pressure in the portal circulation occurs in right-sided heart failure. Dysrhythmias may occur due to impaired cardiac function. Chest pain, dyspnea, and dysrhythmias would be initial signs of a MI. Tetralogy of Fallot occurs as a result of the malformation of the right ventricular infundibulum, which can lead to heart failure.

1184. Which nursing diagnosis has the highest priority for a child diagnosed with Addison's disease? 1. Potential excess fluid volume 2. Disturbed body image 3. Altered development 4. Altered sleep and rest

: 2 There is a high incidence of body image disturbances related to changes in skin pigmentation associated with Addison's disease. There is a potential for fluid volume deficit, but there is not usually a problem with development or sleep and rest patterns

1231. A nurse is caring for a 1-year-old child who had surgery for a gastrostomy tube insertion. Which statement describes the nurse's best action in the care of the child? 1. Place thick dressings under the gastrostomy tube area to keep it clean and dry. 2. Apply the prescribed antibiotic ointment to the insertion site. 3. Apply tension on the gastrostomy device to ensure the balloon is against the stomach wall. 4. Begin tube feedings as soon as the child returns from surgery.

: 2 Topical antibiotics and skin protective creams assist in preventing infections. While surgical sites carry a risk of infection and moist areas increase risk, a thick dressing puts tension on the tube, which could cause accidental removal or impaired tissue integrity from pressure. Tension should be avoided to prevent tissue trauma or accidental removal. Tube feedings are not begun until bowel function has returned.

1211. A 39-week-old infant is postoperative day 1 after emergency surgery for tracheoesophageal atresia. Which nursing action would be unsafe for the infant? 1. Providing a pacifier during gastrostomy feedings 2. Performing both oral and tracheal suctioning 3. After a gastrostomy feeding, elevating the gastrostomy tube, covering it with sterile gauze, and leaving it unclamped 4. Turning the infant frequently

: 2 Tracheal suctioning should be avoided because it can disrupt the suture line from the repair. To remove secretions, oral suctioning can be performed. A pacifier is provided during gastrostomy feedings for sucking pleasure. Gastrostomy feedings are administered below the level of the surgical repair, and the end of the tube is elevated, covered by sterile gauze, and kept unclamped to allow air introduced during the feeling to bubble from the tube and not enter the esophagus and pass the fresh suture line. Any vomitus will project into the gastrostomy tube and not contaminate the fresh sutures. Turning discourages fluid from accumulating in the lungs.

1260. A 6-year-old child is being seen in a clinic after discharge from a hospital for removal of a brain tumor. Which finding, reported by a parent, best suggests the child has likely developed a complication? 1. Reports occasional headaches 2. Voiding large amount of dilute urine 3. Able to walk with use of crutches 4. Ventricular-peritoneal shunt tubing palpable under the skin

: 2 Voiding large amounts of dilute urine is a sign of diabetes insipidus caused by a deficiency in antidiuretic hormone (ADH) secreted by the posterior pituitary gland. When ADH is inadequate, the renal tubules do not reabsorb water, leading to polyuria. Headaches may be a symptom associated with a complication but also may occur due to the neurosurgical procedure. Physical mobility may be impaired because of the location of the tumor. Being able to walk with crutches is an expected outcome if mobility was affected. The ventricular-peritoneal shunt tubing is inserted into the ventricle, and a tract is made that travels behind the ear, along the neck and chest wall, and into the peritoneal cavity. The shunt may be felt under the skin behind the ear and along the neck.

1137. A nurse documented a nursing outcome of oxygen saturation (SaO2) greater than 95% for a pediatric client diagnosed with heart failure. When the nurse obtains a SaO2 value of 90%, the nurse determines that the outcome was not achieved and intervenes by administering oxygen. The nurse's intervention is based on the nurse's knowledge that the 90% SaO2 value indicates a PaO2 value of: 1. 40 mm Hg. 2. 60 mm Hg. 3. 80 mm Hg. 4. 90 mm Hg.

: 2 When the SaO2 value drops to 90%, the PaO2 value is 60 mm Hg. A PaO2 value of 40 mm Hg is too low. When the SaO2 is 95%, the PaO2 is in the normal range of 80 to 100 mm Hg.

1256. An experienced nurse and a new nurse are providing preoperative care for a 5-year-old child diagnosed with Wilms' tumor. The experienced nurse should intervene when observing the new nurse: 1. inform the child that water is not allowed because the procedure will be performed soon. 2. palpate the child's abdomen during assessment. 3. provide the child with a doll for play that has removable kidneys. 4. state to the child, "You'll get some medicine that you breathe or get through your arm to make you sleep."

: 2 Wilms' tumor (nephroblastoma) is an intrarenal abdominal tumor. Palpating the abdomen can potentially spread the cancerous cells. A 5-year-old child would typically be NPO for 2 hours prior to the procedure, although this time period can vary. Play therapy is an appropriate means for teaching the child about the surgical procedure and assisting the child to cope. When explaining the surgery and anesthesia, words that the child understands should be used.

1217. Based on assessment findings, a nurse thinks an infant may have developed necrotizing entercolitis (NEC) and plans interventions. In what order should a nurse plan to intervene for the infant? Place the items in order of priority. ______ Notify the health-care provider (HCP). ______ Immediately stop feedings. ______ Start prescribed antibiotics. ______ Prepare the infant for an abdominal x-ray. ______ Start prescribed intravenous fluids.

: 2, 1, 4, 5, 3 It is critical to immediately stop feedings if NEC is suspected. The earlier the feedings are stopped, the better chance the infant has of preserving the bowel and not needing surgery. Next, notify the HCP, who will prescribe fluid replacement for the blood loss from bleeding from the bowel and prophylactic antibiotics. An x-ray will be prescribed; if perforation has occurred, there will be air in the abdominal cavity. If a portion of the bowel is necrosed and perforated, the infant needs immediate surgical intervention. The abdomen should also be handled gently to prevent bowel perforation.

1293. An 8-year-old child is hospitalized with a femur fracture. A parent tells a nurse that the child's glasses for treating nearsightedness were damaged during the accident that resulted in the hospitalization. Which observations should the nurse expect due to the child's nearsightedness? SELECT ALL THAT APPLY. 1. Reads with a book held at arms length away from the eyes 2. Bends with head close to the table when coloring 3. Squints in an attempt to see objects clearly 4. One eye turns inward 5. Lens of the eye appears opaque

: 2, 3 Nearsightedness is an ability to see objects clearly when at close range but not at a distance, so objects are brought closer to the eyes. Because objects farther away are not as visible, squinting may occur in an attempt to see objects more clearly. Holding a book farther away is observed in farsightedness because objects are better seen when farther away. In esotropia strabismus, an eye turns inward. An opaque lens is seen with cataracts.

1317. A 7-year-old child may have hydrocephalus secondary to a malignancy. Which assessment findings should a nurse anticipate? SELECT ALL THAT APPLY. 1. Increased head circumference 2. Headache 3. Personality change 4. Vomiting 5. Angioedema

: 2, 3, 4 Headache, personality change, and vomiting can each result from a space-occupying lesion that is applying pressure on brain structures. Because a 7-year-old child's sutures should be closed, there should be no increase in head circumference occurring with hydrocephalus. Angioedema is swelling and fluid accumulation under the skin.

1315. A nurse is caring for multiple hospitalized children. In which conditions might the nurse assess for the presence of papilledema? SELECT ALL THAT APPLY. 1. Eczema 2. Craniosynostosis 3. Shaken baby syndrome 4. Hydrocephalus 5. Chest trauma

: 2, 3, 4 Papilledema is edema and inflammation of the optic nerve caused by increased intracranial pressure (ICP). It can be assessed with the use of an ophthalmoscope. Any condition that increases ICP can cause papilledema. Eczema and chest trauma do not cause increased intracranial pressure.

1445. A nurse is caring for a 14-month-old child, hospitalized with a diagnosis of gastroenteritis and severe dehydration. Which prescriptions should a nurse determine are most appropriate for treating the child's diagnosed condition? SELECT ALL THAT APPLY. 1. An antidiarrheal medication to control the toddler's diarrhea 2. Intravenous (IV) fluid therapy for rehydration 3. IV antibiotic therapy to treat infection 4. Antipyretic therapy for fever 5. Oral fluid therapy for fluid rehydration 6. Analgesics for pain and discomfort

: 2, 3, 4 The child who presents with severe dehydration at this age needs IV therapy to stabilize the balance of fluids and electrolytes. Frequently, a symptom of gastroenteritis is fever. Ensuring that the fever is controlled will provide some comfort to the child. There are many causes of gastroenteritis that do require a course of antibiotics to ensure complete recovery. An antidiarrheal medication is contraindicated in a toddler this age. Oral fluid rehydration is appropriate, but not when the toddler presents with severe dehydration. The pain and discomfort the toddler will have is due to fever and cramping from the gastrointestinal illness, thus analgesics will not usually be prescribed.

1351. Which client would be most appropriate for a nurse to instruct on the use of intermittent self-urinary catheterization? 1. A 15-year-old female preparing to have a cesarian section 2. A 18-year-old female newly diagnosed with multiple sclerosis (MS) 3. A 13-year-old male with spinal cord injury and no awareness of urge to void 4. A 16-year-old female who is 8 months pregnant and reports dribbling

: 3 A 13-year-old male with spinal cord injury and no awareness of urge to void would require a means to empty his bladder. The 15-year-old would need only temporary indwelling catheterization. A person newly diagnosed with MS would likely not have bladder involvement requiring intermittent catheterization. This may occur later in the disease process. Dribbling is common in the third trimester of pregnancy. Catheterization would be avoided.

1152. A pediatric nurse evaluates that a nursing assistant knows emergency procedures when the nursing assistant activated the emergency response system for a 6-year-old child admitted with a diagnosis of heart failure. The sign observed by the nurse indicating that lifesaving measures were necessary likely was: 1. gagging. 2. coughing. 3. inability to speak. 4. heart rate of 125 bpm.

: 3 A 6-year-old child should be able to speak. Inability suggests that the airway is compromised. Lifesaving measures are needed to clear the airway of an obstruction. Gagging indicates a patent airway, and coughing indicates that air is being exchanged. In both situations the child should be observed for possible vomiting or signs that the airway is compromised. The nurse should be notified, but the emergency response system does not need to be activated. The heart rate is a little high for a 6-year-old. The nurse should be notified, but the emergency response system does not need to be activated.

1094. A nurse in a clinic is assessing the weight of an infant. Which infant's weight indicates to the nurse that the infant's weight is normal for the infant's age? 1. The baby's weight has tripled in the first 6 months of life 2. The baby's weight has doubled in the first year of life 3. The baby's weight has doubled in the first 6 months of life and tripled in the first year 4. The baby's weight has doubled in the first 6 months and doubled again in the next 6 months

: 3 A baby's weight should double in the first 4 to 6 months of life and triple by the end of the first year. The weight needs to more than double the first year.

1114. A nurse is assessing the nutritional needs of a 1-year-old client. According to recommendations for introducing milk products, which type of milk should a 1-year-old child be drinking? 1. 2% milk beginning at the age of 1 2. 1% milk 3. Whole milk until the age of 2 years 4. Skim milk

: 3 A child should not have fat intake limited until the age of 2 years. Fats are necessary to ensure myelination of nerve fibers. Milk that is 2%, 1%, or skim has reduced amounts of fat.

1282. A 2-year-old child has a bulky dressing in place over 60% of the child's body following a skin-grafting procedure for a severe burn injury. A parent arrives to visit the child and is shocked to see the child's appearance. Which is the most caring action for a nurse? 1. Help the parent don the mask, gown, and gloves that are required to enter the child's room. 2. Bring the parent to a quiet place to allow the parent to talk about immediate concerns. 3. After the parent is appropriately attired, take the parent into the room and show the parent that it is okay to stroke the child's face and hold the child's hand. 4. Arrange for a member of the clergy to come visit with the parent for support.

: 3 A parent may not ask spontaneously to touch the child or hold the child because the parent is likely in a state of grief, so he or she does not react in a normal manner. Both the parent and the child are supported by the nurse's gesture of caring. While assisting the parent to dress in appropriate attire, taking the parent to a quiet room, or arranging for the clergy may be supportive to the parent, the most caring response is offering support to both the parent and child.

1368. A pediatric client with chronic kidney disease has elevations in serum blood urea nitrogen (BUN) and creatinine. A nurse interprets this to mean that the child has a reduction in: 1. serum erythropoietin. 2. growth hormone. 3. glomerular filtration rate. 4. blood flow to the kidneys.

: 3 A reduction in glomerular filtration rate results in an accumulation of nitrogenous wastes. This is reflected in the serum BUN and creatinine. Serum erythropoietin, growth hormone, or the residual urine do not reflect elevations in BUN and creatinine. Serum erythropoietin is a measurement of the degree of hormonal stimulation to the bone marrow to stimulate the release of red blood cells (RBCs). Growth hormone is an anabolic hormone that promotes protein application and mobilizes glucose and free fatty acids. Reduced blood flow to the kidneys is but one cause of renal failure.

1410. In which position should a nurse place a child postoperative tonsil and adenoidectomy (T&A)? 1. Semi-Fowler with the head turned to the side 2. High Fowler, head slightly forward and to the side 3. Side-lying 4. Supine

: 3 A side-lying position is best to facilitate drainage from the oropharynx. A semi-Fowler's position with the head turned to either side would hinder drainage and increase the risk for aspiration. A supine position would allow secretions to pool at the back of the throat or in the buccal cavity.

1423. A 4-year-old child with cystic fibrosis (CF) is prescribed vitamin A supplements. Which finding by a clinic nurse indicates that the vitamin has been effective? 1. Viscosity of secretions is decreased. 2. Pancreatic enzyme absorption is increased. 3. Skin is supple and healthy. 4. Number of bleeding episodes is reduced.

: 3 A water-miscible form of vitamin A is given in children diagnosed with CF because the uptake of the fat-soluble vitamins is decreased. One of the functions of vitamin A is to keep epithelial tissue healthy by aiding the differentiation of specialty cells. Other treatments for CF, such as bronchodilators and recombinant human doxyribonuclease (Pulmozyme®), decrease the viscosity of secretions. Vitamin A has no effect on pancreatic enzyme absorption. Vitamin K, another fat-soluble vitamin administered in CF, increases coagulation.

1433. A nurse practitioner prescribes amoxicillin (Amoxil®) for an 8-month-old client diagnosed with acute otitis media. A nurse correctly explains the primary purpose of the medication to a parent when stating it is prescribed to: 1. shrink swollen tissue in the eustachian tube. 2. reduce the severe pain. 3. treat the probable organism, Haemophilus influenzae. 4. reduce the fever.

: 3 Acute otitis media is frequently caused by the Haemophilus influenzae and Streptococcus pneumoniae bacteria. The primary purpose of amoxicillin is to treat the infection caused by these two organisms. Reducing inflammation of the eustachian tube will occur, but this is not the primary purpose for treatment with amoxicillin. Treating the ear infection will reduce the pain but is not the primary purpose for treatment with amoxicillin. As the infection is treated, the fever will be reduced, but this is not the primary reason for treatment with amoxicillin.

1294.A school nurse has completed a second visual screen for preschool-aged and school-aged children. Which child should the nurse plan to complete a referral for follow-up evaluation of the child's vision? 1. A 4-year-old child who has 20/40 vision in both eyes 2. A 6-year-old child who has 20/30 vision in both eyes 3. A 7-year-old child who has 20/40 vision in both eyes 4. A 9-year-old child who has 20/15 vision in both eyes

: 3 After a second screening, a child 6 years of age or older who has 20/40 vision or worse in one or both eyes should be referred to a physician for corrective eye care. A preschooler with 20/40 vision and a school-aged child with 20/30 vision should not be referred because these are normal findings for this age group. A result of 20/15 on visual acuity tests indicates vision that is better than normal.

1321. A nurse is developing teaching materials for new mothers. The nurse should include information about which common practice that can increase the risk for developmental dysplasia of the hip (DDH)? 1. Carrying a child in a backpack 2. Carrying a child in a frontpack 3. Swaddling 4. Extended time in a car seat

: 3 Although swaddling has many benefits (e.g., neurological and musculoskeletal development, less physiological distress, better motor organization, and more self-regulatory ability), it can cause DDH because the legs are in extension and adduction. Abduction, as in front, or backpacks, or in car seats does not increase this risk.

1257. While an experienced nurse is orienting a new nurse to a pediatric oncology unit, the new nurse asks why there seems to be so many adolescents with osteosarcoma and not other age groups. The experienced nurse explains that osteosarcoma has a peak incidence during adolescence because of the: 1. increase in hormonal production. 2. epiphyseal growth plates have closed. 3. rapid growth spurt experienced during adolescence. 4. increase in sports-related injuries that occurs during this time.

: 3 Although younger children can develop osteosarcoma, the peak incidence is during the rapid growth years: at 13 years for girls and 14 years for boys. Adolescent growth spurts may be influenced by hormones, but the hormones are not directly related to the development of bone tumors. Osteosarcoma can occur before or after epiphyseal plate closure. Bone tissue of osteosarcoma never matures into compact bone. The diagnosis of osteosarcoma may be made when the client seeks medical attention for a sports-related injury.

1342. When preparing to complete a health history for a 9-year-old child diagnosed with mental retardation with an IQ level of 45, which level of participation should a nurse expect? 1. Able to communicate verbally only with twoletter words 2. Able to read and comprehend simple written instruction with large letters 3. Able to walk independently to perform a simple skill 4. Able to perform tasks that requires careful manual dexterity

: 3 An IQ of 45 is considered to be moderate mental retardation. The range of IQ for moderate mental retardation is 36 to 49. At the level of IQ, a 9-year-old child can perform simple manual skills and should be able to walk independently. The child can use simple communication with more than the use of two-letter words. At an IQ level of 45, the child does not progress in functional reading or arithmetic, so would be unable to read and comprehend written instructions even with large lettering. While simple manual skills are able to be performed, skills requiring manual dexterity are not.

1416. A nurse working on a pediatric unit has medications to give at 1000 hours. Which assessments should lead the nurse to conclude that the prescribed medication should be withheld and the health-care provider (HCP) contacted? 1. Oral hydrocodone with acetaminophen (Vicodin®) to a 10-year-old child with burn injuries who is complaining of being dizzy and light headed. 2. Oral acetaminophen (Tylenol®) to a 6-month-old infant with a fever of 102°F (38.9°C) due to an infection secondary to a motor vehicle accident who has developed a rash. 3. Intravenous (IV) clindamycin (Cleocin®) to a 16-year-old male with aspiration pneumonia secondary to a near-drowning experience whose blood pressure (BP) is 92/56 mm Hg. 4. IV phenobarbital to a 5-year-old child with intermittent seizures secondary to a closed head injury who is complaining of being tired and whom the parent reports is acting drowsy.

: 3 An adverse effect of clindamycin is hypotension. A BP of 92/56 mm Hg is low for a 16-year-old. Normal BP for a 16-year-old male is 111/63 mm Hg to 136/90 mm Hg, depending on height percentile. The nurse should compare the previous BP readings with the current one to determine the degree of BP variation before notifying the HCP. In options 1, 2, and 4, the signs and symptoms are consistent with medication side effects. However, none of these would be potentially life threatening at this time. Monitoring all the children for increasing and/or changing side effects from the medications is necessary.

1348. A nurse is taking a history of a 9-month-old infant with altered urinary elimination. Which abnormal finding should the nurse report first? 1. Odorless urine 2. Dark amber urine 3. Not producing a wet diaper in a 24-hour period 4. Urinary output between 250 and 500 mL per day

: 3 Anuria, or a lack of urine production, may be a sign of altered renal function. Infants often have odorless urine, and output is usually between 250 and 500 mL daily. Dark amber urine may require more inquiry, although it is not the priority item to report.

1289. A 4-year-old child is brought to an emergency department after being hit in the side of the head. Ear trauma is suspected, but the child is turning away from a nurse and burrowing against the parent, crying, and not allowing anyone near. Which action is best for the nurse to take to enable assessing the child's ear? 1. Administer an analgesic first and then proceed after it has taken effect. 2. Ask the parent to hold and restrain the child while the child's ear is inspected. 3. Ask the child to place the nurse's hand by the area that was hurt. 4. Lay the child on a bed and mummy wrap the child; have the parent hold the child's head for examination.

: 3 Asking the child to place the nurse's hand will assist in gaining the child's cooperation. Crying can be from fear rather than pain. A pain assessment should be completed before administering an analgesic. Restraining the child (whether in the parent's arms or in a mummy wrap) increases fear, anxiety, and agitation and can cause injury.

1402. A nurse is preparing a 4-year-old child with cystic fibrosis (CF) for discharge to home. The nurse determines that a parent needs further education when the parent states: 1. "Playing on the backyard swings and hanging upside down are exercises our child will enjoy." 2. "If children at the day care center have a cough, fever or flu symptoms, we should keep our child home." 3. "No pancreatic enzyme supplements will be necessary if our child has a good appetite and regular bowel movements." 4. "Three to four times every day we will perform chest therapy and postural drainage, even if our child doesn't seem congested."

: 3 Because children with CF suffer from absorption and nutrition problems due to lack of enzyme production from clogged and atrophied cells producing the enzymes, pancreatic enzymes must be taken before all meals and snacks. Backyard play, physical exercise, and "hanging upside down" should facilitate gravity drainage and can be considered with proper safety measures enforced. Children with CF should be kept away from areas of known communicable disease outbreaks because acquiring a respiratory disease can cause inflammation, tissue damage, and respiratory failure for a child with CF. Performing chest therapy and postural drainage three to four times a day will help to keep hospitalization for the child with CF at a minimum by loosening and clearing secretions.

1168. The parents of a 7-year-old child diagnosed with type 1 diabetes mellitus are planning to drive 1,200 miles for a vacation at the beach. They question the nurse about insulin storage for the trip. Which response by the nurse is most accurate? 1. "Because insulin must be refrigerated, you will need to obtain the medication from a pharmacy at your destination." 2. "Freeze the insulin before you leave home and take it in a cooler; it should be thawed by the time you get to the beach." 3. "Keep the insulin in a cooler with an ice pack and out of direct heat and sunlight for the trip. Store unopened insulin in the refrigerator at your destination." 4. "Because it is illegal to transport needles and syringes across most state lines, you will need to obtain a prescription from your doctor and purchase the insulin and the syringes at your destination."

: 3 Because insulin should be kept out of direct sunlight and extreme heat, it should be transported in a cooler with an ice pack. Insulin is destroyed when frozen. Although it is not illegal to transport needles and syringes, the child should have a prescription to identify the medication and justify the syringes. The prescription also provides a means to obtain additional supplies if needed.

1153. A nurse is suctioning a pediatric client who has just had cardiac surgery. The nurse observes tachypnea, the use of accessory muscles to breathe, and restlessness. Which action should be taken by the nurse? 1. Continue suctioning, as these are expected during the procedure. 2. Continue suctioning, but monitor closely as these could be signs of distress. 3. Discontinue suctioning, carefully monitor the client, and notify the physician immediately. 4. Discontinue suctioning and notify the physician to insert a chest tube.

: 3 Because tachypnea, using accessory muscles to breathe, and restlessness are signs of hypoxia, suctioning should be discontinued, the client carefully monitored, and the physician notified immediately. Continuing with the suctioning further removes oxygen and can worsen the child's status. The child's status should be monitored, but there is no indication that a lung has collapsed; inserting a chest tube should not be indicated.

1223. A nurse is planning care for children diagnosed with inflammatory bowel diseases. After collecting and analyzing the information about the clients, which statement should best reflect the nurse's conclusion about the information? 1. All clients diagnosed with Crohn's disease are adolescent females. 2. None of the clients with a diagnosis of ulcerative colitis have a family history of the condition. 3. Most of the clients with either a diagnosis of Crohn's disease or ulcerative colitis are adolescent males. 4. Those clients diagnosed with Crohn's disease have more severe and bloody diarrhea than those diagnosed with ulcerative colitis.

: 3 Both Crohn's disease and ulcerative colitis are more common in males, perhaps due to a familial or autoimmune tendency. Because Crohn's disease is more common in males, it is unlikely that all clients will be females. Both Crohn's disease and ulcerative colitis show familial tendencies. Because both diseases involve the development of ulcers of the mucosa or submucosal layers of the colon and rectum, both diseases manifest with diarrhea from irritation and unabsorbed fluids. The ileum is affected in Crohn's disease, whereas the colon and rectum are affected in ulcerative colitis. Because of the more severe ulcerations along the colon in ulcerative colitis, stools are more severe and bloody as compared with Crohn's disease.

1166. A 9-year-old child with a history of type 1 diabetes mellitus for the past 6 years is admitted with a diagnosis of diabetic ketoacidosis (DKA). In preparing for the child's arrival to the nursing unit, the nurse should prepare to: 1. add sodium bicarbonate to the current IV fluids. 2. add potassium chloride to the current IV fluids. 3. use either 0.9% or 0.45% saline for the base IV fluid. 4. administer insulin by subcutaneous injection.

: 3 Both water and sodium are depleted in DKA, thus the child will require intravenous saline. Research has shown no benefit to giving sodium bicarbonate to children with DKA to reverse metabolic acidosis. Potassium is added only after laboratory studies have confirmed that the plasma potassium is low. Usually there is not a drop in plasma potassium. Insulin is always given IV in DKA for rapid effect and close monitoring. Insulin onset by subcutaneous route is 15 to 30 minutes and can cause hypoglycemia from potential excessive dosing when attempting to reduce the hyperglycemia.

1388. A nurse is formulating a plan of care for a 22- month-old child with a diagnosis of laryngotracheobronchitis (LTB). Which symptoms should indicate to the nurse that the child is experiencing impending respiratory failure? 1. Restlessness and irritability 2. Retractions of the accessory chest muscles 3. Decreased inspiratory breath sounds 4. Hoarseness

: 3 Decreased inspiratory breath sounds are a sign of physical exhaustion and may indicate impending respiratory failure. Restlessness and irritability are signs and symptoms of LTB, and children may often prefer sitting upright or in a parent's lap. Retractions may also be noted in most children with LTB as they attempt to compensate for the varying degrees of airway obstruction. Hoarseness is usually associated with other respiratory disorders such as asthma or tonsillitis.

1380. A triage nurse determines that a child brought to an emergency department is experiencing severe respiratory distress when observing: 1. diaphoresis, restlessness, tachypnea, and anorexia. 2. pallor, coughing, wheezing, and confusion. 3. retractions, grunting, cyanosis, and bradycardia. 4. agitation, decreased level of consciousness, diarrhea, and tachypnea.

: 3 Emergency intervention is needed if a child exhibits retraction, indicating use of accessory muscles to breathe. Grunting is an involuntary response to end-stage respiratory effort. Cyanosis indicates that a state of hypoxia exists due to lack of circulating oxygen and progression to bradycardia is an ominous sign that the body is so overtaxed that it is wearing out. The diaphoresis, restlessness, and tachypnea indicate the possibility of an electrolyte imbalance and may indicate mild respiratory distress. Anorexia and other effects of impaired nutritional status exist in conditions of respiratory distress, but the dietary issue does not cause immediate concern. Options 2 and 4 may indicate moderate respiratory distress.

1382. A 4-year-old is hospitalized after experiencing a sore throat and difficulty swallowing for a week. Laboratory tests reveal elevated white blood cells (WBCs), bands, and neutrophils. A throat culture completed a week ago showed Haemophilus influenzae type B. Based on the information, which tentative medical diagnosis should a nurse expect to be documented in the client's chart? 1. Tonsillitis 2. Bronchiolitis 3. Epiglottitis 4. Tuberculosis

: 3 Epiglottitis is an acute inflammation of the epiglottis and commonly results from Haemophilus influenzae type B. Diagnosis of tonsillitis is made on the basis of visual inspection and clinical manifestations. Inflammation of the tonsils often occurs with pharyngitis and may be viral or a Group A streptococcus bacterial infection. Viral infections require only symptomatic treatment. Diagnosis of bronchiolitis includes enzyme-linked immunosorbent assay or direct fluorescent assay that is performed on nasal wash specimens. Tuberculosis is diagnosed by intradermal purified protein derivative (PPD) and chest x-ray.

1435. A 17-year-old female is undergoing a drug screening test for employment. The client tells a nurse collecting the urine specimen of a recent complicated urinary tract infection that was treated with antibiotic therapy. Which antibiotic, if identified by the client, could produce a false positive urine screening test for opioids? 1. Amoxicillin (Amoxil®) 2. Cephalexin (Keflex®) 3. Ciprofloxacin (Cipro®) 4. Ceftazidime (Fortaz®)

: 3 Fluoroquinolones, such as ciprofloxacin, can cause false-positive urine opiate screens. Amoxicillin (an aminopenicillin) and cephalexin and ceftazidime (cephalosporin) do not interfere with urine testing for opioids.

1412. A nurse is caring for a 3-year-old client who is postoperative tonsil and adenoidectomy (T&A) surgery. The nurse should suspect complications when assessing: 1. complaints of sore throat and difficulty swallowing. 2. secretions and dried blood at the corners of the mouth. 3. frequent swallowing and clearing of the throat. 4. the presence of "dark coffee ground" emesis.

: 3 Frequent swallowing and clearing of the throat are signs and symptoms of hemorrhage. Sore throat, difficulty swallowing, dried blood around the mouth, or "coffee ground" emesis are expected findings following a T&A.

1274. A nurse is assessing a 16-year-old adolescent in an emergency department who has been admitted because of burns over 25% of the client's body. Upon initial examination, the nurse makes several observations. Which observation should be most concerning to the nurse? 1. Areas on upper extremities are mottled. 2. Areas on upper extremities are moist and red. 3. Areas on lower extremities are waxy white. 4. Red blistering on anterior lower extremities.

: 3 Full-thickness or third-degree burns are most concerning, can present as waxy white or black, and are identified by a leathery appearance. These burns involve the entire epidermis and dermis and extend into the subcutaneous tissue. Hair follicles, nerve endings, and sweat glands are all destroyed. Full-thickness burns do not heal and will require some type of grafting. Mottled, moist-red, or blistering in appearance are consistent with partial-thickness burns, which will heal spontaneously within about 14 days, but scarring will occur.

1332. A 13-year-old is brought to the emergency department following a motor vehicle crash in which the child's head hit the dashboard. The child is diagnosed with a mild head injury. When assessing the child, which score on the Glasgow coma scale should the nurse expect? 1. 5 2. 10 3. 15 4. 20

: 3 Glasgow coma scale scores range from 0 (dead) to 15 (intact). A score of 15 indicates that brain function is intact. A child with a mild head injury should have intact neurological function. The other options are either too low or beyond the score ranges.

1177. A health-care provider prescribes glucagon 0.5 mg subcutaneously for a client with type 1 diabetes mellitus. A nurse determines that glucagon is used to treat: 1. hypoglycemia resulting from too little food intake. 2. hyperglycemia resulting from too much food intake. 3. hypoglycemia resulting from too much insulin intake. 4. hyperglycemia resulting from too little insulin intake.

: 3 Glucagon is a hormone produced by the pancreas that helps release stored glucose from the liver. Since glucagon acts by promoting glycogen breakdown, it will not treat hypoglycemia related to starvation because there is no glycogen. Glucagon does not treat hyperglycemia.

1264. A 5-year-old girl with alopecia secondary to chemotherapy refuses to wear a wig. The child's mother consults a nurse because she thinks her daughter should wear a wig. She states feeling uncomfortable when people stare at her daughter. Which response is most appropriate? 1. "Have you tried having your child wear a colorful hat instead?" 2. "Does your child feel uncomfortable when others are looking at her?" 3. "You seem concerned about people looking at your daughter. Tell me more about what you are feeling." 4. "Your daughter only needs to wear a head covering when she is exposed to sunlight, wind, or the cold."

: 3 Hair loss is often a greater problem for the parents than the child. The parent may be grieving the loss of a normal child. Option 3 acknowledges the parent's feelings and focuses the communication on the parent by using a therapeutic communication technique of a broad opening statement. Option 1 is giving advice. The mother already stated she would like her daughter to wear a wig. Option 2 focuses on the daughter's feelings rather than the mother's feelings. While option 4 is correct, this statement is ignoring the mother's concern of discomfort.

1429. A health-care provider's (HCPs) progress notes state a plan to initiate an oral NSAID for a child's pain. Based on this information, a nurse should consult with the HCP when noting that which medication was prescribed? 1. Acetaminophen (Tylenol®) 2. Tolmetin (Tolectin®) 3. Hydromorphone (Dilaudid®) 4. Naproxen (Naprosyn®)

: 3 Hydromorphone is an opioid analgesic, not an NSAID. Acetaminophen, tolmetin, and naproxen are all NSAIDs.

1147. A pediatric nurse is providing discharge instructions to the parents of an infant with a history of hypoxemia. The nurse teaches the parents about the signs and symptoms associated with hypoxemia. Which signs or symptoms should prompt the parents to notify the practitioner immediately? 1. Weight loss or gain 2. Excessive crying 3. Dehydration and respiratory infection 4. Not achieving developmental milestones

: 3 Hypoxemia is decreased oxygen concentration of arterial blood. Dehydration can increase the risk of stroke in hypoxemic children. Respiratory infection may compromise pulmonary function and increase an infant's hypoxemia. Weight changes, excessive crying, or concerns over developmental milestones should be reported to the practitioner but often are not immediate concerns.

1229. Before administering an enteral feeding to a 2-month-old infant, a nurse aspirates 5 mL of gastric contents. Which action should the nurse take next? 1. Return the aspirate and withhold the feeding. 2. Discard the aspirate and give the full feeding. 3. Return the aspirate before beginning the feeding. 4. Discard the aspirate and add an equal amount of normal saline to the feeding.

: 3 If the amount aspirated is small (a few milliliters), the aspirate should be returned at the beginning of the feeding to prevent the loss of electrolytes and gastric enzymes. If the amount is large, as compared to the amount prescribed for the feeding, then the amount of the feeding should be reduced by the amount of the aspirate. Five milliliters is equivalent to a teaspoon, which is a small amount for a 2-month-old infant. With a small amount the feeding should not be withheld. Discarding the aspirate with each feeding can result in acid-base and electrolyte imbalances, and saline is not an electrolyte replacement for gastric contents.

1174. The mother of a 12-year-old child diagnosed with type 1 diabetes mellitus asks a nurse what changes in the daily routine should be made during attendance at summer camp. The child will be at camp for 4 weeks. Which is the best response by the nurse? 1. "The child will have an increased need for insulin due to the high carbohydrate content of camp food." 2. "The child's food intake should be decreased by 10% while the insulin should be increased by 10%." 3. "The child's food intake should be increased as activity increases; monitor blood glucose levels three to four times a day to evaluate results." 4. "The child's insulin injection should be given before every meal and snack to ensure that the food being consumed at camp can be utilized by the body."

: 3 Increases in muscle activity promote a more efficient utilization of glucose. School-aged children are more physically active in the summer months than during the school year. An increase of insulin with an increase in physical activity and/or a decrease in food consumption will result in profound hypoglycemia. Giving regular insulin before every meal and snack is an option for older individuals who are usually engaged in regular, high-intensity physical activity; it would not be recommended for a 12-year-old child attending 4 weeks of camp.

1385. A 3-month-old child is hospitalized with acute laryngotracheobronchitis (LTB). In formulating a nursing care plan, which nursing diagnosis should be a nurse's priority? 1. Anxiety 2. Risk for deficient fluid volume 3. Ineffective breathing pattern 4. Deficient knowledge

: 3 Ineffective breathing pattern is the most important prioritization based on physiological needs. Fear and anxiety, deficient fluid volume, and deficient knowledge are all applicable but not the priority.

1113. A nurse at a clinic is preparing the immunizations for a 6-month-old baby. The mother says, "My baby is afraid of strangers and afraid of separating from me. My mother-in-law is upset and thinks I am causing it." Which response by the nurse is most appropriate? 1. "Give your baby to strangers while you are present, so your baby gets used to strangers." 2. "Your mother-in-law is correct; you need to include her more in your baby's needs." 3. "Separation anxiety is an important component of a parent-child attachment." 4. "Just let your baby cry for a while; your baby will get used to being separated from you."

: 3 It is a normal developmental stage when a baby exhibits separation anxiety. The mother can be reassured. Mothers are encouraged to have familiar people visit frequently; infants can then safely experience strangers. It is important to be patient through this stage of development. It is not appropriate to force the baby to be with strangers. The baby doesn't need to cry in order to get used to strangers. Eventually the baby will pass through this stage.

1447. A clinic nurse is educating a parent in administration of eye drops to a 3-year-old who has purulent drainage out of both eyes, swollen eye lids, and inflamed conjunctiva and has been diagnosed with bacterial conjunctivitis. The most important fact that the nurse needs to teach the parent is to: 1. have the child sitting when administering the medication. 2. have the child be in Trendelenburg's position when administering the medication. 3. restrain the child prior to administering the medication. 4. tell the child in detail exactly what will take place so that the child will be cooperative.

: 3 It is necessary to secure the child prior to administering the medication to ensure that the child receives the entire prescribed dose. A 3-year-old child is able to understand what is occurring in relation to the limited amount of experiences the child has had. The child is likely to resist instillation of the eye drops because a child is told not to put anything in the eyes and is likely to remember painful experiences such as dust or a foreign object that has gotten into the eye. The child should be supine when ophthalmic medication administration occurs. Telling the child what is happening is extremely important, but at the age of 3 a detailed explanation will not make the child more cooperative.

1224. A parent is describing the stool number, consistency, appearance, and size for a child diagnosed with celiac disease to a nurse. Which changes in the child's stools should prompt the nurse to conclude that the child's ability to absorb nutrients is improving? 1. Disappearance of currant jelly-like stools 2. Reduction of ribbonlike stools 3. Absence of large, bulky, greasy stools 4. Absence of liquid green stools

: 3 Large, bulky, greasy stools describe steatorrhea. When gluten is ingested in celiac disease, changes occur in the intestinal mucosa or villi that prevent the absorption of foods across the intestinal villi into the bloodstream and the inability to absorb fat. Currant jelly-like stools are characteristic of intussusception from the mixing of blood and mucous. Ribbonlike stools occur in Hirschsprung's disease because stool passes through areas of impacted feces and narrowed aganglionic distal segments of the bowel. Liquid green stools appear in diarrhea.

1156. A school nurse assesses a child who was stung by an insect and is beginning to exhibit signs of distress. The nurse is aware of the child's severe allergy to bee stings and immediately contacts emergency medical services (EMS). Which assessment finding reported to the EMS personnel should be questioned? 1. Signs of airway obstruction 2. Bronchospasm 3. Hypertension 4. Weak thready pulse

: 3 Life-threatening reactions may begin within 5 to 10 minutes after a bee sting. The report of hypertension should be questioned because hypotension, and not hypertension, occurs with anaphylactic shock. Signs of airway obstruction secondary to laryngeal edema, bronchospasm, and cardiovascular collapse are signs of anaphylaxis due to immunological mechanisms and the toxin's effect in stimulating mast cells.

1440. A nurse completes teaching insulin administration to the parent of a toddler newly diagnosed with type 1 diabetes mellitus. The nurse concludes that the teaching was successful when the parent states: 1. "NPH insulin (Humulin N®) is only given at night immediately before the bedtime snack." 2. "It is okay to use only the buttocks for the insulin injections until the child is older." 3. "Insulin lispro (Humalog®) acts within 15 minutes and peaks 30 to 90 minutes after injection." 4. "Insulin detemir (Levenir®) can be added to the insulin lispro (Humalog®) pen to reduce the number of injections."

: 3 Lispro is rapid-acting insulin that peaks in 30 to 90 minutes and may last as long as 5 hours in the blood. NPH insulin can be given in the morning, but there is better glucose control if given at night. NPH peaks in 4 to 14 hours, so there is no need to make sure food is available immediately after administration. Insulin injections should always be rotated to prevent subcutaneous tissue damage from giving the injections in the same location. Detemir is long-acting and lispro is rapid-acting insulin. An insulin pen uses pre-filled, multiple-use insulin cartridges; adding other types of insulins should not be attempted.

1336. An infant has been diagnosed with osteogenesis imperfecta (OI). The nurse is teaching the parents about how to care for their infant. Which information is most important for the nurse to include in the instructions to the parents? 1. Check the color of your infant's nailbeds and mucous membranes for signs of circulatory impairment. 2. If you note signs of infection, bring your infant to the clinic because the infant has a significant immune dysfunction. 3. Protect your infant from injury and handle your baby carefully because your infant's bones can break very easily. 4. Notify the health-care provider if your infant does not respond to sound because the infant's central nervous system (CNS) fails to develop completely.

: 3 OI is also known as brittle bone disease, and the infant should be handled carefully and protected from injury. OI is not a disease of circulation, the immune system, or the CNS.

1171. A 10-year-old child with a 6-year history of type 1 diabetes mellitus has been seen in a clinic for enuresis over the past 2 weeks. Which conclusion by the nurse regarding the likely cause of the enuresis is correct? 1. Sustained blood sugar levels lower than normal 2. Acquired adrenocortical hyperfunction 3. Sustained blood sugar levels higher than normal 4. Acquired syndrome of inappropriate antidiuretic hormone (SIADH)

: 3 Research indicates that children with type 1 diabetes mellitus who have higher hemoglobin A1c levels, higher fasting blood sugar levels, and who experience polydipsia and polyuria are at risk for enuresis. Sustained hypoglycemia will not cause enuresis and neither should adrenal hyperfunction. SIADH will cause enuresis; however, a child with type 1 diabetes will most likely develop enuresis due to sustained, elevated blood glucose levels.

1165. A nurse understands that to modify the risk for early cardiovascular disease in children diagnosed with type 1 diabetes mellitus a child should: 1. exercise at least 30 minutes every day. 2. eat a diet that is low in fat and high in protein. 3. maintain optimal management of blood sugar levels. 4. have a cardiac workup at each visit for the diabetes.

: 3 Research indicates that hyperglycemia is the primary mediator for atherosclerosis in children with type 1 diabetes. Since the risk is only modified, the child needs to avoid hyperglycemia. Exercise and a healthy diet are a part of any growing child's needs. There is no need for a cardiac workup with each diabetes visit.

1207. A 2-week-old infant born with cleft lip and palate is being discharged from a hospital. The infant's parents have each demonstrated the proper technique of feeding the infant with a special softsided bottle equipped with a cleft palate nipple. Which complication should a nurse inform the parents to monitor for with this type of feeding? 1. Overstimulation 2. Overfeeding 3. Aspiration 4. Hiccups

: 3 Risk of aspiration is always present when feeding an infant, but in cases where positioning and special feeding equipment are used the risk is greater. In this case, the soft-sided bottle allows the fluid to be introduced into the infant's mouth while squeezing the bottle; this further increases the risk for aspiration. Coughing and sputtering are prime indicators that the fluid has been aspirated. Feeding is usually a time of increasing infant contentment and not a significant source of stimulation. The amount of the feeding has been determined as age appropriate. As hiccups may occur before, during, or after any feeding, and while the act of hiccupping may lead to aspiration, it is the aspiration itself that is the major complication associated with this type of feeding.

1185. An adolescent is admitted with a diagnosis of suspected Addison's disease. Which assessment manifestations should the nurse expect to find if Addison's disease is the correct diagnosis? 1. Long history of fatigue, weight loss, and muscle tetany 2. Sudden onset of skin hypopigmentation, polydipsia, and hyperactivity 3. Gradual onset of salt craving, decreased pubic and axillary hair, and irritability 4. Sudden onset of increasing weight gain, hirsurtism, and skin hyperpigmentation

: 3 Signs and symptoms of Addison's disease do not usually appear until about 90% of the adrenal tissue is nonfunctional, so the onset is very gradual. There is weight loss, fatigue, hyperpigmentation, muscle weakness, decreased pubic hair, and irritability, as well as a craving for salt. It usually occurs from lesions or neoplasms or the cause may be idiopathic. Previously healthy children exhibit the first signs during periods of stress. Options 2 and 4 are incorrect because they describe symptoms that are sudden in onset. Muscle tetany does not occur with Addison's disease

1196. An older adolescent is diagnosed with acromegaly. Which medication should the nurse expect to be prescribed for this individual? 1. Somatropin (Genotropin®) 2. Desmopressin (Desmotabs®) 3. Somatostatin (Sandostatin®) 4. Clozapine (Clozaril®)

: 3 Somatostatin suppresses growth hormone release. Somatropin is human growth hormone, desmopressin is used to treat diabetes insipidus, and clozapine is an antipsychotic medication used in the treatment of schizophrenia.

1269. A nurse is assessing a child who is presenting with burn injuries. Which injuries would least likely trigger the need for further assessment or evaluation for the potential of child abuse and mandatory reporting? 1. Rope burn with edema 2. Cigarette burns 3. Splash burns on the front torso, face, and neck 4. Scald burns of the feet and legs

: 3 Splash burns on the front torso, face, and neck are consistent with a child pulling down a container of hot liquid. Burns that are a result of child abuse are injuries not consistent with burns seen in a child, such as rope, cigarette, and scalding burns. These types of burns should alert the nurse to further assess the situation. Physicians and nurses are mandated reporters, but the nurse need not conduct the investigation.

1338. A teen is brought to an emergency department with a likely spinal cord injury. To minimize the damage from the spinal cord injury, which classification of medications should a nurse expect a health-care provider to prescribe? 1. An antibiotic 2. An analgesic 3. A steroid medication 4. An antihypertensive medication

: 3 Steroids, administered shortly after injury, help to decrease inflammation and prevent further injury. An antibiotic may be unnecessary; there is no indication of impaired skin integrity or an associated risk for infection that could further damage the spinal cord. Pain control is important, but not the most important medication for preventing swelling and compression of the spinal cord. While autonomic dysreflexia is a complication of a spinal cord injury, and the hypertensive crisis is treated with an antihypertensive medication, this will not protect the spinal cord from further injury.

1324. A nurse is asked to provide education for a 15-year-old who requires surgical treatment for scoliosis. Which should be an appropriate explanation for the adolescent? "The goal of surgery is to: 1. allow you to be taller." 2. prevent pain." 3. prevent problems with breathing." 4. allow clothes to fit you better."

: 3 Surgery for scoliosis is to prevent future problems with breathing and to improve any current problems with breathing. While some height may be gained and pain decreased, these are not guaranteed, nor are they primary indications for the surgery. The same is true for wardrobe.

1422. A new nurse is initiating total parenteral nutrition (TPN) for four hospitalized pediatric clients. An experienced nurse should intervene when the new nurse attaches the TPN infusion and tubing to: 1. a catheter inserted in the right external jugular vein of a 2-year-old child. 2. a catheter inserted in the right subclavian vein of a 4-year-old child. 3. a peripherally inserted intravenous catheter in a hand vein of a 12-year-old child. 4. a peripherally inserted central catheter in the right upper arm of a 6-year-old child.

: 3 TPN is a concentrated hypertonic solution containing glucose, vitamins, electrolytes, trace minerals, and protein. Because it is hypertonic, it should be administered through a central intravenous access site or a peripherally inserted central venous catheter (PICC). A major vein is used to avoid inflammatory reactions and venous thrombosis from the high-caloric and high-osmotic fluid. The external jugular vein, subclavian vein, and a PICC are central intravenous access sites.

1107. A nurse is planning to use the Denver Articulation Screening Examination (DASE) for a 4-year-old child. To properly use the DASE, the nurse should plan to ask the child to: 1. read a favorite book at the child's developmental level. 2. read a phrase and tell the nurse the meaning of the phrase. 3. repeat familiar words that are read to the child. 4. ask the child to state the letters of the alphabet.

: 3 The DASE is a 5-minute test to detect articulation disorders in children age 21/2 to 6 years of age and involves having the child repeat words that are stated by the examiner. The other options are not components of the DASE.

1126. A nurse is preparing to consult with an adolescent being seen in a clinic. Which principle is most important for the nurse to consider when interacting with the client? 1. Avoid a straightforward approach because adolescents cannot fully process their health needs. 2. Reassure the teenager that it is unnecessary to answer all questions; however, before the examination is complete the client will need to provide all information. 3. Avoid conveying surprise over comments made by the client. 4. Because adolescents want to be treated as adults, the same cognitive information should be provided as if they were an adult.

: 3 The adolescent needs the trust of the registered nurse (RN). If the RN appears surprised over comments or information provided, the client may not feel safe or valued and may not be forthright in giving information. It is important to provide honest, straightforward information to the client. There may be questions the client does not want to answer, and that is acceptable. Adolescents are not adults, and they should not be addressed in the same manner as adults.

1127. A nurse is preparing to consult with an adolescent being seen in a clinic. Which principle is most important for the nurse to consider when interacting with the client? 1. Avoid a straightforward approach because adolescents cannot fully process their health needs. 2. Reassure the teenager that it is unnecessary to answer all questions; however, before the examination is complete the client will need to provide all information. 3. Avoid conveying surprise over comments made by the client. 4. Because adolescents want to be treated as adults, the same cognitive information should be provided as if they were an adult.

: 3 The adolescent needs the trust of the registered nurse (RN). If the RN appears surprised over comments or information provided, the client may not feel safe or valued and may not be forthright in giving information. It is important to provide honest, straightforward information to the client. There may be questions the client does not want to answer, and that is acceptable. Adolescents are not adults, and they should not be addressed in the same manner as adults.

1200. A nurse is caring for a 2-month-old infant who has been admitted to a pediatric unit for hypovolemia secondary to gastroenteritis. The baby is irritable and will not calm when the parents hold the infant. The nurse's assessment findings include pulse, 180; respiratory rate, 48; blood pressure, 80/50 mm Hg; mucous membranes, dry; and decreased tears. The nurse interprets the information and determines that the child is moderately dehydrated. Which assessment finding further supports this conclusion? 1. Capillary refill greater than 2 seconds 2. Intense thirst 3. Normal to sunken anterior fontanel 4. Absence of tears

: 3 The anterior fontanel closes between 16 and 18 months of age. In the infant with moderate dehydration, the fontanel will be normal or sunken. A moderately dehydrated infant will have slowed capillary refill time of between 2 and 4 seconds. Intense thirst and absence of tears are typical of severe, not moderate, dehydration.

1303. A public health nurse is presenting an educational session to parents with one of the topics being hearing loss/deafness. Which is the most accurate statement that the nurse can make regarding this topic? 1. Routine screening for adequate hearing levels begins at age 1. 2. A child with an ear infection should be tested during the infection period to identify hearing loss. 3. Some children with a minimal hearing loss may be thought to have behavioral problems in school. 4. Cerumen in the ear canal has been documented to substantially decrease hearing.

: 3 The child with a minimal hearing loss may be thought to have behavioral issues because he or she does not follow the instructions of the teacher, participate in discussions in the classroom, or interact with other students. A newborn hearing test is done soon after birth, with routine screenings for adequate hearing levels beginning usually at age 3, not age 1. Children with an ear infection should be tested after the ears are clear, not while infected, to identify a temporary hearing loss. There is no support that cerumen in the ear canal substantially decreases hearing.

1369. An emergency department nurse is triaging a group of pediatric clients. Which child should a nurse address first? 1. A child with periorbital edema that is worse in the morning 2. A previously well child who begins to gain weight insidiously over a period of days 3. A child with a recent renal transplant who has developed fever, local tenderness, decreased urinary output, and elevated blood pressure 4. A child with fever, foul-smelling urine, dysuria, and frequency and urgency on urination

: 3 The child with recent renal transplantation with these symptoms needs to be evaluated immediately for possible rejection. All of the children presented in the other options require prompt evaluation, but are not the first client to be seen.

1288. A 4-year-old child is being seen in a clinic. A parent explains that the child has been bumping into objects not directly in front of the child and complaining of seeing halos around objects and sometimes seeing two objects when there is only one. A referral is made for a tonometry test. A nurse explains to the parent that tonometry of the child's eyes will be testing for: 1. cataracts. 2. strabismus. 3. glaucoma. 4. lazy eye.

: 3 The child's symptoms suggest glaucoma. Tonometry is a method for measuring intraocular pressure (IOP) and detecting glaucoma. Other symptoms of glaucoma include excessive tearing, light sensitivity (photophobia), closure of one or both eyes in the light, one eye may be larger than the other, vision loss, irritability, fussiness, poor appetite, blurred vision, and a cloudy, enlarged cornea. Although some symptoms exhibited are similar to those of cataracts, strabismus, and lazy eye, tonometry is not used to detect these. A characteristic of cataracts is lens opacity, which is not present. While symptoms of bumping into objects and diplopia can occur with strabismus (cross-eyes), it is not evaluated by tonometry. Lazy eye results when one eye does not receive sufficient stimulation and poor vision results in the affected eye; a child may bump into objects because of the poor vision or exhibited diplopia.

1312. A child with myelodysplasia has a TEV (talipes equinovarus) repair that requires a cast application. In the postoperative period, a nurse notes serosanguineous drainage on the cast. What should the nurse do when making this observation? 1. Cut a window where the drainage is seeping through the cast 2. Petal the cast to minimize skin irritation and decrease leakage 3. Measure the area of drainage and document this finding 4. Notify the surgeon

: 3 The circumference of the drainage should be measured and the information documented for further comparisons. It is not the nurse's responsibility to place a window in the cast. Windowing may be used to relieve pressure. Petaling the cast, or bending the edges of the cast away from the skin, might make the cast more comfortable for the child, but it won't help with the drainage. Serosanguineous drainage on the cast is not an emergency situation unless it becomes bloody and copious.

1232. A public health nurse is caring for a 10-year-old child who is diagnosed with hepatitis A. The nurse is instructing the parents to avoid giving their child oral medications. Which is the nurse's rationale for giving this instruction? 1. The child does not need pain medications because there is no pain associated with hepatitis A. 2. The medication of choice is antibiotics, and the child will be on those only while hospitalized. 3. Normal medication doses may become dangerous due to the liver's inability to detoxify and excrete them. 4. The foods provided will contain all of the natural substances the child will need for recovery.

: 3 The function of the liver is altered due to liver damage in hepatitis, and medications cannot be metabolized and detoxified by the liver and excreted from the body. The child is likely to experience epigastric or right upper-quadrant pain in hepatitis, so the child will need pain medication. The child probably will not need antibiotics because hepatitis is viral in origin. Foods are for nutritional needs and will not provide the necessary analgesia for pain control.

1304. A nurse is working in a pediatric clinic where a physician has just removed a peanut from the ear of a preschool child. When teaching the child and parent postprocedure, what is the most important action for the nurse? 1. Discussing methods with the parent for preventing the child from placing a foreign body in the ear again 2. Talking to the child about potential complications of placing a foreign body in the ear again 3. Addressing postprocedural care with the child and the parent 4. Talking to the parents about foreign body objects that can cause obstructions in the ears and other orifices

: 3 The immediate need for the parents is understanding postprocedure care for their child. Developmentally, the child should be involved in the teaching to provide some control over the situation. Discussing prevention measures, potential complications, and other objects that can cause obstruction are correct actions, but are more appropriately done in a follow-up visit with the primary care provider

1268. A child is presenting with burn injuries. What should be the nurse's priority during the initial assessment? 1. Inspect location, extent, and shape of burn injuries. 2. Assess the child's and family's concerns regarding the child's appearance. 3. Assess for signs of smoke inhalation and burns to the face and neck. 4. Assess for signs and symptoms of infection.

: 3 The initial emergency assessment should be the assessment of airway, breathing, and circulation (ABCs). It is imperative to ensure that the airway has not been compromised by smoke or edema related to neck and facial burns. Although inspecting burn injuries should be included in the initial assessment, it should not be the first priority. The child's and family's concerns should be addressed once the physiological status of the child is stabilized. Infection should be a concern after the first 24 hours.

1121. A school nurse is presenting the latest information related to driving under the influence (DUI) of alcohol to high school students. The nurse informs the students that all 50 states and the District of Columbia have laws defining it as a crime to drive with a blood alcohol concentration (BAC) at or above a prescribed level of: 1. 0.1%. 2. 1.0%. 3. 0.08%. 4. 0.8%.

: 3 The level of 0.08% is considered an impaired blood alcohol level. A 0.1% blood level is too low, and 0.8% and 1.0% blood levels are too high.

1339. An adolescent client diagnosed with a T12 spinal cord injury (SCI) is admitted to a rehabilitation unit. A nurse is teaching the client about the need to be diligent in skin protection. The nurse explains that the primary reason for the client's increased risk for alterations in skin integrity is: 1. the inability to perceive extremes in temperature leading to burns. 2. the circulatory changes that cause vasoconstriction and decreased blood supply. 3. the inability to feel skin irritation such as wrinkled clothing. 4. the increased likelihood of bowel and bladder dysfunction and skin irritation.

: 3 The most common cause of altered skin integrity is due to lack of perceived stimuli and sensations to promote movement. The person must be taught to plan movement, even in the lack of a physical prompt (e.g., discomfort) to move. While there are circulatory and sensation changes with a spinal cord injury, these are not the most common cause for altered skin integrity. There is usually a bowel and bladder dysfunction with an SCI; however, good hygiene and a bowel and catheterization regimen minimize the likelihood of this as a cause of skin breakdown.

1295. A public health nurse is presenting an educational session to parents of preschool children about conjunctivitis. Which statement from a parent following the presentation indicates the parent understood the most important point about conjunctivitis? 1. "Viral conjunctivitis is self-limiting." 2. "The most common cause of conjunctivitis is presence of a foreign body." 3. "Prevention of infection in other family members is an important consideration with bacterial conjunctivitis." 4. "Conjunctivitis can occur during the birth process."

: 3 The most common cause of conjunctivitis is a bacterial infection. With this infection, transmission to other family members and to people in direct contact with the child can easily occur. Therefore the washcloth and towel the child uses should be kept separate from the others, tissues used should be disposed of immediately, and good hand-washing techniques should be followed by everyone. Viral conjunctivitis is selflimiting, but the fact that bacterial conjunctivitis is extremely contagious is the most important information. Conjunctivitis can occur after birth because of exposure to bacteria during the birthing process, but the audience being addressed is parents of preschoolers.

1132. A nurse is interpreting an ECG rhythm strip for a 2-year-old child with heart failure secondary to a congenital heart defect. In analyzing the rhythm, the nurse notes the measurements of PR interval is 0.26 seconds, the QRS is 0.08 seconds, and the QT is 0.28. The ventricular rate is 126 bpm. A nurse interprets the rhythm as: 1. sinus bradycardia. 2. sinus rhythm with a bundle branch block. 3. sinus rhythm with a first-degree AV block. 4. sinus tachycardia with a first-degree AV block.

: 3 The normal heart rate for a 2-year-old is 80 to 130 bpm. A normal PR interval measures 0.12 to 0.20 seconds. The QRS is normal (0.6 to 0.10 seconds), and the QT is rate dependent. If the rate is fast, the QT will be shorter. It is within the normal range for the ventricular rate. The ventricular rate in sinus bradycardia for a 2-year-old should be less than 80. In a bundle branch block, the QRS interval should be greater than or equal to 0.12 seconds. In sinus tachycardia, the ventricular rate should be greater than 130 bpm for a 2-year-old.

1161. Before administering oral digoxin (Lanoxin®) to a pediatric client, a nurse notes that the child has bradycardia and mild vomiting. Which is the nurse's most appropriate action? 1. Explain to the parent that bradycardia is an expected effect of the digoxin. 2. Administer the medication, document the observations, and reevaluate after the next dose. 3. Withhold the medication and immediately notify the prescriber because these are signs of toxicity. 4. Administer an oral beta-blocker medication.

: 3 The nurse should hold the medication and immediately notify the prescriber because these are signs of digoxin toxicity. Digoxin slows and strengthens the heart. Though digoxin slows the heart rate, bradycardia is a side effect. The medication should be held if the heart rate is slow. Continuing to administer the medication would be unsafe. A beta-blocking agent should not be administered because it may further slow the rate. If symptomatic, epinephrine would be prescribed to increase the heart rate.

1244. During a routine physical examination, a parent states to a nurse, "When I am taking pictures of my baby using the camera flash, I see a red coloration to my baby's left eye but the right eye has a white reflection. Is this normal?" Which response by the nurse is correct? 1. "Yes, the white reflection is normal; sometimes the light from the camera flash only catches one eye directly." 2. "Interesting. Your baby's eyes may be changing color. Many babies are born with what appears to be blue eyes but later they change to brown." 3. "It is good that you brought this to our attention because it is not the usual response. After further examining your baby's eyes, we can discuss what the white reflection may suggest." 4. "You seem concerned that your baby's eyes have different responses to the flash of the camera. Tell me more about your concern."

: 3 The presence of a white reflection, rather than the normal red pupillary reflex, is a classic sign of retinoblastoma. Further examination is needed to specify a diagnosis. This response acknowledges the mother's concern, yet lets the mother know that this is not the usual response and that further follow-up is needed before her question can be answered. The nurse should not tell the mother that a white reflection is normal when it is not a normal finding. Stating the baby's eyes may be changing color is an incorrect interpretation by the nurse. Stating that the mother seems concerned is using a therapeutic communication technique but does not address the mother's question.

1413. A nurse is preparing an educational program on immunizations for parents of children 11 to 12 years of age. To ensure the information presented is accurate for this age group, which immunizations should the nurse plan to address? 1. Mumps, measles, and rubella (MMR); diphtheriatetanus-pertussis (Dtap); and hepatitis B 2. Haemophilus influenza, varicella, and human papillomavirus (HPV) 3. Diphtheria-tetanus-pertussis (Dtap), meningococcal, and Haemophilus influenza 4. Mumps, measles, and rubella (MMR); pneumococcal (PPSV); and hepatitis A

: 3 The recommended immunization schedule for children 11 to 12 years old include a Dtap booster and meningococcal and Haemophilus influenzae vaccines. Others include HPV, PPSV, and hepatitis A series. Both MMR and varicella vaccines are administered at 12 to 15 months, with the second dose at 4 to 6 years. A hepatitis B vaccine is administered to all newborns prior to hospital discharge, with the second dose at 1 to 2 months and the third dose at 6 to 18 months. The first dose of hepatitis A vaccine is administered before 1 year of age, with the second dose 6 months after the first dose.

1335. A 10-year-old is scheduled to receive methotrexate (Rheumatrex®) to treat juvenile arthritis. Which laboratory findings should lead the nurse to decide to withhold the dose and contact the health-care provider? 1. Urine pH 7.4 2. Hemoglobin 13 g/dL 3. Serum creatinine 2.2 mg/dL 4. Alanine aminotransferase (ALT) less than 40 international units/L (U/L)

: 3 The serum creatinine is elevated, indicating a concern for excreting methotrexate. Urine pH should be kept above 7.0 to prevent renal damage. The hemoglobin and ALT values are normal. For a 10-year-old, normal hemoglobin is 10 to 15.5 g/dL and ALT is 4 to 36 U/L.

1122. The American Academy of Pediatrics lists safety tips for adolescents who drive a motor vehicle. When teaching a group of parents with teenagers, which statement should a nurse include in the teaching? 1. Nighttime driving is okay as long as the teenager is not fatigued. 2. A lack of experience will motivate the teenager to want to practice driving longer. 3. Transportation of other teenagers by a teenage driver should be avoided. 4. Although teenagers tend to use safety belts, reminders are always important.

: 3 The teenager should be discouraged from transporting other teenagers because the driver's alertness is decreased from the distraction. Motor vehicle accidents are the most frequent cause of unintentional accidents for adolescents. Nighttime driving is not okay for teenagers because it is more difficult to judge and to see objects at night. Although teenagers need practice driving with a reliable, adult teacher, the lack of experience does not motivate them to practice longer. Teenagers continue to be inconsistent with wearing of safety belts.

1189. Which nursing diagnosis has the highest priority for an infant diagnosed with congenital adrenal hypoplasia? 1. Disproportionate growth 2. Excess fluid volume 3. Impaired parent-infant attachment 4. Knowledge deficit of lifelong medication requirements

: 3 There is a high risk for impaired parental-infant attachment due to underdetermined gender identity. Although there may be growth issues later as the child grows and the parents must be able to demonstrate understanding of the treatments to provide appropriate care, these are of lesser priority. Fluid volume excess is not a problem.

1115. A nurse is teaching the parents of children between the ages of 2 and 3 years old about nutritional intake. The nurse should teach the parents that the percentage of the daily total intake of fat should be no more than which percentage? 1. 20% to 25% 2. 50% 3. 30% 4. 10% to 20%

: 3 Toddlers can have a higher fat level in their diet than older children. Therefore 30% is acceptable for nutrition and digestive needs. A diet of 50% fat is too high for proper health. Any fat intake less than 30% is insufficient for the child's digestion and absorption of nutrients.

1297. A nurse is counseling the parents of an infant who was born blind. Which statement indicates that a parent needs additional teaching? 1. "We or others will need to play with our infant so the infant will learn how to play." 2. "As our child grows we should attach Braille tags to clothing to help the child to learn to dress independently." 3. "We should seek the services of a speech therapist early because blind children also have difficulty with learning verbal skills." 4. "We have already discussed obtaining a seeing eye dog for our child so the child can get used to the animal at a young age."

: 3 Verbalizing the need for a speech therapist indicates that further teaching is needed. Blind children have little difficulty in learning verbal skills and are able to communicate with age-mates and participate in suitable activities. Because blind children cannot imitate others or actively explore the environment, they rely much more on others to stimulate them and teach them to play. Braille tags to clothing and other articles will help the child to distinguish items, colors, and prints. The use of a dog guide helps the child become independent in navigation.

1108. Which result should a nurse expect if a 4-year-old child's visual acuity test is normal for the child's developmental age? 1. 10/10 2. 20/20 3. 20/40 4. 40/40

: 3 Visual acuity is normally 20/40 during the toddler years. A visual acuity result of 10/10 or 20/20 is exceptional acuity for a toddler. Visual acuity would be 20/20 by age 6. A visual acuity result of 40/40 warrants further follow-up because the higher numbers indicate less visual acuity

1170. An adolescent client is taught how to use a continuous subcutaneous insulin infusion pump for tight glucose control of type 1 diabetes mellitus. Which statement by the client indicates the need for additional teaching? 1. "I can put in the number of carbohydrates that I consume, and the insulin pump will calculate the bolus insulin dose that I will receive." 2. "I must still check my blood glucose levels with meals and snacks and calculate the amount of carbohydrates I consume to ensure I get the correct bolus dose of insulin." 3. "As my blood glucose control improves with the use of the insulin pump, I should see a drop in the weight that I have gained." 4. "Every 2 to 4 days, I will need to change the syringe, catheter, and site moving the site away at least 1 inch from the last site."

: 3 Weight gain, and not weight loss, commonly occurs as blood glucose control improves. Newer pumps calculate bolus insulin dose to the carbohydrates consumed. Monitoring blood glucose levels and carbohydrates consumed is still necessary with an insulin pump. About every 2 to 4 days when a syringe is empty, a new syringe and tubing is attached to the pump along with a new skin setup. The site is also changed to prevent lipoatrophy.

1344. A nurse is educating the parent of a 6-month-old infant on bringing a urine sample to the health-care facility for examination. Which instruction indicates that the nurse is misinformed about the correct procedure? 1. Properly applying the collection device and storing the specimen is necessary. 2. Bring the specimen to the designated place as soon as possible. 3. Store the sample in a warm, dark place until brought to the facility. 4. Notify the examiner of the time lapsed between collecting and delivering the sample.

: 3 When a urine sample is brought into a health-care facility for examination, it is stored in the refrigerator if there is any lapse in time between collection and delivery. It is important to instruct the parent on properly applying the collection device and delivering the specimen as soon as possible because delays may alter the results of the urine testing.

1302. A nurse is planning care for a toddler with otitis media who is returning from surgery following a tympanostomy. Which nursing diagnoses should be addressed by the nurse first? Prioritize the nursing diagnoses by placing each in the correct order. ______ Interrupted family process related to illness and/or hospitalization of child, temporary hearing loss ______ Acute pain related to pressure caused by inflammatory process ______ Infection related to presence of infective organisms ______ Risk for delayed growth and development related to potential hearing loss

: 3, 1, 2, 4 Pain should be the primary nursing diagnosis immediately after surgery. The presence of pain influences vital signs and participation in measures needed for recovery. The next nursing diagnosis that should be addressed is infection related to the presence of infective organisms. This is second because measures should be taken to eliminate the infection. Next is interrupted family process related to illness and hospitalization. The final nursing diagnosis is risk for delayed growth and development.

1205. A clinic nurse is providing instructions to the parents of an 18-month-old child experiencing acute diarrhea. The child weighs 12 kilograms. When teaching the parents, which points should the nurse emphasize? SELECT ALL THAT APPLY. 1. "Have your child drink plenty of fluids, including apple juice and other fruit juices." 2. "Put your child on a diet of bananas, rice, applesauce, tea, and toast (BRATT) until the diarrhea resolves." 3. "Encourage your child to eat small amounts of foods included in the child's normal diet, except cow's milk or milk products." 4. "Avoid using commercial baby wipes that contain alcohol to cleanse your child's skin." 5. "Wash your hands often, especially after changing your toddler, and keep soiled articles away from clean areas." 6. "Give 1/2 glass (120 mL) of an oral replacement fluid, such as Pedialyte® for each diarrheal stool."

: 3, 4, 5, 6 A normal diet and appropriate administration of fluids is advised. While a normal diet may increase stool output initially, the stool pattern is outweighed by the benefits of a better nutritional outcome with fewer complications and a shorter illness course. Cow's milk and milk products can be irritating to the gut. Hand washing prevents the spread of gastroenteritis. Oral replacement solutions are used during maintenance fluid therapy and replacing fluid losses from diarrheal stools. Rehydration is 50 mL/kg during the first 4 hours and 10 mL/kg for each diarrheal stool. Fruit juices should be avoided because these contain a high amount of carbohydrates, pull circulating fluids into the gut, and can prolong diarrhea. A BRATT diet is low in energy and protein and no longer recommended.

1201. An infant is hospitalized with a diagnosis of infectious gastroenteritis and dehydration. A nurse determines that a nursing assistant caring for the infant understands the necessary precautions when the nursing assistant states: SELECT ALL THAT APPLY. 1. "I should put on a mask, gown, and gloves when I enter the room." 2. "I should put on gloves when I am holding the baby." 3. "I should wear gown and gloves to change the baby's diapers." 4. "I should keep the door to the baby's room closed most of the time." 5. "I should perform hand hygiene each time I change the baby's diaper." 6. "I should keep the baby in the room unless instructed otherwise."

: 3, 5, 6 Because the organisms causing gastroenteritis are eliminated in the feces, contact precautions should be used. This includes the standard precautions plus wearing a gown and gloves when soiling may be possible or when in contact with infected surfaces or items. The infant's movement outside the room is limited to only what is absolutely necessary. A mask is worn for droplet precautions and is unnecessary; the microorganisms causing gastroenteritis are passed in the feces. If soiling is expected when holding the baby, then gloves and a gown should be worn. If soiling or contact with infected surfaces or items is unlikely, neither a gown nor gloves is needed. The door is closed for airborne precautions; gastroenteritis is not airborne.

1378. A 2-year-old child, admitted to an acute care pediatric unit with a sore throat, is tentatively diagnosed with epiglottitis. Which diagnostic test should a nurse plan to review to confirm the diagnosis? 1. Blood culture 2. Complete blood count (CBC) 3. Throat culture 4. Lateral neck x-ray

: 4 A lateral neck x-ray will show the enlarged epiglottis. A blood culture will identify the organism and suggest antibiotic selection, but results will be delayed until the microorganism grows in the culture media. The CBC will identify elevated white blood cells and the presence of an infection but will not confirm the diagnosis. A throat culture is usually not performed because gagging can cause complete airway obstruction.

1227. A nurse is caring for a 10-year-old child who has been diagnosed with peritonitis secondary to a ruptured appendix. Which prescription by the healthcare provider (HCP) should the nurse question? 1. Irrigation of incision site bid with 0.9% NaCl solution 2. Empty and measure JP drain q8 hrs. or as needed 3. Continue IV fluids as previously requested and keep NPO 4. NG to high intermittent suction (HIS); empty and measure q8 hr.

: 4 A nasogastric tube (NG) is needed until there is evidence of intestinal movement; however, the NG on a child this age would be kept on low intermittent suction (LIS) and not high intermittent suction (HIS) to ensure that there would be no injury to the gastric lining. Irrigating the incision site, emptying the Jackson-Pratt drain, and continuing IV fluids and keeping the client NPO are all appropriate interventions.

1449. Methylphenidate hydrochloride (Ritalin®) is prescribed for a 6-year-old child with attention deficit hyperactivity disorder (ADHD). A nurse should teach the parents to administer the medication: 1. whenever the child exhibits inattention behaviors. 2. whenever the child exhibits hyperactive behaviors. 3. with a snack before bed to calm the child for sleep. 4. during or after meals if the medication decreases appetite.

: 4 A side effect of methylphenidate hydrochloride is anorexia. It should be given during or immediately after breakfast and lunch to prevent decreased intake of foods and fluids. Methylphenidate is usually administered twice daily at or before breakfast and at noon and not whenever inattention or hyperactive behaviors occur. Abrupt withdrawal of the medication may result in severe depression and psychotic behavior. The last dose of the medication should be given before 6 p.m. to prevent insomnia

1345. A 3-year-old client presents with vomiting and diarrhea for 24 hours. On routine urinalysis, which finding should indicate to a nurse that the child is dehydrated? 1. Specific gravity 1.000. 2. Specific gravity 1.010. 3. Specific gravity 1.020. 4. Specific gravity 1.030.

: 4 A specific gravity (SG) greater than 1.025 may indicate dehydration. The normal SG range is between 1.010 and 1.025. Options 1, 2, and 3 are less than 1.025. A low specific gravity can indicate dilute urine because as urine becomes more concentrated, its specific gravity increases.

1310. A nurse is administering multiple anticonvulsant medications to children. For which medication should a nurse teach the parents about ensuring that their child has good oral care to prevent gingival hyperplasia? 1. Carbamazepine (Tegretol®) 2. Valproic acid (Depakene®) 3. Phenobarbital 4. Phenytoin (Dilantin®)

: 4 About 20% of people taking phenytoin have gingival hyperplasia. This can be minimized with thorough oral care. Gingival hyperplasia is unique to phenytoin among antiepileptic medications and is not a side effect of carbamazepine, valproic acid, or phenobarbital.

1178. A child's parents inform a nurse about how they care for their 12-year-old child with type 1 diabetes mellitus, including sick day management, treating hyperglycemia, and managing ketosis. In which situation could the parents safely manage the child's care at home? 1. Child's blood glucose level is 280 mg/dL; skin turgor very poor; lips and mouth parched. 2. Child's blood glucose level is 250 mg/dL; vomiting and dizziness; complains of double vision. 3. Child's blood glucose level is 240 mg/dL; large amounts of urine output decreasing to 100 mL total output for the last 8 hours. 4. Child's blood glucose level is 300 mg/dL; urine tested positive for ketones; skin is hot, flushed, and dry.

: 4 According to the consensus statement from the American Diabetes Association, children who have established diabetes and who are in DKA without vomiting and severe dehydration can be managed at home if the parents are trained in sick day, hyperglycemia, and ketoacidosis management. In addition, the care must be supervised through the emergency department by an experienced diabetes team. The child should be managed in the hospital if showing signs of severe dehydration, vomiting, or cerebral involvement. Poor skin turgor and parched mucous membranes are signs of severe dehydration. Dizziness, double vision, and vomiting could be signs of cerebral involvement. Urine output less than 30 mL per hour is a sign of severe dehydration.

1393. A 3-year-old child is brought to an emergency department with acute pulmonary edema. The child was seen in an emergency department 48 hours earlier for a near-drowning incident. Treatment was provided at that time, and the child was monitored and discharged home. Current chest radiography indicates diffuse bilateral infiltrates. There is no history of cardiopulmonary disease. Which pulmonary dysfunctions should a nurse think about when assessing the child? 1. Foreign body aspiration 2. Aspiration pneumonia 3. Bronchopneumonia 4. Acute respiratory distress syndrome

: 4 Acute respiratory distress syndrome is recognized in children as well as adults and has been associated with clinical conditions and injuries such as sepsis, viral pneumonia, smoke inhalation, and near drowning. It is characterized by respiratory distress and hypoxemia occurring within 72 hours of the injury. Foreign body aspiration occurs when any solid or liquid substance is inhaled into the respiratory tract. It is common in infants and young children and can present in life-threatening acute situations. Aspiration pneumonia is caused by aspiration of meconium or amniotic fluids during the birth process. Bronchopneumonia is often basilar, affects the lower lobes of the lungs, and is often nosocomial or community acquired.

1102. A nurse case manager is meeting with the parents of an 8-year-old client. The 8-year-old is scheduled for surgery to repair a cleft palate. The parents ask the case manager when they should discuss and explain the surgery to their child. Based on the child's developmental age, which is the best response by the nurse? 1. Explain the surgery immediately before it is carried out. 2. Explain the surgery 1 to 2 hours before it is carried out. 3. Explain the surgery up to 1 week before it is carried out. 4. Explain the surgery several days before it is carried out.

: 4 An 8-year-old can receive teaching several days before surgery. School-age children have concrete operational thought and can remember events. A preschooler will remember explanations for only a couple of hours. A toddler has limited attention span and limited ability to remember things. An adolescent can handle information for the longest period of time..

1135. A nurse is preparing to perform an electrocardiogram (ECG) on several pediatric clients. Which client would not benefit from an ECG? 1. A 4-year-old with tachycardia 2. A 3-year-old with bradycardia 3. A 10-year-old with an irregular pulse 4. An infant with a splitting of the S2 heart sound only when the infant takes a deep breath

: 4 An infant with a splitting of the S2 heart sound when the child takes a deep breath would not benefit from an ECG because it is a normal finding in infants and young children. ECG is useful in diagnosing tachycardia, bradycardia, and irregular pulses. These rhythms can be recorded through an ECG.

1411. A nurse is formulating a preoperative plan of care for a 5-year-old child who is scheduled for a tonsil and adenoidectomy (T&A). Which nursing diagnosis is priority? 1. Imbalanced nutrition: less than body requirements related to surgery 2. Pain related to surgery 3. Ineffective airway clearance related to hesitation or reluctance to cough secondary to pain 4. Anxiety related to surgery

: 4 Anxiety related to surgery is a preoperative diagnosis. Anxiety is a key element for children admitted to the hospital. Options 1, 2, and 3 are all considered postoperative diagnoses and are applicable following the procedure.

1096. An 8-month-old baby girl, who is developing appropriately, is admitted to a pediatric unit for respiratory syncytial virus (RSV). The baby is crying and being held by her mother. A nurse wants to provide appropriate care based on Erikson's developmental stages. In which stage is this baby, according to Erikson's theory? 1. Punishment versus obedience orientation 2. Oral stage 3. Initiative versus guilt 4. Trust versus mistrust

: 4 Based on Erikson's stages of development, trust versus mistrust is appropriate for a child under a year old. The child learns to love and be loved. Experiences that add to security include soft sounds, touch, and visual stimulation for an active child. Punishment-versus-obedience orientation is not a developmental stage. Initiative versus guilt is a developmental task in the preschool stage. The oral phase is based on Freud not Erikson.

1372. An adolescent with a history of a renal transplant has not been adhering to the pharmacological regime. The result has been damage to the kidney and loss of the graft. The multidisciplinary team is considering whether the client should receive a second transplant. Which course of action is most appropriate for the nurse to consider next? 1. Allow the client to decide. 2. Allow the managing health-care provider (HCP) to decide. 3. Follow through with the procedure only if the client can cover the costs. 4. Refer the case to the institution's ethics committee.

: 4 Because of a scarcity of donors, expense of the procedure, and costly medications, this case should be referred to the institutional ethics committee. Actions may include discussing the decision with the client and HCP, but allowing either to decide is not the next action. Performing another transplant only if the client can pay is unethical.

1350. A nurse is caring for multiple clients preparing for placement of an external diversional urinary system. Which client has the greatest need for interventions to promote a positive body image? 1. An infant with spina bifida 2. A toddler who has recently been toilet trained 3. A school-aged child in foster care 4. An adolescent who has recently become sexually active

: 4 Because of the developmental stage, an external device would put an adolescent at the highest risk for body image disturbance, and therefore that age group has the greatest need for interventions. According to Havighurst's age periods, an adolescent's developmental task is accepting one's physique and using the body effectively. The developmental stage according to Erikson's Developmental Task Theory is to achieve a coherent sense of self. An infant's developmental stage is trust versus mistrust. The infant is unaware of body alterations. A toddler's developmental level is autonomy versus shame and doubt and he or she could be at risk for altered body image, depending on the response of parents and others. A school-aged child's developmental level is industry versus inferiority. The child would be at risk for body image disturbance if unable to develop competence with the device or not achieving a sense of wholeness (one of Havighurst's developmental tasks).

1415. A new nurse asks an experienced nurse why the first dose of the measles, mumps, and rubella (MMR) vaccine is given only between 12 to 15 months of age and not any earlier. Which explanation by the experienced nurse is correct? 1. "The second dose of the vaccine is given before a child reaches puberty, and giving the first dose of the vaccine at 12 to 15 months of age allows the correct interval between vaccinations." 2. "Because a live virus is administered, the chance of developing measles, mumps, or rubella is much higher if given at an earlier age." 3. "A first dose at this age provides passive immunity and decreases the incidence of a child developing any of the diseases." 4. "If administered earlier, the vaccine will neutralize the passive immunity to measles from the child's mother and no immunity will result."

: 4 Because the MMR vaccine is a live virus, a person develops a mild form of the diseases after administration, stimulating the body to develop immunity. The passively acquired antibodies to measles can interfere with the child's immune response to the vaccine, and no immunity will result. The second dose of the MMR vaccine can be given earlier, provided that at least 4 weeks has elapsed since the first dose. However, a second dose is usually not given earlier because sufficient immunity is usually present. The chance of developing measles is greater only if the vaccine is given at a younger age because the vaccine may neutralize the passive antibodies. The MMR provides active (not passive) immunity.

1259. A nurse is planning care for a child following removal of brain tumor. The child is confused, disoriented, and restless. Which nursing diagnosis should receive the highest priority? 1. Sensory perceptual alterations related to neurological surgery 2. Self-care deficit related to confusion and restlessness 3. Impaired verbal communication related to confusion 4. Risk for injury related to disorientation and restlessness

: 4 Because the child is experiencing altered cognition, the priority is promoting client safety and preventing injury. The remaining nursing diagnoses are pertinent but not the priority.

1316. A nurse is caring for a child immediately following insertion of a ventriculoperitoneal (VP) shunt for treatment of hydrocephalus. The nurse's postoperative care should include: 1. maintaining the head of the bed in an elevated position. 2. ensuring that the child minimizes movement of the extremities. 3. providing a pressure dressing over the cephalic insertion site. 4. changing the child's position every 2 hours.

: 4 Because the child should be placed in a flat position following insertion of the shunt, it is necessary to protect the child's skin by turning the child every 2 hours. An elevation of the head of the bed is contraindicated because it will increase the flow from the shunt. Range of motion of the extremities should be done as the child recovers. A clear dressing (not a compression dressing) is placed over the incision sites to allow observation. ➧ Test-taking Tip:Think about the immediate postoperative position of the child before making a selection.

1273. A nurse is reviewing orders received for a newly admitted child with second- and third-degree burns over 10% of the total body surface area (TBSA). The child weighs 20 kg. The nurse should seek further clarification from a physician when the physician's order is: 1. Ringer's lactate (RL) at 50 mL per hour for the next 8 hours. 2. insert a urinary catheter. 3. elevate the extremities above the level of the heart. 4. morphine sulfate IV prn for pain control.

: 4 Because the order for morphine sulfate does not state a dose, the order should be clarified with the physician. If the physician intended the dose to be based on the weight of the child, then this should be included in the order. In the first 24 hours, fluid resuscitation is 4 mL/kg body weight per percentage of burn TBSA, with half over the first 8 hours and the remaining over the next 16 hours (4 mL 20 kg = 80 mL; 80 mL 10 = 800 mL for 24 hours; half of this is 400 mL over 8 hours; 400 ÷ 8 = 50 mL). A Foley urinary catheter is inserted so that urine output can be closely monitored as a guide for volume status. During the resuscitation phase, edema formation can decrease perfusion. Elevating the limbs above the heart level promotes gravity-dependent drainage.

1313. A nurse is reinforcing teaching to the parents of a child with myelomeningocele, which was diagnosed at birth and surgically corrected, about safety considerations. The nurse's instructions should include: 1. making sure that braces lie smoothly against the child's skin. 2. teaching the child to shift position at least every 3 hours. 3. placing a blanket between the child and the wheelchair. 4. checking all of the child's skin daily for redness or irritation.

: 4 Because with a myelomeningocele the spinal cord ended at the point where the spinal cord and the meninges protruded through the vertebrae, the child's motor and sensory function are absent beyond this point. A daily thorough skin check is the best way to evaluate circulatory and skin alterations due to the child's immobility, altered sensation, or therapeutic treatments, such as braces. Braces should never be directly against the skin; there should always be a cloth between the braces and the skin. The child should be taught to shift every hour (not every 3 hours). The child should sit on a gel cushion because blanket folds can interrupt circulation.

1401. A nurse is preparing to perform chest physiotherapy on a 7-year-old client diagnosed with cystic fibrosis. When should the nurse plan to perform the treatment? 1. Before performing postural drainage 2. Before a nebulized aerosol treatment 3. After suctioning the upper respiratory tract 4. One hour before meals

: 4 Chest physiotherapy is done between meals to prevent esophageal reflux. Postural drainage is most effective after treatments and chest physiotherapy because secretions have been loosened. Nebulizer treatments help to loosen secretions and are beneficial prior to chest physiotherapy. Suction is utilized after treatments and therapy to help clear the airways.

1169. A nurse explains to a parent who has a child with type 1 diabetes mellitus that the most important reason for counting the child's grams of carbohydrate intake is to: 1. lower blood glucose levels. 2. supply energy for growth and development. 3. provide consistent glucose to prevent hypoglycemia. 4. attain metabolic control of glucose and lipid levels.

: 4 Children need energy for growth and development. A child who has type 1 diabetes mellitus needs a consistent intake of carbohydrate; but the overall goal of nutritional management is to achieve and maintain control of glucose and lipids metabolism. The other alternatives are only a partial answer to the question.

1394. A child with asthma is being discharged to home and has an order for a bronchodilator (albuterol) to be administered via a metered dose inhaler (MDI). Which point should a nurse address for appropriate administration of this medication? 1. When administering medication via a MDI, avoid shaking the canister before discharging the medication. 2. Medication is ordered in two "puffs"; press on the canister twice in succession to discharge the medication. 3. There should be a tight seal around the mouthpiece of the inhaler before discharging the medication. 4. There should be a 2- to 3-inch space (or spacer device) between the inhaler and the open mouth of the child.

: 4 Children often have difficulty learning to depress and inhale their medications at the same time, and holding the MDI 2 to 3 inches away from the mouth or utilizing a "spacer" (an attachable device that provides space and contains the medication in a confined area) improves the effects of the medication. Shaking the MDI canister well before use supplies a better delivery of the aerosolized medication. When using two "puffs" of medication, waiting 1 minute between puffs allows for better absorption of the inhaled medication. When using inhaled medications via an MDI, the client should be instructed that wrapping the lips tightly around the mouthpiece consolidates the medication in the buccal cavity and decreases the effectiveness of inhaled medications.

1398. A nurse is caring for a 5-year-old child diagnosed with bronchial asthma. Which statement is most important for the nurse to make when teaching the parents? 1. "Bronchial asthma is also called hyperactive airway disease." 2. "Frequent occurrences of bronchiolitis before 5 years of age could be a sign of asthma." 3. "Severe respiratory alkalosis can result from respiratory failure in asthma." 4. "Severe bronchoconstriction can occur when exposed to cold air and irritating odors."

: 4 Children with asthma can have sensitization to inhalant antigens such as pollens, molds, house dust, food, and exposure to cold air or irritating odors, such as turpentine, smog, or cigarette smoke. Although bronchial asthma is also called hyperactive airway disease, this is not the most important statement. Asthma tends to occur initially before 5 years of age, but in the early years it may be diagnosed as bronchiolitis rather than asthma. Severe respiratory acidosis, not alkalosis, can result from respiratory failure in asthma.

1427. An adolescent, who is receiving morphine sulfate via a patient-controlled analgesia (PCA), complains of itching. Which medication should a nurse plan on administering to relieve the itching? 1. Naloxone hydrochloride (Narcan®) 2. Diazepam (Valium®) 3. Butenafine hydrochloride (Mentax®) 4. Diphenhydramine (Benadryl®)

: 4 Diphenhydramine is an antihistamine that blocks histamine release by competing for the histamine receptors. Naloxone is a narcotic antagonist that reverses the effects of opiates. Diazepam acts on the central nervous system to produce sedation, hypnosis, skeletal muscle relaxation, and anticonvulsant activity. Butenafine is an antifungal antibiotic used to treat tinea pedis, tinea corporis, and tinea cruris.

1452. A nurse is providing information to parents whose 3-year-old child has been treated with vincristine sulfate for the diagnosis of Wilms' tumor. In teaching the parents, the nurse should inform the parents to immediately notify the health-care provider if: 1. the child's hair begins to fall out. 2. the child develops diarrhea. 3. the child has signs or symptoms of depression. 4. the child develops dysphagia and paresthesias.

: 4 Dysphagia and paresthesias are central nervous system adverse effects from vincristine that can result in injury to the child. Hair loss is a common adverse reaction to the medication and is reversible. Both diarrhea and severe constipation are adverse effects of vincristine, and prophylactic treatment is implemented at the beginning of therapy to decrease the potential of these occurring. Three-year-olds may not show signs or symptoms of depression. If present, the signs and symptoms should be distinguished as being associated with the neoplastic disease itself or side effects of the medication.

1443. A 6-year-old child is to start on medication therapy for enuresis that has not resolved with behavioral interventions. Which medication should a nurse anticipate being prescribed for the child? 1. Nitrofurantoin (Furadantin®) 2. Spironolactone (Aldactone®) 3. Lorazepam (Ativan®) 4. Desmopressin (DDAVP, Stimate®)

: 4 Enuresis is involuntary passage of urine past the age when bladder control should be expected. Desmopressin is an analog of arginine vasopressin, which acts as an antidiuretic. It promotes resorption of water in the renal tubule or decreases bladder filling. Nitrofurantoin is a urinary tract antiseptic used to treat urinary tract infections. Spironolactone is a potassium-sparing diuretic that would promote, not inhibit, diuresis. Lorazepam is a benzodiazepine used to treat anxiety. A side effect is drowsiness, which could impair a child's ability to waken for voiding.

1241. Which finding should a nurse expect when reviewing the laboratory results of an infant newly diagnosed with hemophilia A? 1. Prolonged prothrombin time (PT) 2. Decreased hemoglobin level 3. Decreased hematocrit level 4. Prolonged activated partial thromboplastin time (aPTT)

: 4 Hemophilia A results from a deficiency of factor VIII. Because aPTT measures the activity of thromboplastin and factor VIII is deficient, the aPTT is prolonged. The PT measures the action of prothrombin and will be prolonged with deficiencies of prothrombin and factors V, VII, and X. The hemoglobin and hematocrit is not used to diagnose hemophilia A and should be normal unless the child is bleeding.

1318. A health-care provider's progress notes states that an infant with meningitis is in an opisthotonus position. The nurse should expect to observe: 1. resistance with specific leg movement. 2. knee or hip flexion with head flexion. 3. a high-pitched cry with neck flexion. 4. hyperextension of the head and neck.

: 4 In an opisthotonus position the child's neck and head are hyperextended to relieve discomfort. Resistance with particular leg movement is Kernig's sign; Brudzinski's sign is knee or hip flexion with head flexion. The specific quality of cry with neck flexion is not characteristic of opisthotonus.

1243. A child has iron overload from receiving multiple blood transfusions for treating thalassemia major. A nurse should anticipate the physician will likely: 1. order intravenous (IV) fluids to dilute the excess iron and increase urinary excretion. 2. change the type of blood product being transfused. 3. reduce the frequency of blood transfusions. 4. begin chelation therapy.

: 4 In chelation therapy, an iron-chelating drug, such as deferoxamine, is administered to bind excess iron so it can be excreted by the kidneys. Hemodilution, changing blood products, and reducing the frequency of transfusions do not treat the iron overload. IV fluids will not increase the urinary excretion of iron.

1432. An initial treatment regimen of isoniazid (Laniazid®), rifampin (Rifadin®), and ethambutol (Myambutol®) are prescribed for a 16-year-old client who has a positive tuberculin skin test. The client confides that she thinks she may have become pregnant since she was diagnosed and asks if she should be taking the medication while pregnant. On which rationale should a nurse base a response to the client's question? 1. These drugs cross the placental barrier and treatment should be withheld until the postpartum period. 2. The medications should be taken but the diagnosis is an indication for termination of the pregnancy. 3. The medications should be postponed because the risk for hepatitis is greatly increased in the intrapartum period. 4. The medications should be taken because untreated tuberculosis represents a far greater hazard to a pregnant woman and her fetus than does the treatment of the disease.

: 4 Infants born to women with untreated tuberculosis may be of lower birth weight than those born to women without tuberculosis, and, rarely, the infant may acquire congenital tuberculosis. Isoniazid, rifampin, and ethambutol are all considered safe for use in pregnancy as reported by the Centers for Disease Control and Prevention (CDC). The medications do not cross the placental barrier, so treatment should not be withheld. Administering antituberculosis medications would not be an indication for termination of pregnancy because the medications are safe during pregnancy. The risk of hepatitis is slightly increased with the use of antituberculosis medications in pregnant women; however, the benefits of treatment strongly outweigh postponement of treatment.

1130. An emergency department nurse is assessing a pediatric client suspected of having acute pericarditis. Which assessment finding should the nurse conclude supports the diagnosis of acute pericarditis? 1. Bilateral lower extremity pain 2. Pain on expiration 3. Pleural friction rub 4. Pericardial friction rub

: 4 Inflammation of the pericardial sac from acute pericarditis produces a pericardial friction rub. Decreased perfusion to the extremities can cause extremity pain, but this does not occur with pericarditis. Pain on inspiration, not expiration, is present with pericarditis. The friction rub is pericardial, not pleural.

1439. A 12-year-old hospitalized child, diagnosed with type 1 diabetes mellitus, is learning to use insulin pens for basal-bolus insulin therapy with both a very long-acting insulin and rapid-acting insulin. The short-acting insulin is administered with each meal and snack based on the carbohydrate grams consumed, and the long-acting insulin is administered at bedtime. Which action by the child should indicate to a nurse that additional teaching is needed? 1. The child counts the number of carbohydrates eaten at breakfast and selects the insulin lispro (Humalog®) pen for covering the carbohydrates eaten. 2. The child determines that the blood glucose level at bedtime is within the normal range, eats a piece of turkey, and tells the nurse that rapidacting insulin coverage is not needed. 3. The child holds the insulin glargine (Lantus®) pen against the skin for 10 seconds after administering the correct amount of insulin. 4. The child counts the number of carbohydrates eaten at lunch and selects the insulin glargine (Lantus®) pen for covering the carbohydrates eaten.

: 4 Insulin glargine (Lantus®) is very long-acting insulin administered once daily. Insulin lispro (Humalog®) is rapid-acting insulin with an onset of 5 to 10 minutes. Either rapid-acting or short-acting (regular) insulin is administered with each meal and snack based on the carbohydrate grams consumed and the before-meal blood glucose level. Turkey does not contain carbohydrates; bolus insulin is only administered to cover the carbohydrates eaten. To ensure that the medication is administered with the insulin pens, the pen is held in place for 10 seconds after delivery of the medication.

1305. A preschool-aged child who has purulent, foul-smelling drainage from the nares is brought to an urgent-care clinic. The first action a registered nurse should take is to: 1. ask the child to tell the story of what happened. 2. obtain a set of vital signs. 3. provide comfort for the child. 4. complete a focused physical examination.

: 4 Judging by the quick interpretation of the presenting symptoms, the child most likely has a foreign body in one or two nare(s). The focused assessment would provide information to determine what instruments would be appropriate for removal of the object. The child could provide the story of what happened, but the needed information is the time frame in which the odor was noted by the parent. Preschoolers do not have the concept of time and could not provide that information. Obtaining a set of vital signs is not an appropriate first intervention. If there is concern for infection, obtaining a temperature could be significant. Providing comfort to the child is extremely important and should be done along with the physical examination.

1328. A 7-year-old has had hip pain for several months. Because it was mild pain, the parent did not pay a great deal of attention. The child was ultimately given a diagnosis of Legg-Calvé-Perthes disease. In preparing the child and family for treatment, the nurse should instruct the parents that: 1. most of the child's treatment will be done while the child is hospitalized. 2. activities that promote hip adduction are encouraged. 3. treatment is likely to continue for about 6 months. 4. the desired outcome is a pain-free joint with full range of motion.

: 4 Legg-Calvé-Perthes disease is avascular necrosis of the proximal femoral epiphysis occurring in association with incomplete clotting factors. The treatment goal is full function. Treatment is community based rather than hospitalization. Abduction, not adduction, allows proper placement of the joint. Treatment of the disease is likely to occur for as long as 2 years.

1320. What is the most important factor for a nurse to consider when teaching a child with cerebral palsy (CP)? 1. Age 2. Type of cerebral palsy 3. The child's prior experiences with illness 4. Developmental level

: 4 Many children with CP have some degree of cognitive impairment. This makes considerations of age and experience much less salient variables. Cognitive impairment also occurs across types of CP.

1255. A nurse suspects that a 10-year-old client diagnosed with non-Hodgkin's lymphoma (NHL) has superior vena cava syndrome when assessing that the client has: 1. thrombocytopenia and leukocytosis. 2. hyperuricemia, hypocalcemia, and hyperphosphatemia. 3. tingling and paresthesias of the lower extremities and pain on light touch. 4. cyanosis of the upper chest, neck, face, upper extremity edema, and distended neck veins.

: 4 Mediastinal tumors, especially from NHL, may cause compression of the great vessel (superior vena cava syndrome). Signs of compression include cyanosis of the upper chest, neck, face, upper extremity edema, and distended neck veins. Thrombocytopenia and leukocytosis increases the risk for vascular damage and life-threatening hemorrhage. Hyperuricemia, hypocalcemia, and hyperphosphatemia are signs of tumor lysis syndrome. A mass obstructing the spinal cord can be manifested by symptoms of tingling and paresthesias of the lower extremities and pain on light touch.

1337. During a physical examination of a 1-month-old infant, a nurse notes that the infant has blue sclerae. The nurse suspects that the infant may have: 1. juvenile arthritis. 2. Tay-Sachs disease. 3. muscular dystrophy (MD). 4. osteogenesis imperfecta.

: 4 Newborns not uncommonly have sclerae that appear blue; however, in children older than newborns, blue sclerae are an indicator for osteogenesis imperfecta, a genetic disorder that causes fragile bones as well as other health problems. For a child who has not yet fractured a bone, blue sclerae may be the first sign that promotes a further workup. Blue sclerae are not present in Tay-Sachs disease, MD, or juvenile arthritis. Juvenile arthritis is a chronic inflammation of the joints that can lead to damaged joint cartilage and bone, causing deformity and impaired use of the joint. It is believed to be caused by a combination of genetic and environmental factors. Tay-Sachs disease is a fatal genetic lipid storage disorder in which infants appear to develop normally for the first few months of life. Then, deterioration of mental and physical abilities occurs as nerve cells become distended with fatty material. MD is a genetic disease causing progressive weakness and skeletal muscle degeneration of muscles used during voluntary movement.

1163. A nurse is administering medications to a pediatric client with hypertension. Which oral antihypertensive medication ordered for a child should the nurse question? 1. ACE inhibitor 2. Calcium channel blocker 3. Diuretic 4. Nitrate

: 4 Nitrates are used to treat angina and not prescribed to treat hypertension in children. ACE inhibitors, beta blockers, calcium channel blockers, angiotensin-receptive blockers, and diuretics are all oral antihypertensive medications used in treating hypertension in children.

1212. A nurse completes teaching the parents of a 3-month-old infant diagnosed with pyloric stenosis who underwent surgical correction. Which statement by the parents indicates teaching has been effective? 1. "We should use a special infant feeder, such as a Breck® feeder, so our baby does not get so much air." 2. "Increasing the amount of formula at each feeding will help to expand our baby's stomach." 3. "After feedings, our baby should be handled as little as possible." 4. "Once put back to bed after the feeding, our baby should be positioned on the right side."

: 4 Positioning side lying will prevent aspiration of vomitus; the right side aids stomach emptying through the pyloric valve by gravity. A Breck® feeder is used in feeding children with cleft lip or palate and is unnecessary. Only the amount of prescribed fluid should be given to preserve the integrity of the surgical repair site. The infant should be bubbled well after a feeding so there is no pressure from air in the stomach.

1238. A child diagnosed with aplastic anemia has had human leukocyte antigen (HLA) typing, evaluation of organ function, and laboratory studies completed as an outpatient. Which action should a nurse plan to implement first when the child is admitted to a transplant center for a hematopoietic stem cell transplant? 1. Checking the patency of the central line catheter 2. Placing the child in protective isolation 3. Ensuring that all food entering the child's room has been irradiated 4. Preparing the child to receive high doses of chemotherapy

: 4 Preparing the child both physically and psychologically is priority. The child will be given high doses of chemotherapy and sometimes total body irradiation to destroy circulating blood cells and the diseased bone marrow before the transplantation of donor stem cells. Chemotherapy, and later donor stem cells, will be administered by intravenous infusion through a central line catheter. Although patency must be checked prior to administration, preparing the child is priority. During chemotherapy and pancytopenia, strict protective isolation is maintained. Strict isolation includes irradiating food and sterilizing utensils and other items used in the room.

1248. A hospitalized child diagnosed with leukemia is being discharged after an initial treatment with chemotherapy. A nurse is teaching the parents about the allopurinol (Zyloprim®), which the child will continue to take at home. The nurse explains that the purpose of this medication is to: 1. help promote the child's sleep. 2. treat the joint pain and swelling caused by the child's gout. 3. prevent the child from developing gouty arthritis. 4. protect the child's kidneys by reducing the formation of uric acid.

: 4 Rapid cell destruction from chemotherapy results in a high level of uric acid being excreted during treatment. This can plug the glomeruli and renal tubules, causing loss of kidney function. While allopurinol can cause drowsiness, this is not the purpose for this child. There is no indication that the child has gout or is likely to develop gout.

1409. Which nursing diagnosis should have the highest priority for an infant newly diagnosed with transesophageal fistula? 1. Risk for impaired infant attachment 2. Risk for infection 3. Risk for imbalanced nutrition 4. Risk for aspiration

: 4 Risk for aspiration reflects a basic physiological need according to Maslow's Hierarchy of Needs and is the priority nursing diagnosis. Though infection and impaired nutrition are also physiological problems, aspiration and subsequent respiratory malfunction would be a higher priority. Although impaired infant attachment may occur in this situation, it would be a second or third level of need.

1195. A nurse is assessing a 4-year-old child diagnosed with precocious puberty. Which physical assessment manifestation should the nurse expect to find? 1. Short stature 2. Hypothalmic tumor 3. Advanced bone age 4. Pubic and axillary hair

: 4 Secondary sex characteristics are present in children under 8 years of age. They may also have a hypothalamic tumor and advanced bone age, but the nurse will not be able to assess those on physical exam. Children with precocious puberty do not have short stature, but are tall for their age.

1396. A nurse determines that a mother understands the benefits of using a peak flow meter when the mother states: 1. "I will have my child lie flat and obtain the meter reading each morning before getting out of bed, with the meter set on the average peak flow." 2. "I will have my child obtain the meter reading after completing a morning exercise routine to encourage better airflow before testing the peak flow." 3. "I will encourage my child to test the peak flow meter every day and set the meter at zero before testing the peak flow and record the reading once a month." 4. "I will have my child stand and 'huff and cough' two or three times to clear the airway and set the meter gauge on zero before beginning to test the peak flow."

: 4 Standing for the peak flow test provides for full expansion of the lungs; huff and cough clears the airways to allow for a complete full breath before testing a peak flow. Lying flat would not allow for full lung expansion and would not provide an accurate peak flow rate. Performing peak flow rates before exercise will help parents and children to determine if it would be safe to exercise. A peak flow rate tested after exercise is ineffective. Encouraging a child to test the peak flow meter every day and to set the meter at zero before testing is an accurate statement; however, recording peak flow rating on a daily basis is essential.

1397. A 4-year-old child is hospitalized and diagnosed with mild intermittent asthma. Oxygen is ordered via simple facemask. A nurse should plan to instruct the parents that which items can be harmful while oxygen is being administered? 1. Plastic blocks and handheld toys 2. Electronic educational toys and books 3. Cotton-filled toys and clothing 4. Synthetic toys and clothing

: 4 Synthetic toys and clothing are restricted during oxygen use because these items can build up static electricity, create a spark, and start a fire. Plastic blocks, handheld toys, electronic toys, books, cotton-filled toys, and clothing are safe and entertaining toys to provide children while they are receiving oxygen therapy.

1105. A nurse in a clinic is asked to teach a 13-year-old boy diagnosed with asthma. The nurse assesses that the child is developmentally on task. Which consideration should the nurse include when teaching this client? 1. The client is unable to differentiate cause and effect, so keep it simple. 2. The client is discovering new properties of objects and events, so expect many questions. 3. The client is not developmentally able to remember information, so handouts are necessary. 4. The client needs explanations of the physiology of asthma and demonstrations of appropriate interventions.

: 4 The adolescent has formal operational thought and can understand the diagnosis and appropriate interventions both verbally and by demonstration. All other considerations are necessary for younger children.

1098. A 10-month-old child reaches the 9- to 12-month developmental stage. Which nursing action is most appropriate for providing tactile stimulation for this child? 1. Caress the child while diaper changing 2. Give the child a soft squeeze toy 3. Swaddle the child at nap time 4. Let the child squash and mash food while sitting in a high chair

: 4 The most appropriate action is to let the child squash and mash food. At this age, an infant should be ready to touch and manipulate food and is capable of sitting up. Caressing a baby during diaper change, giving a baby a soft toy, and swaddling a baby at nap time are age-appropriate activities for providing tactile stimulation for younger babies.

1124. A pediatric nurse is to perform a head-to-toe assessment on a toddler who is admitted to a hospital for nausea and vomiting. Which is most important for the nurse to consider before beginning the examination? 1. Making sure the parents are present 2. Using a firm tone to settle the child down for the examination 3. Waiting until the child is ready to cooperate 4. Preparing for a physical examination based on the child's developmental age

: 4 The nurse should relate to the child based on the developmental age of the child. Considering the developmental level of the child will help the nurse to know how to proceed with the examination. Each individual family can decide which family members should be present. It may be perceived as threatening to the child to be told to settle down. Many times an assessment may have to be completed before a child is ready to cooperate.

1150. A pediatric client presents with tachycardia, edema, dyspnea, orthopnea, and crackles. A nurse performs a physical assessment of the client and notifies a physician immediately. Which condition does the nurse most likely suspect? 1. Right-sided heart failure 2. Rheumatic fever 3. Kawasaki disease 4. Left-sided heart failure

: 4 The nurse suspects that the client has left-sided heart failure. A child in left-sided heart failure will present with pulmonary symptoms of dyspnea, orthopnea, and crackles because of the fluid accumulation in the lungs from the ineffective pumping action of the heart. In right-sided heart failure, the child would present with jugular venous distention, liver enlargement, splenomegaly, or ascites due to a reduced preload to the right side of the heart. In rheumatic fever, the child would present with an elevated temperature and a systolic murmur, mitral insufficiency, and a prolonged PR and QT interval. In Kawasaki disease, the child would present with fever, rash, and lymph node enlargement.

1128. A nurse is performing a physical assessment of a pediatric client. While auscultating the heart, the nurse hears physiological splitting of S2 when the child takes a deep breath. Which action should be taken by the nurse? 1. Notify the provider of suspected atrial-septal defect. 2. Notify the provider of suspected pulmonary stenosis. 3. Follow institutional policy for initiating an emergency response. 4. Document the findings as a normal finding.

: 4 The physiological splitting of S2 on deep inspiration is normal in pediatric clients. Normally, the aortic valve closes just before the pulmonary valve, but the valves are so close together that the S2 sound is uniform and instantaneous. When a person takes in a deep breath, the decrease in intrathoracic pressure increases venous return. The right atrium and ventricle then fill slightly more than usual. With the extra blood, the pulmonary valve stays open slightly longer than usual, and the normally small difference between aortic and pulmonary valve closure becomes a noticeable split S2. If this occurs during normal respiration, it is abnormal and may indicate an atrial-septal defect or pulmonary stenosis. Initiating an emergency response is unnecessary because the findings are normal.

1270. A 4-year-old girl has been hospitalized for moderate burns. A nurse plans care based on knowing that the most developmentally appropriate response to the injuries and resultant treatment for a child this age is likely to be: 1. anger and hostility while trying to not appear young. 2. pushing boundaries to further autonomy. 3. wanting clear instructions regarding details of treatment. 4. believing that she is responsible for the bad things that are happening to her.

: 4 The preschool-aged child is increasingly verbal but has limitations. The preschool child may experience feelings of guilt, anxiety, and fear when thoughts or actions differ from perceived expected behavior. This may cause the child to believe that the injury was caused by that variation in expected behavior. Anger and hostility is an adolescent response. Pushing boundaries would be a response by a toddler. Wanting clear instructions would be characteristic of a school-aged child.

1376. A nurse is caring for a pediatric client with acute renal failure. A health-care practitioner prescribes a sodium biphosphate and sodium phosphate (Fleet®) enema. The next most appropriate nurse action is to: 1. administer the enema as ordered. 2. administer the enema as ordered after discussing the risks with the parent. 3. administer the enema as ordered after discussing the risks with the practitioner. 4. refuse administration of the enema without careful investigation because of the lethal risks.

: 4 The use of sodium biphosphate and sodium phosphate enemas in children with renal failure is potentially lethal because of hyperphosphatemia and should not be implemented without careful investigation. The other actions are incorrect because these include administering the enema.

1437. A nurse is caring for a 4-year-old child, diagnosed with sickle cell disease, who has undergone a splenectomy. The child is allergic to penicillin. Which medication should the nurse anticipate that the child will be required to take after discharge? 1. Morphine sulfate 2. Epoetin (Procrit®) 3. Amoxicillin (Amoxil®) 4. Erythromycin ethylsuccinate (EryPed®)

: 4 There is a decreased capability to fight infection following a splenectomy. Daily prophylactic antibiotics are administered. Erythromycin is of the macrolide class of antibiotics. Opioids are administered in sickle cell crises or for severe pain; its use depends on the severity of pain and may not be required. Epoetin stimulates the bone marrow to produce red blood cells (erythropoiesis). In sickle cell disease, the red blood cells "sickle," increasing the levels of hemoglobin S (HbS). Increasing the production of sickled red blood cells can worsen the condition. Amoxicillin is an aminopenicillin and is contraindicated when allergies to penicillin are present.

1210. A novice nurse is assessing a newly born infant with respiratory distress and copious oral secretions. The nurse's initial thought is tracheoesophageal atresia (TEA). Which nursing action should confirm this? 1. Respiratory distress decreases with oral suctioning. 2. Nasogastric tube (NG) easily advances to obtain stomach contents. 3. Catheter tip noted to be in the stomach on x-ray. 4. Soft NG advances, but unable to obtain stomach contents.

: 4 There is a pouch at the end of the esophagus that does not connect to the stomach, so the soft NG will not advance past the pouch. With TEA respiratory distress is caused by aspiration and will not improve with oral suctioning. In TEA a NG cannot be advanced to the stomach and would not be visible by x-ray.

1097. A nurse is caring for a 3-month-old infant. Based on the developmental age of the child, which motor skill should the nurse expect to see during an assessment? 1. Bangs objects held in hand 2. Begins to grab objects using a pincer grasp 3. Grabs objects using a palmar grasp 4. Looks and plays with own fingers

: 4 Three-month-old babies can play with their own fingers. At 3 months, infants can reach for attractive objects in front of them, but because their grasp is unpracticed, they usually miss them. A 2-month-old infant will hold an object for a few minutes. A 10-month-old infant uses a pincer grasp. A 6-month-old infant uses a palmar grasp.

1307. A malnourished child has cheilosis of the lips, burning and itching eyes, and seborrheic dermatitis. The child is diagnosed with a vitamin B2 (riboflavin) deficiency. Which additional findings on the assessment should a nurse deem consistent with the diagnosis? 1. Parasthesia 2. Irregular heart rate 3. Acanthosis nigricans 4. Irritation and cracks at the nasal angles of the nose

: 4 Vitamin B2 functions in maintaining healthy skin, especially around the mouth, nose, and eyes. Irritation and cracks at the nasal angles of the nose is consistent with the diagnosis of a vitamin B2 deficiency. Paresthesia, an abnormal sensation, is seen with vitamin B2 excess. Cardiac arrhythmias that could present with an irregular heart rate are seen in niacin excess. Acanthosis nigricans—a brown to black, poorly defined, velvety hyperpigmentation in skin folds of the neck or under the arms—are seen with niacin excess.

1234. Which nursing diagnosis should be the priority for a child hospitalized in sickle cell crisis? 1. Risk for deficient fluid volume related to inadequate fluid intake 2. Chronic pain related to chronic physical disability and clustering of sickled cells 3. Risk for infection related to ineffectively functioning spleen 4. Ineffective tissue perfusion related to pulmonary infiltrates of abnormal blood cells

: 4 While all are appropriate nursing diagnoses, ineffective tissue perfusion is priority because it can be life threatening. Acute chest syndrome is a common cause of hospitalization for sickle cell disease. The other nursing diagnoses are not life threatening.

1120. A school nurse is teaching adolescents about sexual activity and how the human papilloma virus (HPV) is contracted and prevented. Which statements should the nurse include when teaching about HPV? SELECT ALL THAT APPLY. 1. HPV can be contracted by oral sex. 2. HPV can be contracted on the toilet seat. 3. HPV is so common that most people can get it soon after becoming sexually active. 4. HPV is contracted through vaginal sex. 5. HPV is contracted through anal sex. 6. Condom use can fully protect against contracting the HPV virus.

: 4, 5 HPV is contracted by vaginal and anal sex and passed on by genital contact. It cannot be contracted by the other means, such as oral sex or the toilet seat. Condoms do not fully protect against contracting the HPV virus because HPV can infect areas that are not covered by a condom.

1420. A new nurse, under the supervision of an experienced nurse, is preparing to cannulate a 2-month-old infant's scalp vein for an intravenous infusion. In which order should the steps be performed to complete the procedure correctly. Prioritize the steps in the order that they should be performed by the new nurse. ______ Return in 60 minutes and reswaddle the infant in a mummy restraint. ______ Restrain the infant with a mummy restraint for preparing the site. ______ Cleanse the site over the temporal bone and shave the hair at the insertion site. ______ With an assistant holding the child's head, insert a scalp vein needle and observe for blood return. ______ Apply lidocaine/prilocaine cream (EMLA® cream) to the chosen site and unswaddle the infant after the EMLA application. ______ Ask the supervising nurse to turn the infant's head to the side and hold it firmly in position, with one hand on the occiput and the other on the front of the head. ______ Cleanse the shaved area with an antiseptic solution. ______ Remove the mummy restraint after initiating the infusion and comfort the infant. ______ Initiate the infusion and cover the infusion needle with a gauze dressing.

: 5, 1, 3, 7, 4, 2, 6, 9, 8 The first step should be to restrain the infant with a mummy restraint for preparing the site. The next step for the new nurse is to ask the supervising nurse to turn the infant's head to the side and hold it firmly in position, with one hand on the occiput and the other on the front of the head. Next, the new nurse should cleanse the site over the temporal bone and shave the hair at the insertion site, apply EMLA cream to the chosen site, and unswaddle the infant. The new nurse should return in 60 minutes, reswaddle the infant in a mummy restraint, and cleanse the shaved area with an antiseptic solution. With an assistant holding the child's head, the scalp vein needle should be inserted and observed for blood return. The new nurse should then initiate the infusion and cover the infusion needle with a gauze dressing. Finally, the mummy restraints should be removed after initiating the infusion and the infant comforted.

1421. An 8-year-old child with gastroenteritis is prescribed to receive 500 mL of lactated Ringer's (LR) solution over the next 10 hours. The rate in milliliters per hour that a nurse should infuse the solution is _______ mL.

: 50 The 50 mL per hour is obtained by dividing 10 hours into 500 mL of LR.

1430. A nurse is caring for a pediatric client experiencing wheezing from an asthma episode. A dose of albuterol (Proventil®) 5 mg by nebulization is prescribed. The medication vial contains 2.5 mg per 3 mL. The nurse should prepare ______ mL of medication for nebulization.

: 6 Use a proportion formula: 2.5 mg : 3 mL :: 5 mg : X mL Multiple the outside values and then the inside values and solve for X 2.5 X 15 X 6 mL

1203. A nurse is recording intake for a child hospitalized with diarrhea who has now begun to eat. The nurse should document ____ mL for the 3 ounces of popsicle that the child consumed.

: 90 One ounce is equal to 30 mL. Using the proportion formula: 1 oz : 30 mL :: 3 oz : X mL Multiply the means and the extremes. X 90 mL.


Related study sets

Chapter 18: Society and Politics in the Gilded Age, 1865-1900

View Set

Personal Financial Literacy - Chapter 12

View Set

Contraindication for Postural Drainage

View Set

C763 HEALTH INFORMATION SYSTEMS MANAGEMENT

View Set

Unit 40 Person Centered Planning

View Set

Talent Acquisition Mid Term Exam

View Set